Annotation of db/baza/ovsch14.txt, revision 1.4

1.1       rubashki    1: Чемпионат:
                      2: Открытый Всероссийский синхронный чемпионат - 2014/15
                      3: 
                      4: Дата:
                      5: 28-Sep-2014 - 01-Mar-2015
                      6: 
                      7: Тур:
                      8: Этап 1
                      9: 
                     10: Дата:
                     11: 28-Sep-2014
                     12: 
                     13: Редактор:
                     14: Александр Коробейников (Саратов), при участии Владимира Цвингли (Москва)
                     15: 
                     16: Инфо:
                     17: Редактор благодарит за тестирование вопросов Дмитрия Великова, Ивана
                     18: Ефремова, Михаила Иванова, Тимура Кафиатуллина, Владислава Короля,
                     19: Екатерину и Николая Лёгеньких, Дениса Маркова, Максима Мерзлякова,
                     20: Наталию Новыш, Игоря Петрова и Михаила Савченкова.
                     21: 
                     22: Вопрос 1:
                     23: В Рио-де-Жанейро собираются построить так называемый Музей Завтра. Его
                     24: архитектор Сантьяго Калатрава замечает, что музей должен содержать
                     25: ПЕРВЫЕ, а не ВТОРЫЕ. Через минуту напишите ВТОРОЙ.
                     26: 
                     27: Ответ:
                     28: Ответ.
                     29: 
                     30: Комментарий:
                     31: Музей будущего должен не давать ответы, а ставить вопросы.
                     32: 
                     33: Источник:
                     34:    1. http://www.bbc.com/travel/blog/20111201-the-futurist-rios-museum-of-tomorrow
                     35:    2. Лекция Сантьяго Калатравы в институте "Стрелка", 22.06.2014 г.
                     36: 
                     37: Автор:
                     38: Владимир Цвингли (Москва)
                     39: 
                     40: Вопрос 2:
                     41: В турецкой пословице упоминается не вода, а йогурт. Какое действие с ним
                     42: совершают?
                     43: 
                     44: Ответ:
                     45: На него дуют.
                     46: 
                     47: Зачет:
                     48: По слову "дуть" или однокоренным к нему.
                     49: 
                     50: Комментарий:
                     51: Русская пословица - "Обжегшись на молоке, будешь дуть и на воду". В
                     52: турецкой пошли еще дальше: там упоминается продукт, всегда употребляемый
                     53: холодным.
                     54: 
                     55: Источник:
                     56:    1. http://turkishcrossroads.wordpress.com/2011/05/12/more-turkish-idioms/
                     57:    2. http://www.kolikler.com/liste/cevap/8281/yogurt/yogurtla-ilgili-deyim-ve-atasozleri-nelerdir
                     58:    3. http://dic.academic.ru/dic.nsf/michelson_new/6797/
                     59: 
                     60: Автор:
                     61: Владимир Цвингли (Москва)
                     62: 
                     63: Вопрос 3:
                     64: На одном логотипе буквы "l" [эль], "f" [эф] и "g" [джи] имеют вид
                     65: ПЕРВЫХ, а еще одна буква выполнена в виде ВТОРОЙ. Назовите ПЕРВУЮ и
                     66: ВТОРУЮ, каждую тремя словами.
                     67: 
                     68: Ответ:
                     69: Клюшка для гольфа, лунка для гольфа.
                     70: 
                     71: Комментарий:
                     72: Так слово "golf" изображено на эмблеме одного из гольф-клубов.
                     73: 
                     74: Источник:
                     75: http://www.adme.ru/tvorchestvo-dizajn/30-umnyh-logotipov-700760/
                     76: 
                     77: Автор:
                     78: Александр Коробейников (Саратов)
                     79: 
                     80: Вопрос 4:
                     81: В фильме Франсуа Озона похититель удерживает девушку в подвале. В одной
                     82: из сцен эта героиня долго смотрит на НЕЕ. ОНА - это секрет. Назовите ЕЕ.
                     83: 
                     84: Ответ:
                     85: Паутина.
                     86: 
                     87: Комментарий:
                     88: Положение девушки сходно с положением пойманного насекомого в паутине.
                     89: Секрет - это выделяемая животным жидкость, результат секреции.
                     90: 
                     91: Источник:
                     92:    1. Х/ф "Криминальные любовники" (1999), реж. Ф. Озон, 43:35.
                     93:    2. http://ru.wikipedia.org/wiki/Паутина
                     94: 
                     95: Автор:
                     96: Владимир Цвингли (Москва)
                     97: 
                     98: Вопрос 5:
                     99: Лирического героя одной из своих песен Ник Кейв называет Ларри и
                    100: утверждает, что этот герой смог добиться большего, чем Гарри Гудини. Под
                    101: каким именем этого Ларри знаем мы?
                    102: 
                    103: Ответ:
                    104: Лазарь.
                    105: 
                    106: Комментарий:
                    107: Лазарь смог воскреснуть и выбраться из могилы. Гарри Гудини известен
                    108: схожими трюками, но до уровня Лазаря ему далеко!
                    109: 
                    110: Источник:
                    111: http://www.bbc.co.uk/music/reviews/w8p2
                    112: 
                    113: Автор:
                    114: Владимир Цвингли (Москва)
                    115: 
                    116: Вопрос 6:
                    117: Петр I считал Карла XII надменным юнцом, не способным управлять страной.
                    118: Сюжет о каком герое стал главной декоративной темой оформления корабля
                    119: "Полтава"?
                    120: 
                    121: Ответ:
                    122: О Фаэтоне.
                    123: 
                    124: Комментарий:
                    125: Петр же себя ассоциировал с Зевсом, наказавшим самонадеянного мальчишку.
                    126: Фаэтон оказался не способен управлять колесницей своего отца Гелиоса и
                    127: потерпел крах, как и Карл XII под Полтавой.
                    128: 
                    129: Источник:
                    130:    1. http://ru.wikipedia.org/wiki/Полтава_(линейный_корабль,_1712)
                    131:    2. http://poltava1712.ru/sites/default/files/vse_risunki_dzheniya-14_0.jpg
                    132: 
                    133: Автор:
                    134: Александр Коробейников (Саратов)
                    135: 
                    136: Вопрос 7:
                    137:    <раздатка>
                    138:    It came with my Pea Sea.
                    139:    It plane lee marks four my revue
                    140:    Miss Steaks I can knot sea.
                    141:    </раздатка>
                    142:    Перед вами отрывок написанного в 1992 году стихотворения Джеррольда
                    143: ЗАра. Ответьте словом с двумя английскими корнями, недостатки чего хотел
                    144: продемонстрировать Зар.
                    145: 
                    146: Ответ:
                    147: Спеллчекера.
                    148: 
                    149: Зачет:
                    150: Spellchecker.
                    151: 
                    152: Комментарий:
                    153: Стихотворение составлено из набора слов, созвучного осмысленному тексту.
                    154: Первые спеллчекеры работали, сравнивая каждое из встреченных слов со
                    155: своим словарем, но не учитывая связь слов в предложении.
                    156: 
                    157: Источник:
                    158: http://en.wikipedia.org/wiki/Spell_checker
                    159: 
                    160: Автор:
                    161: Владимир Цвингли (Москва)
                    162: 
                    163: Вопрос 8:
                    164: Герой Андрея Гребенщикова говорит, что люди, родившиеся в метро, начисто
                    165: лишены перспективы, и упоминает ИКС. ИКСУ посвящена серия других
                    166: романов. Назовите ИКС двумя словами.
                    167: 
                    168: Ответ:
                    169: Плоский мир.
                    170: 
                    171: Комментарий:
                    172: В данном случае перспектива - это еще и умение видеть объем и
                    173: многогранность, а мир метро кажется герою двумерным, плоским. Плоский
                    174: Мир - место действия многих романов Терри Пратчетта.
                    175: 
                    176: Источник:
                    177: А. Гребенщиков. Обитель снов. http://www.flibusta.net/b/325458/read
                    178: 
                    179: Автор:
                    180: Артем Викторов (Саратов)
                    181: 
                    182: Вопрос 9:
                    183: В начале двадцатого века в Италии были популярны карандаши, красные с
                    184: одного конца и синие с другого. Что основал художник Джорджо МунджиАни,
                    185: придерживавшийся космополитических взглядов?
                    186: 
                    187: Ответ:
                    188: Клуб "Интернационале" (Милан).
                    189: 
                    190: Зачет:
                    191: По словам "Интернационале" или "Интер".
                    192: 
                    193: Комментарий:
                    194: Ориентируясь на форму уже существовавшего клуба "Милан", Мунджиани
                    195: оставил в ней черный цвет, но дополнил его синим в противовес красному
                    196: цвету "Милана".
                    197: 
                    198: Источник:
                    199: http://www.sports.ru/tribuna/blogs/marinamargarita/669690.html
                    200: 
                    201: Автор:
                    202: Александр Коробейников (Саратов)
                    203: 
                    204: Вопрос 10:
                    205: Герой одного сериала - шахматист по фамилии БАлаган, который играет
                    206: только по Интернету. Его соперники, не знавшие его в лицо, дали ему
                    207: прозвище, заменив в его фамилии один слог двумя другими. Напишите это
                    208: прозвище.
                    209: 
                    210: Ответ:
                    211: Балаклава.
                    212: 
                    213: Комментарий:
                    214: Маски-балаклавы закрывают лицо, оставляя только глаза.
                    215: 
                    216: Источник:
                    217: Телесериал "Шах и мат", 6-я серия.
                    218: 
                    219: Автор:
                    220: Александр Коробейников (Саратов)
                    221: 
                    222: Вопрос 11:
                    223: Известный путешественник Грэм Хьюз уверяет, что без этой тяжелой, а
                    224: порой мокрой и дурно пахнущей штуки можно и обойтись. Что же Хьюз,
                    225: вопреки известному источнику, рекомендует не брать с собой?
                    226: 
                    227: Ответ:
                    228: Полотенце.
                    229: 
                    230: Комментарий:
                    231: В отличие от Дугласа Адамса, автора "Автостопом по Галактике", который
                    232: писал, что полотенце - неотъемлемый элемент экипировки автостопщика,
                    233: Хьюз считает, что без него легко обойтись.
                    234: 
                    235: Источник:
                    236: "Discovery", 2014, N 5. - С. 103.
                    237: 
                    238: Автор:
                    239: Александр Коробейников (Саратов)
                    240: 
                    241: Вопрос 12:
                    242: Дизайнер ДжОшуа РенУф совместил будильник с НЕЙ, и просыпается хозяин
                    243: этого агрегата не только от звукового сигнала. Назовите ЕЕ двукоренным
                    244: словом.
                    245: 
                    246: Ответ:
                    247: Кофемашина.
                    248: 
                    249: Зачет:
                    250: Кофеварка; кофемолка.
                    251: 
                    252: Комментарий:
                    253: К пробуждению автомат готовит чашечку ароматного кофе, от запаха
                    254: которого хозяин и просыпается. Впрочем, звуковой сигнал для контроля
                    255: тоже есть, хотя и довольно тихий. Возможно, в перерыве некоторые игроки
                    256: успеют попить кофе.
                    257: 
                    258: Источник:
                    259: http://www.adme.ru/tvorchestvo-dizajn/etot-budilnik-nalet-vam-s-utra-chashechku-kofe-736610/
                    260: 
                    261: Автор:
                    262: Александр Коробейников (Саратов)
                    263: 
                    264: Вопрос 13:
                    265: В пояснительной табличке к перламутровой чаше на одной выставке автор
                    266: вопроса увидел название, получившее широкую известность в СССР в
                    267: середине 80-х годов. Напишите это название.
                    268: 
                    269: Ответ:
                    270: Nautilus [Pompilius].
                    271: 
                    272: Зачет:
                    273: "Наутилус [Помпилиус]".
                    274: 
                    275: Комментарий:
                    276: Табличка сообщала, что для перламутровых инкрустированных чаш обычно
                    277: использовали раковину моллюска вида Nautilus pompilius. Советская и
                    278: российская рок-группа в 1985 году сменила название с "Наутилус" на
                    279: "Nautilus Pompilius".
                    280: 
                    281: Источник:
                    282:    1. Выставка "Сокровищница Медичи. Из собрания Музея Серебра и других
                    283: музеев Флоренции".
                    284:    2. http://www.kreml.ru/exhibitions/moscow-kremlin-exhibitions/sokrovishchnitsa-medichi-iz-sobraniya-muzeya-serebra-i-drugikh-muzeev-florentsii/
                    285:    3. А.К. Троицкий. Рок-музыка в СССР. - М.: Книга, 1990. - С. 239-240.
                    286: 
                    287: Автор:
                    288: Владимир Цвингли (Москва)
                    289: 
                    290: Вопрос 14:
                    291: Герой документального фильма в юности был членом Армии Крайовой. По
                    292: словам Михаила ТрофимЕнкова, он выжил, а не обратился в ПЕРВЫЙ, чтобы
                    293: возродиться ВТОРЫМ. Назовите ПЕРВЫЙ и ВТОРОЙ.
                    294: 
                    295: Ответ:
                    296: Пепел, алмаз.
                    297: 
                    298: Комментарий:
                    299: Герой фильма "Пепел и алмаз", как и герой описанного в вопросе
                    300: документального фильма "Тихая заводь", тоже входил в антифашистское и
                    301: антикоммунистическое польское подполье, но погиб.
                    302: 
                    303: Источник:
                    304: http://seance.ru/n/21-22/vertigo-2/prezumptsiya-vinovnosti/
                    305: 
                    306: Автор:
                    307: Владимир Цвингли (Москва)
                    308: 
                    309: Вопрос 15:
                    310: Игорь Можейко пишет, что своей славой ОН обязан тому, что сделал всё сам
                    311: - без помощи армии, жрецов и аппарата принуждения. Назовите ЕГО имя.
                    312: 
                    313: Ответ:
                    314: Герострат.
                    315: 
                    316: Комментарий:
                    317: Храмы сжигали и раньше, но Герострат обошелся собственными силами.
                    318: 
                    319: Источник:
                    320: И.В. Можейко. 7 и 37 чудес. http://www.flibusta.net/b/351970/read
                    321: 
                    322: Автор:
                    323: Александр Коробейников (Саратов)
                    324: 
                    325: Вопрос 16:
                    326: Герой одного романа, чтобы устрашить преследующих его ночных животных,
                    327: делает с одним из них то же, что известный человек, по его собственным
                    328: словам, сделал по ошибке. Назовите этого человека.
                    329: 
                    330: Ответ:
                    331: [Оззи] Осборн.
                    332: 
                    333: Комментарий:
                    334: Эти существа - летучие мыши, одной из которых герой и откусывает голову.
                    335: Подобным же поступком известен и Оззи Осборн, хотя, по его собственному
                    336: мнению, считал, что летучая мышь резиновая.
                    337: 
                    338: Источник:
                    339: Р. Глушков. Грань Бездны. http://www.litmir.net/br/?b=154646&p=71
                    340: 
                    341: Автор:
                    342: Артем Викторов (Саратов)
                    343: 
                    344: Вопрос 17:
                    345: В Нью-Йорке находятся штаб-квартиры и представительства многих
                    346: американских компаний, работающих в сфере компьютерных технологий. Хотя
                    347: они расположены и не на одной улице, этот конгломерат получил состоящее
                    348: из двух слов прозвище. Напишите его по-английски.
                    349: 
                    350: Ответ:
                    351: Silicon Alley [сИликон Элли].
                    352: 
                    353: Комментарий:
                    354: Буквально - "Кремниевая аллея", по аналогии с Кремниевой долиной
                    355: (Silicon Valley) в Калифорнии.
                    356: 
                    357: Источник:
                    358: http://en.wikipedia.org/wiki/Silicon_Alley
                    359: 
                    360: Автор:
                    361: Владимир Цвингли (Москва)
                    362: 
                    363: Вопрос 18:
                    364: Во время оккупации Варшавы немцами герою книги Джека Майера удалось
                    365: раздобыть кусок дефицитного угля, который его подруга сравнила с НЕЙ.
                    366: Другую "ЕЕ" сыграл "Сансет". Назовите ЕЕ двумя словами.
                    367: 
                    368: Ответ:
                    369: Черная жемчужина.
                    370: 
                    371: Комментарий:
                    372: Уголь представлял собой настоящую драгоценность. Корабль "Черная
                    373: жемчужина" из франшизы "Пираты Карибского моря" создан на основе корабля
                    374: HMS Sunset.
                    375: 
                    376: Источник:
                    377:    1. Дж. Майер. Храброе сердце Ирены Сендлер.
                    378: http://www.flibusta.net/b/351708/read
                    379:    2. http://pirates.wikia.com/wiki/Sunset_(ship)
                    380: 
                    381: Автор:
                    382: Александр Коробейников (Саратов)
                    383: 
                    384: Вопрос 19:
                    385: Во время прОводов родственника герой одного романа называет ИКСА злобным
                    386: таксистом. Какое имя мы заменили ИКСОМ?
                    387: 
                    388: Ответ:
                    389: Харон.
                    390: 
                    391: Комментарий:
                    392: Родственника провожали в последний путь и основательно потратились.
                    393: 
                    394: Источник:
                    395: А. Гребенщиков. Обитель снов. http://www.flibusta.net/b/325458/read
                    396: 
                    397: Автор:
                    398: Артем Викторов (Саратов)
                    399: 
                    400: Вопрос 20:
                    401: Алексей Леонов был зачислен в первый отряд космонавтов и, конечно,
                    402: мечтал первым отправиться в космос. Интересно, что среди его картин есть
                    403: историческое полотно "ИКС в Антарктиде". Назовите ИКС.
                    404: 
                    405: Ответ:
                    406: Шлюп "Восток".
                    407: 
                    408: Зачет:
                    409: Корабль "Восток"; "Восток".
                    410: 
                    411: Комментарий:
                    412: Леонов наверняка хотел полететь на космическом корабле "Восток" (а в
                    413: итоге слетал на "Восходе-2"). Шлюпом "Восток" командовал Беллинсгаузен,
                    414: открыватель Антарктиды.
                    415: 
                    416: Источник:
                    417:    1. http://www.seaexpo.ru/ru/miscellaneous/company-news/1_849.html
                    418:    2. http://pyatkovvova.narod.ru/index/0-381
                    419: 
                    420: Автор:
                    421: Александр Коробейников (Саратов)
                    422: 
                    423: Вопрос 21:
                    424: Рэпер Мистер Си в одной песне собирается на завтрак залить себе овсяные
                    425: хлопья ИМ. Согласно одному мифу, ОН появился из-за того, что героине
                    426: было тяжело каждый день возвращаться домой пешком. Назовите ЕГО.
                    427: 
                    428: Ответ:
                    429: Млечный Путь.
                    430: 
                    431: Комментарий:
                    432: Рэпер несколько преувеличивает: он хочет просто много молока. Героиня
                    433: мифа создала реку и с тех пор плавала по ней в лодке.
                    434: 
                    435: Источник:
                    436:    1. http://webkind.ru/text/967964471_1556824p524440981_text_pesni_pentimento-bozze-music.html
                    437:    2. http://www.myfhology.info/stella-myth/milk-way.html
                    438: 
                    439: Автор:
                    440: Артем Викторов (Саратов)
                    441: 
                    442: Вопрос 22:
                    443: [Ведущему: не сообщать о наличии отточия, оно не имеет значения для
                    444: вопроса.]
                    445:    Выражение "Во всём виноваты евреи и велосипедисты" происходит из
                    446: романа Ремарка "Черный обелиск". Вот цитата из романа:
                    447:    - При чем тут велосипедисты? - <...> удивляется Генрих.
                    448:    - А при чем тут евреи? - ДЕЛАЕТ ЭТО Ризенфельд.
                    449:    Какие четыре слова мы заменили словами "ДЕЛАЕТ ЭТО"?
                    450: 
                    451: Ответ:
                    452: Отвечает вопросом на вопрос.
                    453: 
                    454: Комментарий:
                    455: Забавно, что Ризенфельд, защищающий евреев в диалоге, делает то же, что
                    456: в обыденном представлении часто делают евреи.
                    457: 
                    458: Источник:
                    459: Э.М. Ремарк. Черный обелиск. http://www.flibusta.net/b/326124/read
                    460: 
                    461: Автор:
                    462: Владимир Цвингли (Москва)
                    463: 
                    464: Вопрос 23:
                    465: Родовитый князь из романа Евгения Водолазкина не имел детей, так что
                    466: образно называл себя ИКСОМ. ИКС упоминается в Евангелиях от Луки и
                    467: Матфея. Назовите ИКС одним словом.
                    468: 
                    469: Ответ:
                    470: Сучок.
                    471: 
                    472: Комментарий:
                    473: Князь называл себя сучком на генеалогическом древе своего рода.
                    474: Противопоставление сучка в чужом глазу и бревна в собственном восходит к
                    475: Евангелию от Луки.
                    476: 
                    477: Источник:
                    478:    1. Е.Г. Водолазкин. Похищение Европы.
                    479: http://www.flibusta.net/b/338557/read
                    480:    2. http://www.allbible.info/bible/sinodal/lu/6/
                    481:    3. http://www.allbible.info/bible/sinodal/mt/7
                    482: 
                    483: Автор:
                    484: Александр Коробейников (Саратов)
                    485: 
                    486: Вопрос 24:
                    487: Персонаж одного сериала отмечает, что у НИХ прибавилось работы после
                    488: Первой мировой войны. ФрЕдерик МАйерс считал, что ИХ работа связана с
                    489: автоматическим проявлением подсознательного. Назовите ИХ.
                    490: 
                    491: Ответ:
                    492: Медиумы.
                    493: 
                    494: Зачет:
                    495: Спириты.
                    496: 
                    497: Комментарий:
                    498: Первая мировая война принесла множество смертей, и многие захотели
                    499: пообщаться со своими погибшими родными и близкими. Майерс считал, что
                    500: многие явления деятельности медиума связаны с автоматическим проявлением
                    501: его подсознания.
                    502: 
                    503: Источник:
                    504:    1. Телесериал "Леди-детектив Фрайни Фишер", s02e01.
                    505:    2. http://ru.wikipedia.org/wiki/Медиум
                    506: 
                    507: Автор:
                    508: Александр Коробейников (Саратов)
                    509: 
                    510: Вопрос 25:
                    511: Полина Павлова пишет, что на выборах некоторые кандидаты, надеясь на
                    512: одобрение избирателей, подчеркивают, что у них имеется ЭТО. ЭТО имеет
                    513: "команда". Назовите ЭТО.
                    514: 
                    515: Ответ:
                    516: [Буквосочетание] ДА.
                    517: 
                    518: Комментарий:
                    519: КандиДАт предлагает сказать себе "ДА", при этом особенно везет
                    520: кандидатам, которые имеют нужное сочетание букв и в фамилии. Часто
                    521: спекулируют и на добавлении в лозунги слова "правДА". В слове "команда"
                    522: тоже есть слог "ДА".
                    523: 
                    524: Источник:
                    525: http://www.tverlife.ru/news/11181.html
                    526: 
                    527: Автор:
                    528: Илья Токарев (Заречный)
                    529: 
                    530: Вопрос 26:
                    531: Кличка призрака, якобы живущего в известном здании, - сокращение от слов
                    532: "демонический кот". В каком городе находится это здание?
                    533: 
                    534: Ответ:
                    535: Вашингтон.
                    536: 
                    537: Комментарий:
                    538: Кличка - DC [ди-си], как и сокращенное название округа Колумбия, в
                    539: котором находится Вашингтон. Призрак кота якобы периодически появляется
                    540: в Белом доме.
                    541: 
                    542: Источник:
                    543: http://sfilm.org/articles/010/демонический-кот-и-другие-привидения/
                    544: 
                    545: Автор:
                    546: Александр Коробейников (Саратов)
                    547: 
                    548: Вопрос 27:
                    549: Во время Первой мировой войны ЗИгберт ТАрраш написал пародию на
                    550: шовинистическую истерию на примере анализа шахматных дебютов. Так, он
                    551: подверг критике французскую защиту и английское начало. Еще один дебют
                    552: Тарраш охарактеризовал восторженно и дал ему название, которое всего на
                    553: одну букву длиннее реально существующего названия другого дебюта.
                    554: Напишите это новое название.
                    555: 
                    556: Ответ:
                    557: Прусская партия.
                    558: 
                    559: Комментарий:
                    560: "Прусской партией" Тарраш в духе немецкого патриотизма назвал защиту
                    561: двух коней (впрочем, мог бы назвать так и любой другой дебют). Русскую
                    562: партию он в статье, естественно, тоже критикует.
                    563: 
                    564: Источник:
                    565: Ю.Л. Авербах. О чем молчат фигуры. http://www.flibusta.net/b/236155/read
                    566: 
                    567: Автор:
                    568: Владимир Цвингли (Москва)
                    569: 
                    570: Вопрос 28:
                    571: Герои Йосипа НовАковича выдернули почти все флажки с уличных растяжек
                    572: перед первомайской демонстрацией. Растяжки после этого сравниваются с
                    573: АЛЬФАМИ старика. Действие "АЛЬФ" происходит в основном на острове.
                    574: Назовите АЛЬФЫ.
                    575: 
                    576: Ответ:
                    577: Челюсти.
                    578: 
                    579: Комментарий:
                    580: Остались только некоторые флажки, как редкие зубы во рту у старика.
                    581: Неудивительно, что в фильме про акул место действия - остров.
                    582: 
                    583: Источник:
                    584:    1. Й. Новакович. День дурака. http://www.flibusta.net/b/273308/read
                    585:    2. http://www.kinopoisk.ru/film/396/
                    586: 
                    587: Автор:
                    588: Александр Коробейников (Саратов)
                    589: 
                    590: Вопрос 29:
                    591: Герой одного фильма сначала выиграл много денег у бандитов, а затем
                    592: снова встретился с ними за карточным столом. Герой с горькой иронией и
                    593: вместе с тем с оптимизмом заметил, что он - ИКС. Какое слово, оба корня
                    594: в котором латинские, мы заменили ИКСОМ?
                    595: 
                    596: Ответ:
                    597: Амбидекстр.
                    598: 
                    599: Комментарий:
                    600: Бандиты из мести сломали герою руку, однако он, по счастью, владеет
                    601: другой рукой так же хорошо. Слово "амбидекстр" происходит от слов "оба"
                    602: и "правый". Хотя автомат "однорукий бандит" никакого отношения к вопросу
                    603: не имеет, но может помочь его взять.
                    604: 
                    605: Источник:
                    606: Х/ф "Город грехов - 2" (2014), реж. Ф. Миллер, Р. Родригес.
                    607: 
                    608: Автор:
                    609: Артем Викторов (Саратов)
                    610: 
                    611: Вопрос 30:
                    612: В статье пятой Уголовно-процессуального кодекса России даны определения
                    613: слов и выражений, использующихся в кодексе. Назовите несклоняемым словом
                    614: определяемое в этой статье понятие, которое ни в одном другом месте
                    615: кодекса не встречается.
                    616: 
                    617: Ответ:
                    618: Алиби.
                    619: 
                    620: Комментарий:
                    621: Раньше оно встречалось еще и в части 6 статьи 234 кодекса. Однако
                    622: законом от 3 июня 2006 года статья была признана утратившей силу, а
                    623: определение осталось. "Alibi" переводится с латыни как "в другом месте".
                    624: 
                    625: Источник:
                    626:    1. http://www.zakonrf.info/upk/5/
                    627:    2. Поиск по: Уголовно-процессуальный кодекс Российской Федерации. СПС
                    628: "Консультант Плюс".
                    629: 
                    630: Автор:
                    631: Владимир Цвингли (Москва)
                    632: 
                    633: Вопрос 31:
                    634: В 2013 году Национальный исторический музей Румынии провел серию
                    635: мероприятий под названием "ОНА MCM [эм-це-эм]". Назовите ЕЕ.
                    636: 
                    637: Ответ:
                    638: Колонна.
                    639: 
                    640: Зачет:
                    641: Колонна Траяна, Траянова колонна.
                    642: 
                    643: Комментарий:
                    644: Праздновалась 1900-летняя годовщина со времени сооружения колонны
                    645: Траяна.
                    646: 
                    647: Источник:
                    648: http://www.mnir.ro/index.php/columna-mcm-celebrarea-a-1900-de-ani-de-la-ridicarea-columnei-lui-traian/
                    649: 
                    650: Автор:
                    651: Александр Коробейников (Саратов)
                    652: 
                    653: Вопрос 32:
                    654: На концепцию ресторана "Twin Stars" [твин старз] его создателя Алексея
                    655: Ходорковского натолкнуло королевство. Какое именно?
                    656: 
                    657: Ответ:
                    658: Кривых зеркал.
                    659: 
                    660: Комментарий:
                    661: В ресторане все официантки и бармены - близнецы.
                    662: 
                    663: Источник:
                    664: http://www.uznayvse.ru/ekonomika/v-stolichnom-restorane-twin-stars-posetiteley-obsluzhivayut-bliznetsyi-55475.html
                    665: 
                    666: Автор:
                    667: Александр Коробейников (Саратов)
                    668: 
                    669: Вопрос 33:
                    670: Леонид Костюков называет бессмысленные, повторяющие одно и то же
                    671: рассуждения доступными. Какие две буквы мы пропустили в этом вопросе?
                    672: 
                    673: Ответ:
                    674: во.
                    675: 
                    676: Комментарий:
                    677: Рассуждающий словно воду в ступе толчет.
                    678: 
                    679: Источник:
                    680: http://ps.1september.ru/article.php?ID=200005708
                    681: 
                    682: Автор:
                    683: Александр Коробейников (Саратов)
                    684: 
                    685: Вопрос 34:
                    686: Собеседник героини Нила Стивенсона хочет говорить только о технике, а
                    687: когда речь заходит, например, о стихах, умолкает. Какой фамилией этот
                    688: собеседник представляется?
                    689: 
                    690: Ответ:
                    691: Тьюринг.
                    692: 
                    693: Комментарий:
                    694: На самом деле собеседник - компьютер.
                    695: 
                    696: Источник:
                    697: http://www.erlib.com/Нил_Стивенсон/Алмазный_век,_или_Букварь_для_благородных_девиц/26/
                    698: 
                    699: Автор:
                    700: Александр Коробейников (Саратов)
                    701: 
                    702: Вопрос 35:
                    703: Герой Сири Джеймс предлагает героине начать всё сначала - так сказать,
                    704: СДЕЛАТЬ ЭТО. ЭТО часто ДЕЛАЮТ, например, перед русским языком. Что такое
                    705: "СДЕЛАТЬ ЭТО"?
                    706: 
                    707: Ответ:
                    708: Вытереть доску.
                    709: 
                    710: Зачет:
                    711: Стереть с доски.
                    712: 
                    713: Комментарий:
                    714: Так сказать, изготовить tabula rasa.
                    715: 
                    716: Источник:
                    717: С. Джеймс. Тайные дневники Шарлотты Бронте.
                    718: http://www.flibusta.net/b/298085/read
                    719: 
                    720: Автор:
                    721: Александр Коробейников (Саратов)
                    722: 
                    723: Вопрос 36:
                    724: Латинское выражение "сказано с НЕГО" обозначает какое-то темное и
                    725: непонятное выражение. Мистики утверждают, что ОН символизирует
                    726: восходящее, полуденное и заходящее солнце. Назовите ЕГО.
                    727: 
                    728: Ответ:
                    729: Треножник.
                    730: 
                    731: Комментарий:
                    732: Дельфийская пифия вещала с треножника и давала как можно более
                    733: обтекаемые и туманные ответы. На них мы, кстати, и намекаем вторым
                    734: фактом: что имеют в виду астрологи, не очень понятно - на то они и
                    735: астрологи, зато сущностей здесь три.
                    736: 
                    737: Источник:
                    738:    1. http://dic.academic.ru/dic.nsf/michelson_old/15570/
                    739:    2. http://www.vrata11.ru/index.php?option=com_content&task=view&id=232&Itemid=74
                    740: 
                    741: Автор:
                    742: Александр Коробейников (Саратов)
                    743: 
                    744: Тур:
                    745: Этап 2
                    746: 
                    747: Дата:
                    748: 26-Oct-2014
                    749: 
                    750: Редактор:
                    751: Александр Кудрявцев (Николаев)
                    752: 
                    753: Инфо:
                    754: Автор игрового пакета благодарит за тестирование и ценные замечания:
                    755: Дмитрия Алёхина, Тимура Барского, Евгения Быстрова, Александра Ваксмана,
                    756: Галину Воловник, Владислава Говердовского, Владимира Городецкого,
                    757: Евгения Громова, Людмилу Губаеву, Антона Гусакова, Андрея Данченко,
                    758: Глеба Дрепина, Екатерину Дубровскую, Игоря Мазина, Вадима Молдавского,
                    759: Дмитрия Овчарука, Наталью Руберте, Дмитрия Слоуща, Екатерину Сосенко и
                    760: Дмитрия Чеснокова. Персональная благодарность Александру Коробейникову и
                    761: Максиму Мерзлякову.
                    762: 
                    763: Вопрос 1:
                    764: По мнению героя романа "2001: Космическая одиссея", эти пять цифр
                    765: слишком дорого стоили человеческим нервам. Напишите эти цифры в
                    766: правильном порядке.
                    767: 
                    768: Ответ:
                    769: 5, 4, 3, 2, 1.
                    770: 
                    771: Зачет:
                    772: 4, 3, 2, 1, 0.
                    773: 
                    774: Комментарий:
                    775: Герой Кларка имеет в виду обратный отсчет при запуске космического
                    776: корабля.
                    777: 
                    778: Источник:
                    779: Артур Кларк. 2001: Космическая одиссея.
                    780: 
                    781: Автор:
                    782: Александр Кудрявцев (Николаев)
                    783: 
                    784: Вопрос 2:
                    785: Эти аппараты состоят из нагревателя, в котором плавится сырье,
                    786: крутящейся головки, выбрасывающей волокно, и емкости-улавливателя. Что
                    787: производят с помощью этих аппаратов?
                    788: 
                    789: Ответ:
                    790: Сахарную вату.
                    791: 
                    792: Зачет:
                    793: Сладкую вату.
                    794: 
                    795: Источник:
                    796: "Галилео", 04.05.2007 г.
                    797: 
                    798: Автор:
                    799: Александр Кудрявцев (Николаев)
                    800: 
                    801: Вопрос 3:
                    802: Среди причин исчезновения голубых ара - резкое увеличение численности
                    803: диких ИХ, заселивших привычные для этого вида попугаев места
                    804: гнездования. ОНИ дали название модели аппарата для производства сахарной
                    805: ваты. Назовите ИХ.
                    806: 
                    807: Ответ:
                    808: Пчелы.
                    809: 
                    810: Комментарий:
                    811: Завезенные в Америку африканские пчелы захватили все дупла, в которых
                    812: привыкли устраивать гнезда голубые ара. Аппарат получил название
                    813: "Пчелка".
                    814: 
                    815: Источник:
                    816:    1. http://ru.wikipedia.org/wiki/Голубой_ара
                    817:    2. http://www.food-service.com.ua/apparat-saharnoy-vaty-pchelka-e-p71326
                    818: 
                    819: Автор:
                    820: Александр Кудрявцев (Николаев)
                    821: 
                    822: Вопрос 4:
                    823: Названием детища известной IT-компании [ай-ти-компании] стал глагол,
                    824: восходящий к слову со значением "свеча". Это не преминули подметить
                    825: язвительные комментаторы, упомянув известную антиутопию. Назовите
                    826: упомянутое детище.
                    827: 
                    828: Ответ:
                    829: (Amazon) Kindle [амазон киндл].
                    830: 
                    831: Комментарий:
                    832: "Amazon Kindle" - электронная книга компании "Amazon". Глагол "kindle",
                    833: означающий "зажигать огонь", происходит от древнескандинавского
                    834: "kyndill" [киндил], которое, кстати, не имеет отношения к латинскому
                    835: "candela" [кандела]. Создатели имели в виду свет знаний, который несут
                    836: книги. Комментаторы шутили, что бумажные книги теперь можно сжечь, и
                    837: упоминали антиутопию "451 градус по Фаренгейту".
                    838: 
                    839: Источник:
                    840:    1. http://www.etymonline.com/index.php?term=kindle&allowed_in_frame=0
                    841:    2. http://nancyfriedman.typepad.com/away_with_words/2008/12/how-the-kindle-got-its-name.html
                    842: 
                    843: Автор:
                    844: Александр Кудрявцев (Николаев)
                    845: 
                    846: Вопрос 5:
                    847: Считается, что прообразом ИКСА стала МавронЕри, населенная
                    848: микроорганизмами, выделяющими смертельно опасные токсины. Назовите ИКС
                    849: односложным словом.
                    850: 
                    851: Ответ:
                    852: Стикс.
                    853: 
                    854: Комментарий:
                    855: МавронЕри - река на Пелопоннесе, название которой переводится как
                    856: "Черная вода".
                    857: 
                    858: Источник:
                    859:    1. http://en.wikipedia.org/wiki/Mavroneri
                    860:    2. http://en.wikipedia.org/wiki/Calicheamicin
                    861:    3. http://listverse.com/2014/09/29/10-real-world-entrances-to-mythical-locations/
                    862: 
                    863: Автор:
                    864: Александр Кудрявцев (Николаев)
                    865: 
                    866: Вопрос 6:
                    867: В телепрограмме "Человеческий фактор" ЭТО было названо надводной частью
                    868: айсберга. "Подводную" часть составляют, в том числе, три однотипных
                    869: элемента. Назовите тот, название которого не содержит уменьшительного
                    870: суффикса.
                    871: 
                    872: Ответ:
                    873: Наковальня.
                    874: 
                    875: Комментарий:
                    876: ЭТО - ушная раковина. Молоточек, наковальня и стремечко - косточки
                    877: слухового аппарата.
                    878: 
                    879: Источник:
                    880: Человеческий фактор. Звуки музыки.
                    881: 
                    882: Автор:
                    883: Александр Кудрявцев (Николаев)
                    884: 
                    885: Вопрос 7:
                    886: [Ведущему: четко прочитать окончание в слове "ВТОРОЕ".]
                    887:    На исторической картине Клавдия Лебедева перед алчным князем стоит
                    888: склонившийся ПЕРВЫЙ, а на рисунке, изображающем последовавшие события, -
                    889: склоненные ВТОРЫЕ. Слова "ПЕРВЫЙ" и "ВТОРОЕ" происходят от общего корня.
                    890: Какие слова мы заменили словами "ПЕРВЫЙ" и "ВТОРОЕ"?
                    891: 
                    892: Ответ:
                    893: Древлянин, дерево.
                    894: 
                    895: Комментарий:
                    896: На картине Лебедева запечатлен князь Игорь, собирающий полюдье с
                    897: древлян. На рисунке изображена казнь Игоря, которого разорвали, привязав
                    898: к склоненным деревьям. Древлянам, по объяснению летописца, их имя дано
                    899: потому, что они жили в лесах.
                    900: 
                    901: Источник:
                    902:    1. http://ru.wikipedia.org/wiki/Игорь_Рюрикович
                    903:    2. http://kzeteron.ucoz.ru/photo?photo=21
                    904:    3. http://ru.wikipedia.org/wiki/Древляне
                    905: 
                    906: Автор:
                    907: Александр Кудрявцев (Николаев)
                    908: 
                    909: Вопрос 8:
                    910: Замкнутые дети - не единственная проблема детских психологов. К примеру,
                    911: когда поведение шестилетней Салиши Джонсон заставило администрацию
                    912: детского сада беспомощно развести руками, были использованы ОНИ.
                    913: Текстильные ОНИ имеют прочность на разрыв не менее ста сорока
                    914: килограммов. Назовите ИХ.
                    915: 
                    916: Ответ:
                    917: Наручники.
                    918: 
                    919: Комментарий:
                    920: Салиша Джонсон безобразничала в детском саду, и администрация вызвала
                    921: полицию. Стражам порядка пришлось замкнуть наручники на запястьях
                    922: маленькой хулиганки.
                    923: 
                    924: Источник:
                    925:    1. http://lenta.ru/news/2012/04/17/kindergartner/
                    926:    2. http://www.euro-security.info/euro-security.info/ru/textile-disposable-handcuffs.html
                    927: 
                    928: Автор:
                    929: Александр Кудрявцев (Николаев)
                    930: 
                    931: Вопрос 9:
                    932: [Ведущему: не сообщать игрокам, что слово "мостов" написано в кавычках.]
                    933:    Необычайное совпадение заставило ЕГО предложить вместо "мостов"
                    934: подвижную модель. Назовите ЕГО.
                    935: 
                    936: Ответ:
                    937: [Альфред ЛОтар] ВЕгенер.
                    938: 
                    939: Комментарий:
                    940: Ранее считалось, что общность флоры и фауны удаленных один от другого
                    941: материков объясняется существованием в далеком прошлом "сухопутных
                    942: мостов", связывавших материки. Вегенер обратил внимание на необычайное
                    943: совпадение их береговых линий и предложил теорию дрейфа материков.
                    944: 
                    945: Источник:
                    946: http://ru.wikipedia.org/wiki/Теория_дрейфа_материков
                    947: 
                    948: Автор:
                    949: Александр Кудрявцев (Николаев)
                    950: 
                    951: Вопрос 10:
                    952: Внимание, в тексте вопроса мы немного изменили одно из слов.
                    953:    На обложке альбома метал-группы "Австралийская машина убийств"
                    954: известный персонаж восседает на горе из черепов. Назовите этого
                    955: персонажа.
                    956: 
                    957: Ответ:
                    958: Терминатор.
                    959: 
                    960: Зачет:
                    961: T-800.
                    962: 
                    963: Комментарий:
                    964: Группа называется "Австрийская машина убийств", на обложках ее альбомов
                    965: изображен терминатор, в чертах которого легко угадывается Арнольд
                    966: Шварценеггер.
                    967: 
                    968: Источник:
                    969:    1. http://www.metalinjection.net/upcoming-releases/tim-lambesis-austrian-death-machine-triple-brutal-gets-a-release-date-new-song-released
                    970:    2. http://ru.wikipedia.org/wiki/T-800_(робот)
                    971: 
                    972: Автор:
                    973: Александр Кудрявцев (Николаев)
                    974: 
                    975: Вопрос 11:
                    976: (pic: 20140514.jpg)
                    977:    На снимке - одно из НИХ, так что даже самым большим скептикам
                    978: придется признать, что ОНИ - не вымысел. Назовите ИХ двумя словами.
                    979: 
                    980: Ответ:
                    981: Кладбища слонов.
                    982: 
                    983: Зачет:
                    984: Слоновьи кладбища.
                    985: 
                    986: Комментарий:
                    987: Снимок сделан в Таиланде. Существует миф, что слоны хоронят своих
                    988: умерших сородичей в особых местах, называемых слоновьими кладбищами.
                    989: 
                    990: Источник:
                    991:    1. http://www.avivas.ru/photo_view/77884.html
                    992:    2. http://www.pravda.ru/science/useful/11-01-2013/1140703-elephants_necro-0/
                    993: 
                    994: Автор:
                    995: Александр Кудрявцев (Николаев)
                    996: 
                    997: Вопрос 12:
                    998: Недавно служители одного зоопарка услышали доносившиеся из львиного
                    999: вольера крики о спасении. Однако кричавший человек показался львам
                   1000: неубедительным и потерпел предсказуемую неудачу, так и не сумев... Что
                   1001: именно сделать?
                   1002: 
                   1003: Ответ:
                   1004: Обратить львов в христианство.
                   1005: 
                   1006: Зачет:
                   1007: Крестить львов; сделать львов христианами.
                   1008: 
                   1009: Комментарий:
                   1010: Некий Чен Чун Хо из Тайбея, возжелав, чтобы львы стали добрыми
                   1011: христианами, прыгнул в львиный вольер и закричал: "Иисус спасет вас!".
                   1012: 
                   1013: Источник:
                   1014: http://www.newsru.com/world/04nov2004/lion.html
                   1015: 
                   1016: Автор:
                   1017: Александр Кудрявцев (Николаев)
                   1018: 
                   1019: Вопрос 13:
                   1020: Гюнтер Лав как-то решил продать инструмент, дважды приносивший ему
                   1021: первое место на международных конкурсах. В объявлении Гюнтер указал, что
                   1022: тот в отличном состоянии, не имеет заметных повреждений и стоит всего
                   1023: полторы тысячи евро. Назовите заглавного героя, имеющего
                   1024: основополагающее сходство с этим инструментом.
                   1025: 
                   1026: Ответ:
                   1027: Человек-невидимка.
                   1028: 
                   1029: Комментарий:
                   1030: Гюнтер Лав продавал невидимую гитару. Конкурсы по игре на невидимой
                   1031: гитаре проводятся уже около тридцати лет.
                   1032: 
                   1033: Источник:
                   1034: http://strannovosti.ru/strange/humans/2199
                   1035: 
                   1036: Автор:
                   1037: Александр Кудрявцев (Николаев)
                   1038: 
                   1039: Вопрос 14:
                   1040: Известный итальянец в 1877 году сделал рисунок, весьма далекий от
                   1041: оригинала, а интерпретируя нарисованное, использовал термин, вызвавший
                   1042: всеобщее оживление. Каким бранным словом назвал за это упомянутого
                   1043: итальянца автор вопроса?
                   1044: 
                   1045: Ответ:
                   1046: Каналья.
                   1047: 
                   1048: Комментарий:
                   1049: Известный итальянец - астроном Джованни Скиапарелли, "первооткрыватель"
                   1050: марсианских каналов. Скиапарелли линии на своем рисунке назвал
                   1051: итальянским словом "canali" [канАли], которое обозначает протоки как
                   1052: искусственного, так и естественного происхождения. При переводе на
                   1053: английский было использовано слово "canals" [кЭнэлс], обозначающее
                   1054: исключительно каналы искусственного происхождения.
                   1055: 
                   1056: Источник:
                   1057: "Вселенная" (The Universe), s01e02. "Марс: Красная планета".
                   1058: 
                   1059: Автор:
                   1060: Александр Кудрявцев (Николаев)
                   1061: 
                   1062: Вопрос 15:
                   1063: Внимание, в вопросе имеется некоторая натяжка.
                   1064:    Прием, применявшийся при постановке поединков в фильмах "Матрица" и
                   1065: "Убить Билла", позволил выделить эти фильмы в поджанр, который получил
                   1066: название "ТАКОЕ кунг-фу". Назовите праздничный атрибут, снабжаемый ТАКОЙ
                   1067: уздечкой.
                   1068: 
                   1069: Ответ:
                   1070: Бутылка шампанского.
                   1071: 
                   1072: Зачет:
                   1073: Шампанское; пробка [от] шампанского.
                   1074: 
                   1075: Комментарий:
                   1076: Этот поджанр - wire-fu [вАйер фу], название которого можно перевести и
                   1077: как "кунг-фу на тросах", и как "проволочное кунг-фу". С помощью тросов в
                   1078: названных фильмах снимались головокружительные схватки. Проволочная
                   1079: уздечка-мюзле удерживает пробку шампанского.
                   1080: 
                   1081: Источник:
                   1082:    1. http://en.wikipedia.org/wiki/Wire_fu
                   1083:    2. http://ru.wikipedia.org/wiki/Мюзле
                   1084: 
                   1085: Автор:
                   1086: Александр Кудрявцев (Николаев)
                   1087: 
                   1088: Вопрос 16:
                   1089: На сайте forumklassika.ru [форум классика точка ру] в шутку
                   1090: утверждается, что появление опорного шпиля способствовало эмансипации
                   1091: женщин, ведь раньше пышные одеяния мешали женщинам заниматься ЭТИМ.
                   1092: Назовите ЭТО тремя словами.
                   1093: 
                   1094: Ответ:
                   1095: Игра на виолончели.
                   1096: 
                   1097: Комментарий:
                   1098: Раньше виолончель удерживали между ног, что не совсем удобно в пышных
                   1099: юбках.
                   1100: 
                   1101: Источник:
                   1102: http://www.forumklassika.ru/showthread.php?t=92900
                   1103: 
                   1104: Автор:
                   1105: Александр Кудрявцев (Николаев)
                   1106: 
                   1107: Вопрос 17:
                   1108: [Ведущему: о кавычках не сообщать.]
                   1109:    ЕЕ начинают "доить" в семь лет, а по достижении двадцати пяти лет,
                   1110: когда качество "молока" ухудшается, поступают самым беспощадным образом.
                   1111: Назовите ЕЕ одним словом.
                   1112: 
                   1113: Ответ:
                   1114: Гевея.
                   1115: 
                   1116: Комментарий:
                   1117: Когда возраст дерева-каучуконоса достигает 25 лет и качество млечного
                   1118: сока становится неудовлетворительным, гевею обычно срубают.
                   1119: 
                   1120: Источник:
                   1121: Программа "В поисках приключений", посвященная Индии.
                   1122: 
                   1123: Автор:
                   1124: Александр Кудрявцев (Николаев)
                   1125: 
                   1126: Вопрос 18:
                   1127: Этот эффектный и рискованный прием, часто демонстрируемый на
                   1128: специализированных шоу, у нас называют "свеча", а в англоязычных странах
                   1129: - "wheelie" [вИли]. Что укорачивают, чтобы во время выполнения этого
                   1130: приема избежать контакта с покрытием?
                   1131: 
                   1132: Ответ:
                   1133: Выхлопную трубу.
                   1134: 
                   1135: Комментарий:
                   1136: Свеча - это постановка мотоцикла на заднее колесо в положение "12
                   1137: часов". Чтобы выхлопные трубы не царапали дорожное покрытие, их
                   1138: необходимо подпилить. Термин "wheelie" - уменьшительное от "wheel"
                   1139: [вил], что значит "колесо".
                   1140: 
                   1141: Источник:
                   1142: http://www.r6-club.ru/forums/index.php?showtopic=547&st=20
                   1143: 
                   1144: Автор:
                   1145: Александр Кудрявцев (Николаев)
                   1146: 
                   1147: Вопрос 19:
                   1148: [Ведущему: о кавычках не сообщать.]
                   1149:    Прекраснейший бриллиант КондЕ обнаружила голодная горничная,
                   1150: прибиравшаяся в номере, в котором жили его похитители. Украденный камень
                   1151: отыскался в ИКСЕ. В "ИКСЕ" можно отыскать Александра Александровича.
                   1152: Назовите ИКС.
                   1153: 
                   1154: Ответ:
                   1155: Яблоко.
                   1156: 
                   1157: Комментарий:
                   1158: Преступники засунули бриллиант в яблоко, которым решила полакомиться
                   1159: горничная. В слове "яблоко" есть буквосочетание "блок". Слово
                   1160: "прекраснейший" - подсказка.
                   1161: 
                   1162: Источник:
                   1163:    1. http://www.masteruvelir.ru/spravochnik-yuvelira/samyie-znamenityie-kamni/conde.html
                   1164:    2. Тщательный анализ слова "яблоко".
                   1165: 
                   1166: Автор:
                   1167: Александр Кудрявцев (Николаев)
                   1168: 
                   1169: Вопрос 20:
                   1170: Безработный из Бронкса Майкл Перес разрисовал себя черным маркером за
                   1171: несколько минут до выхода на сцену, где должна была состояться
                   1172: торжественная церемония. Что купил Перес двумя неделями ранее?
                   1173: 
                   1174: Ответ:
                   1175: Лотерейный билет.
                   1176: 
                   1177: Комментарий:
                   1178: Майкл Перес, прежде чем выйти на сцену, где ему должны были вручить чек
                   1179: на 14 миллионов долларов, нарисовал маркером бородку и усы. Победитель,
                   1180: живший в неблагополучном районе, не хотел, чтобы его узнали проживавшие
                   1181: по соседству бандиты.
                   1182: 
                   1183: Источник:
                   1184: http://www.nypost.com/2008/04/16/hidden-wealth/
                   1185: 
                   1186: Автор:
                   1187: Александр Кудрявцев (Николаев)
                   1188: 
                   1189: Вопрос 21:
                   1190: Группу женщин из фильма "Дюна" некий остряк назвал словосочетанием,
                   1191: получившим известность два десятилетия назад. Напишите это
                   1192: словосочетание.
                   1193: 
                   1194: Ответ:
                   1195: (The) Spice Girls [зэ спайс гёрлз].
                   1196: 
                   1197: Комментарий:
                   1198: В мире, созданном воображением фантаста Фрэнка Херберта, вещество под
                   1199: названием "спайс" имеет огромную важность. Женская группа "Спайс Гёрлз"
                   1200: была образована в 1994 году.
                   1201: 
                   1202: Источник:
                   1203:    1. https://vk.com/frank_herberts_dune?w=wall-50406988_172
                   1204:    2. http://ru.wikipedia.org/wiki/Пряность_(Дюна)
                   1205:    3. http://ru.wikipedia.org/wiki/Spice_Girls
                   1206: 
                   1207: Автор:
                   1208: Александр Кудрявцев (Николаев)
                   1209: 
                   1210: Вопрос 22:
                   1211: Исследования одной южноафриканской пещеры обернулись для Андре КЕйсера
                   1212: неожиданной находкой - ученый откопал человекообразное существо,
                   1213: предположительно женского пола. Какое греческое имя получило это
                   1214: существо?
                   1215: 
                   1216: Ответ:
                   1217: Эвридика.
                   1218: 
                   1219: Комментарий:
                   1220: Орфей отправился в подземное царство мертвых, чтобы вывести оттуда свою
                   1221: Эвридику. Однако не совладал с собой и обернулся, нарушив поставленное
                   1222: условие, из-за чего навсегда ее потерял.
                   1223: 
                   1224: Источник:
                   1225:    1. http://www.istorion.ru/prehist/p_robustus.html
                   1226:    2. http://ru.wikipedia.org/wiki/Эвридика
                   1227: 
                   1228: Автор:
                   1229: Александр Кудрявцев (Николаев)
                   1230: 
                   1231: Вопрос 23:
                   1232: Какая страна более двадцати лет находится под властью кровавого режима
                   1233: ИсАйяса АфевОрки?
                   1234: 
                   1235: Ответ:
                   1236: Эритрея.
                   1237: 
                   1238: Комментарий:
                   1239: ИсАйяс АфевОрки - первый и единственный президент Эритреи, которая
                   1240: провозгласила независимость в 1993 году. Название страны происходит от
                   1241: греческого "&#7952;&#961;&#965;&#952;&#961;&#972;&#962;" [эрИтрос] -
                   1242: "красный".
                   1243: 
                   1244: Источник:
                   1245:    1. http://ru.wikipedia.org/wiki/Афеворк,_Исайяс
                   1246:    2. http://www.ipress.ua/ljlive/erytreya_afrykanska_derzhava_shcho_bula_b_dorechnoyu_v_yevropi_foto_17810.html
                   1247: 
                   1248: Автор:
                   1249: Александр Кудрявцев (Николаев)
                   1250: 
                   1251: Вопрос 24:
                   1252: В басне Эзопа человек, купивший заморского раба, считает, что прежний
                   1253: хозяин не заботился о нем, поскольку в буквальном смысле ДЕЛАЛ ЭТО.
                   1254: Несмотря на проявленное усердие, труды нового хозяина пропадают впустую.
                   1255: Какие слова мы заменили на "ДЕЛАТЬ ЭТО"?
                   1256: 
                   1257: Ответ:
                   1258: Держать раба в черном теле.
                   1259: 
                   1260: Зачет:
                   1261: Держать его в черном теле; держать в черном теле.
                   1262: 
                   1263: Комментарий:
                   1264: В басне "Эфиоп" новый хозяин посчитал, что раба-африканца попросту
                   1265: следует вымыть.
                   1266: 
                   1267: Источник:
                   1268: http://www.deti-online.com/basni/basni-ezopa/yefiop/
                   1269: 
                   1270: Автор:
                   1271: Александр Кудрявцев (Николаев)
                   1272: 
                   1273: Вопрос 25:
                   1274: В 1978 году в Иордании неожиданно прервалась подходившая к завершению
                   1275: трансляция "Евровидения", и лишь через некоторое время иорданским
                   1276: телезрителям сообщили, что победил бельгийский певец Жан ВаллЕ. В какой
                   1277: стране прошло "Евровидение" в 1979 году?
                   1278: 
                   1279: Ответ:
                   1280: Израиль.
                   1281: 
                   1282: Комментарий:
                   1283: Арабские страны враждебно относятся к Израилю. Трансляцию прервали,
                   1284: когда стало ясно, что побеждает именно он. Как известно, местом
                   1285: проведения "Евровидения" становится страна, победившая в минувшем году.
                   1286: 
                   1287: Источник:
                   1288: http://www.israellycool.com/2014/05/09/israels-exit-from-eurovision-no-scandal/
                   1289: 
                   1290: Автор:
                   1291: Александр Кудрявцев (Николаев)
                   1292: 
                   1293: Вопрос 26:
                   1294: Герб заокеанской страны совсем не похож на герб наибольшей из ее
                   1295: территорий. Тем не менее, крупное млекопитающее, изображенное на гербе
                   1296: этой территории справа от щита, появилось там явно неслучайно. Назовите
                   1297: это млекопитающее словом скандинавского происхождения.
                   1298: 
                   1299: Ответ:
                   1300: Нарвал.
                   1301: 
                   1302: Комментарий:
                   1303: Канада делится на провинции и территории. И на гербе Канады, и на гербе
                   1304: территории НУнавут изображены единороги - мифический и вполне реальный.
                   1305: В гербе Канады за основу взят герб Великобритании со львом и единорогом.
                   1306: Нарвал и единорог - два названия одного и того же китообразного.
                   1307: 
                   1308: Источник:
                   1309:    1. http://ru.wikipedia.org/wiki/Герб_Канады
                   1310:    2. http://ru.wikipedia.org/wiki/Герб_Нунавута
                   1311:    3. http://www.etymonline.com/index.php?term=narwhal&allowed_in_frame=0
                   1312:    4. http://ru.wikipedia.org/wiki/Нарвал
                   1313:    5. http://ru.wikipedia.org/wiki/Герб_Великобритании
                   1314: 
                   1315: Автор:
                   1316: Александр Кудрявцев (Николаев)
                   1317: 
                   1318: Вопрос 27:
                   1319: Объединитель Норвегии Харальд Косматый СДЕЛАЛ ЭТО после победы при
                   1320: Хаврсфьорде. Назовите многодетного отца, который, согласно шутке,
                   1321: получил свое прозвище за то, что не любил ДЕЛАТЬ ЭТО.
                   1322: 
                   1323: Ответ:
                   1324: Всеволод [Юрьевич] Большое Гнездо.
                   1325: 
                   1326: Комментарий:
                   1327: ДЕЛАТЬ ЭТО - расчесываться. Харальд Косматый дал слово не расчесываться,
                   1328: пока не объединит Норвегию. Когда же это произошло, Харальд получил
                   1329: новое прозвище - Прекрасноволосый. Всеволод Большое Гнездо в
                   1330: действительности получил свое прозвище не за то, что не любил
                   1331: расчесываться, а оттого, что имел много детей. Про растрепанного,
                   1332: взлохмаченного человека говорят, что у него гнездо на голове.
                   1333: 
                   1334: Источник:
                   1335:    1. http://ru.wikipedia.org/wiki/Харальд_I_Прекрасноволосый
                   1336:    2. http://www.novosti-kosmonavtiki.ru/forum/messages/forum16/topic13192/message1025670/#message1025670
                   1337:    3. http://ru.wikipedia.org/wiki/Всеволод_Юрьевич_Большое_Гнездо
                   1338:    4. http://dic.academic.ru/dic.nsf/proverbs/17011/
                   1339: 
                   1340: Автор:
                   1341: Александр Кудрявцев (Николаев)
                   1342: 
                   1343: Вопрос 28:
                   1344: (pic: 20140515.jpg)
                   1345:    Среди женщин самым высокооплачиваемым исполнительным директором
                   1346: является МартИна РОтблатт, которая до 1994 года была ИМ. В экранизации
                   1347: 1955 года ЕМУ дарует свободу былой обидчик. Назовите этого обидчика.
                   1348: 
                   1349: Ответ:
                   1350: Нильс [ХОльгерсон].
                   1351: 
                   1352: Комментарий:
                   1353: ОН - Мартин. До 1994 года Мартина была Мартином, но сменила пол. В
                   1354: мультфильме "Заколдованный мальчик" Нильс в начале попадает в гуся
                   1355: Мартина камнем, а в конце выпускает из птичника.
                   1356: 
                   1357: Источник:
                   1358:    1. http://www.meetup.com/Terasem/members/37506202/
                   1359:    2. http://en.wikipedia.org/wiki/Martine_Rothblatt
                   1360:    3. http://www.svoboda.org/content/article/26583241.html
                   1361:    4. http://www.youtube.com/watch?v=CU5PsW3f9So
                   1362: 
                   1363: Автор:
                   1364: Александр Кудрявцев (Николаев)
                   1365: 
                   1366: Вопрос 29:
                   1367: Содержащийся в НЕЙ белок в шесть раз сильнее морфина, поэтому действие,
                   1368: которое описывается устойчивым словосочетанием, имеет еще и
                   1369: обезболивающий эффект. Тренер школьной команды из штата Орегон совершил
                   1370: это действие на одной из тренировок, чтобы доказать, что он по-собачьи
                   1371: предан своим подопечным. Назовите это действие двумя словами.
                   1372: 
                   1373: Ответ:
                   1374: Зализывание ран.
                   1375: 
                   1376: Комментарий:
                   1377: ОНА - слюна. Как оказалось, слюна обладает не только бактерицидными
                   1378: свойствами. Тренер сказал, что готов даже зализывать раны своим
                   1379: ученикам, после чего принялся вылизывать одному из них разбитое колено.
                   1380: 
                   1381: Источник:
                   1382:    1. http://en.wikipedia.org/wiki/Opiorphin
                   1383:    2. http://www.newsru.com/world/05aug2005/lick.html
                   1384: 
                   1385: Автор:
                   1386: Александр Кудрявцев (Николаев)
                   1387: 
                   1388: Вопрос 30:
                   1389: На лицевой стороне киргизской банкноты в сто сомов, имевшей хождение в
                   1390: конце 1990-х, изображен народный акын ТоктогУл СатылгАнов. Какой хан
                   1391: изображен на оборотной стороне этой банкноты?
                   1392: 
                   1393: Ответ:
                   1394: Хан-ТЕнгри.
                   1395: 
                   1396: Зачет:
                   1397: ТЕнгри; Небесный [хан]; [Хан] неба.
                   1398: 
                   1399: Комментарий:
                   1400: Второй по высоте пик Тянь-Шаня. Обычно портреты выдающихся деятелей
                   1401: размещают только на одной стороне банкноты, помещая на вторую
                   1402: изображения природных объектов, архитектурных шедевров и т.п.
                   1403: 
                   1404: Источник:
                   1405:    1. http://ru.wikipedia.org/wiki/Киргизский_сом
                   1406:    2. http://ru.wikipedia.org/wiki/Сатылганов,_Токтогул
                   1407:    3. http://ru.wikipedia.org/wiki/Киргизия
                   1408: 
                   1409: Автор:
                   1410: Александр Кудрявцев (Николаев)
                   1411: 
                   1412: Вопрос 31:
                   1413: Известный американец, предрекая повторный взлет популярности, сообщил
                   1414: читателям своего блога, что их любимая жевательная резинка в 2016 году
                   1415: снова войдет в моду. Коллега этого американца добавил, что спустя 25 лет
                   1416: они обязаны дать ответы и на другие вопросы, не дававшие покоя
                   1417: поклонникам. Назовите этого американца.
                   1418: 
                   1419: Ответ:
                   1420: [Дэвид] Линч.
                   1421: 
                   1422: Комментарий:
                   1423: Режиссер сообщил своим почитателям, что вскоре следует ожидать выхода на
                   1424: экран продолжения сериала "Твин Пикс", и в 2016 году он вновь будет на
                   1425: пике популярности. Ответ на самый знаменитый вопрос сериала, "Кто убил
                   1426: Лору Палмер?", зрители узнали, но остались и другие вопросы.
                   1427: 
                   1428: Источник:
                   1429: http://tjournal.ru/paper/twinpeaks-2016
                   1430: 
                   1431: Автор:
                   1432: Александр Кудрявцев (Николаев)
                   1433: 
                   1434: Вопрос 32:
                   1435: В замке ЛИндерхоф есть Грот Венеры, а в нем - искусственное озеро, для
                   1436: плавания по которому построена лодка в форме АЛЬФЫ. Дизайнеры интерьеров
                   1437: создали АЛЬФУ в форме АЛЬФЫ. Назовите АЛЬФУ.
                   1438: 
                   1439: Ответ:
                   1440: Раковина.
                   1441: 
                   1442: Комментарий:
                   1443: Венера появилась на свет из раковины. Лодка сделана в форме створки
                   1444: раковины двустворчатого моллюска. Дизайнеры сделали кухонную раковину в
                   1445: виде отпечатка спиральной раковины аммонита.
                   1446: 
                   1447: Источник:
                   1448:    1. http://ru.wikipedia.org/wiki/Линдерхоф
                   1449:    2. http://www.novate.ru/blogs/290608/9637/
                   1450: 
                   1451: Автор:
                   1452: Александр Кудрявцев (Николаев)
                   1453: 
                   1454: Вопрос 33:
                   1455: Глубинные гидротермальные источники являются своеобразными подводными
                   1456: оазисами, около которых, по словам Дмитрия Писаренко, ПРОИСХОДИТ ЭТО.
                   1457: Ответьте двумя словами, в которых все гласные одинаковы, что именно
                   1458: происходит.
                   1459: 
                   1460: Ответ:
                   1461: Кипит жизнь.
                   1462: 
                   1463: Комментарий:
                   1464: Температура воды в черных курильщиках, о которых, собственно, и идет
                   1465: речь, может превышать 400 градусов. Эти термальные источники привлекают
                   1466: к себе большое количество глубоководных существ.
                   1467: 
                   1468: Источник:
                   1469:    1. http://gazeta.aif.ru/_/online/aif/1183/15_02
                   1470:    2. http://ru.wikipedia.org/wiki/Гидротермальные_источники_срединно-океанических_хребтов
                   1471: 
                   1472: Автор:
                   1473: Александр Кудрявцев (Николаев)
                   1474: 
                   1475: Вопрос 34:
                   1476: Словом "ИКС" мы заменили другое слово.
                   1477:    Арт-акция Дилана Майера, завершившаяся убийством ИКСА, возмутила
                   1478: жителей Сиэтла. Оправдываясь, Майер сказал, что схватка шла с переменным
                   1479: успехом, и ИКС едва его не задушил. Жертвой ИКСА можно считать мужчину,
                   1480: убитого 20 марта 1989 года. Назовите этого мужчину двумя или тремя
                   1481: словами, начинающимися на одну и ту же букву.
                   1482: 
                   1483: Ответ:
                   1484: Комиссар Коррадо Каттани.
                   1485: 
                   1486: Зачет:
                   1487: Комиссар Каттани; Коррадо Каттани.
                   1488: 
                   1489: Комментарий:
                   1490: ИКС - спрут. Дайвер Дилан Майер спустился под воду и убил гигантского
                   1491: тихоокеанского спрута. "Спрут" - сериал об итальянской мафии.
                   1492: 
                   1493: Источник:
                   1494:    1. http://www.npr.org/blogs/thetwo-way/2012/11/04/164287403/man-who-killed-octopus-for-art-project-angers-divers
                   1495:    2. http://ru.wikipedia.org/wiki/Комиссар_Каттани
                   1496: 
                   1497: Автор:
                   1498: Александр Кудрявцев (Николаев)
                   1499: 
                   1500: Вопрос 35:
                   1501: На рождественской открытке трижды написано слово "Праздники", и в каждом
                   1502: из этих слов выделены одни и те же две буквы. Напишите эти буквы.
                   1503: 
                   1504: Ответ:
                   1505: H, o [эйч оу].
                   1506: 
                   1507: Комментарий:
                   1508: На рождественской открытке трижды написано слово "Holidays" [хОлидейз].
                   1509: Известно, что Санта смеется "Хо-хо-хо!".
                   1510: 
                   1511: Источник:
                   1512: https://visualtextproject.wordpress.com/2011/12/24/visual-text-greeting/
                   1513: 
                   1514: Автор:
                   1515: Александр Кудрявцев (Николаев)
                   1516: 
                   1517: Вопрос 36:
                   1518: Александр Твардовский, принимавший участие в освобождении оккупированных
                   1519: фашистами советских территорий, часто рассказывал историю о трехлетнем
                   1520: мальчике, которого угостили кусочком хлеба. Какие два слова произнес
                   1521: мальчик, взяв хлеб?
                   1522: 
                   1523: Ответ:
                   1524: Danke Sch&ouml;n.
                   1525: 
                   1526: Зачет:
                   1527: Данке шон; Данке шён.
                   1528: 
                   1529: Комментарий:
                   1530: С рождения русский мальчик привык к немецкой речи.
                   1531: 
                   1532: Источник:
                   1533: Документальный фильм "Три жизни поэта".
                   1534: 
                   1535: Автор:
                   1536: Александр Кудрявцев (Николаев)
                   1537: 
                   1538: Тур:
                   1539: Этап 3
                   1540: 
                   1541: Дата:
                   1542: 16-Nov-2014
                   1543: 
                   1544: Редактор:
                   1545: Алексей и Мария Трефиловы (Калуга)
                   1546: 
                   1547: Инфо:
                   1548: Редакторы благодарят за тестирование и ценные замечания: Баура
                   1549: Бектемирова, Евгения Быстрова, Галину Воловник, Владимира Городецкого,
                   1550: Людмилу Губаеву, Глеба Дрепина, Михаила Иванова, Дмитрия Когана, Николая
                   1551: Константинова, Екатерину Лагуту, Михаила Малкина, Инну Семенову, Евгения
                   1552: Миротина, Николая Рябых, Павла Столярова, Ивана Суманеева, Антона
                   1553: Тахтарова, Александра Толесникова, Игоря Тюнькина.
                   1554: 
                   1555: Вопрос 1:
                   1556: В конце XIX века АнтонИн Дворжак написал в Нью-Йорке симфонию, известную
                   1557: также как "Симфония Нового света". Через 76 лет известный человек взял
                   1558: запись этой симфонии с собой. Назовите этого человека или любого из его
                   1559: спутников.
                   1560: 
                   1561: Ответ:
                   1562: [Нил] Армстронг.
                   1563: 
                   1564: Зачет:
                   1565: [Майкл] Коллинз, [Эдвин/Базз] Олдрин.
                   1566: 
                   1567: Комментарий:
                   1568: Символично, что эту симфонию Армстронг взял с собой в полет на Луну. Из
                   1569: одного Нового света - к другому.
                   1570: 
                   1571: Источник:
                   1572: http://ru.wikipedia.org/wiki/Симфония_%E2%84%96_9_(Дворжак)
                   1573: 
                   1574: Автор:
                   1575: Игорь Тюнькин (Москва)
                   1576: 
                   1577: Вопрос 2:
                   1578: Индейский герой Тийо получил от НЕЕ в подарок плетеную корзину. Другая
                   1579: ОНА, появившаяся в тридцать втором выпуске в феврале 1977 года, была
                   1580: дочерью биологов. Назовите ЕЕ.
                   1581: 
                   1582: Ответ:
                   1583: Женщина-паук.
                   1584: 
                   1585: Зачет:
                   1586: Spider-Woman. Незачет: Паучиха.
                   1587: 
                   1588: Комментарий:
                   1589: То, что корзина плетеная - намекает на паутину. Во втором случае - речь
                   1590: про героиню комиксов.
                   1591: 
                   1592: Источник:
                   1593:    1. Spider Woman Stories: Legends of the Hopi Indians, p. 32.
                   1594:    2. http://en.wikipedia.org/wiki/Spider-Woman
                   1595: 
                   1596: Автор:
                   1597: Баур Бектемиров (Алматы)
                   1598: 
                   1599: Вопрос 3:
                   1600: В абсурдистском романе Бориса Виана повар применил к НЕМУ
                   1601: стоматологические щипцы. Рассказывают, что муж одной актрисы давал ЕГО
                   1602: своей жене перед съемками. Назовите ЕГО.
                   1603: 
                   1604: Ответ:
                   1605: Чеснок.
                   1606: 
                   1607: Зачет:
                   1608: Зубчик чеснока.
                   1609: 
                   1610: Комментарий:
                   1611: Ревнивый муж давал жене есть чеснок, если по сценарию предполагалась
                   1612: сцена с поцелуем. В романе Виана повар выдирал зубчик чеснока щипцами.
                   1613: 
                   1614: Источник:
                   1615:    1. http://lib.ru/WIAN/pena0.txt
                   1616:    2. Легенды мирового кино. Грегори Пек.
                   1617: 
                   1618: Автор:
                   1619: Мария Трефилова (Калуга), Игорь Тюнькин (Москва)
                   1620: 
                   1621: Вопрос 4:
                   1622: Ирина Можаева рассказывает, что нередко ЭТО было частью приданого, так
                   1623: как служило напоминанием невесте, уходящей в новый дом, о каждом из ее
                   1624: родственников. Назовите ЭТО двумя словами.
                   1625: 
                   1626: Ответ:
                   1627: Лоскутное одеяло.
                   1628: 
                   1629: Комментарий:
                   1630: Это одеяло часто шили из лоскутков тканей, использовавшихся для пошива
                   1631: одежды других членов семьи.
                   1632: 
                   1633: Источник:
                   1634: http://www.artrussian.com/vopros_72.html
                   1635: 
                   1636: Автор:
                   1637: Алексей Трефилов (Калуга)
                   1638: 
                   1639: Вопрос 5:
                   1640: [Ведущему: не сообщать командам, что слово "Родина" взято в кавычки.]
                   1641:    В советское время в бухте закрытого военного городка Балаклава на
                   1642: одном из кораблей иногда натягивали полотно. И тогда местные жители о
                   1643: "Родине" уже не вспоминали. Сам этот корабль они называли плавучим...
                   1644: Чем?
                   1645: 
                   1646: Ответ:
                   1647: Кинотеатром.
                   1648: 
                   1649: Зачет:
                   1650: Кинотеатр.
                   1651: 
                   1652: Комментарий:
                   1653: Морякам показывали кино, и гражданским тоже было хорошо видно, поэтому в
                   1654: кинотеатр "Родина" в эти вечера они не шли.
                   1655: 
                   1656: Источник:
                   1657: "Остров Крым". Серия про Балаклаву.
                   1658: 
                   1659: Автор:
                   1660: Алексей Трефилов (Калуга)
                   1661: 
                   1662: Вопрос 6:
                   1663: (pic: 20140516.jpg)
                   1664:    Эта модель обуви названа в честь НЕГО. В честь НЕГО назван и мост в
                   1665: окрестностях Рима. Назовите ЕГО.
                   1666: 
                   1667: Ответ:
                   1668: [Абебе] БикИла.
                   1669: 
                   1670: Комментарий:
                   1671: Эта минималистичная модель спортивной обуви позволяет стопе чувствовать
                   1672: себя естественно. Как известно, Абебе БикИла в 1960 году выиграл
                   1673: "золото" в марафоне Римской олимпиады босиком.
                   1674: 
                   1675: Источник:
                   1676: http://en.wikipedia.org/wiki/Abebe_Bikila
                   1677: 
                   1678: Автор:
                   1679: Алексей Трефилов (Калуга)
                   1680: 
                   1681: Вопрос 7:
                   1682: В романе Роберта Ирвина персонаж-писатель расстался с любимой. Так
                   1683: сказать, любовная лодка разбилась о быт. Спустя несколько лет он захотел
                   1684: ее разыскать. И для этого решил издать автобиографическую книгу.
                   1685: Назовите тремя словами то, с чем он сравнил эту книгу.
                   1686: 
                   1687: Ответ:
                   1688: Письмо в бутылке.
                   1689: 
                   1690: Зачет:
                   1691: Послание в бутылке.
                   1692: 
                   1693: Комментарий:
                   1694: Герой не знал, как разыскать бывшую возлюбленную, и рассчитывал, что она
                   1695: прочтет его книгу и откликнется. Хотя и понимал, что шансов немного.
                   1696: Метафора Маяковского про "любовную лодку" - намек на морскую тематику.
                   1697: 
                   1698: Источник:
                   1699: http://www.e-libra.ru/read/222591-utonchennyj-mertvec-exquisite-corpse.html
                   1700: 
                   1701: Автор:
                   1702: Мария Трефилова, Алексей Трефилов (Калуга)
                   1703: 
                   1704: Вопрос 8:
                   1705: На одной из своих картин Рене Магритт выразил отношение к женщине как к
                   1706: немому объекту плотских утех и изобразил, так сказать, ЕЕ наоборот.
                   1707: Назовите ЕЕ.
                   1708: 
                   1709: Ответ:
                   1710: Русалка.
                   1711: 
                   1712: Комментарий:
                   1713: (pic: 20140517.jpg)
                   1714:    Изобразил немую верхнюю половину рыбы и нижнюю обнаженную половину
                   1715: женщины.
                   1716: 
                   1717: Источник:
                   1718: Гении и злодеи. Рене Магритт.
                   1719: 
                   1720: Автор:
                   1721: Игорь Тюнькин (Москва)
                   1722: 
                   1723: Вопрос 9:
                   1724: Однажды поэт Евгений Винокуров сделал заказ в ресторане. Но потом
                   1725: окликнул уходящего официанта и заказал один кофе. А что он заказал
                   1726: сначала?
                   1727: 
                   1728: Ответ:
                   1729: Одно кофе.
                   1730: 
                   1731: Комментарий:
                   1732: Винокуров решил, что поэту не следует быть небрежным в языке, и
                   1733: исправился.
                   1734: 
                   1735: Источник:
                   1736: http://www.poezia.ru/salon.php?sid=21773
                   1737: 
                   1738: Автор:
                   1739: Алексей Трефилов (Калуга)
                   1740: 
                   1741: Вопрос 10:
                   1742: Леонардо да Винчи считал, что у НЕЕ нет органов пищеварения. Раньше в
                   1743: Англии ЕЮ называли кочергу. Назовите ЕЕ.
                   1744: 
                   1745: Ответ:
                   1746: Саламандра.
                   1747: 
                   1748: Комментарий:
                   1749: В старину считали, что саламандра питается от огня через кожу. Кочерга,
                   1750: как и саламандра, тоже часто оказывалась в очаге.
                   1751: 
                   1752: Источник:
                   1753:    1. http://en.wikipedia.org/wiki/Salamanders_in_folklore_and_legend
                   1754:    2. http://en.wiktionary.org/wiki/salamander
                   1755: 
                   1756: Автор:
                   1757: Баур Бектемиров (Алматы)
                   1758: 
                   1759: Вопрос 11:
                   1760: Внимание, дуплет! Два вопроса по 30 секунд обсуждения на каждый.
                   1761: Напишите сначала один ответ, и рядом - второй.
                   1762:    1. В одном хокку ОНА сыпала жемчуг в изумруд. Назовите ЕЕ.
                   1763:    2. На иллюстрации учебника естественной истории 1864 года изображены
                   1764: два орангутана. Что держит в руке меньший из них?
                   1765: 
                   1766: Ответ:
                   1767:    1. Яблоня.
                   1768:    2. Яблоко.
                   1769: 
                   1770: Комментарий:
                   1771: В учебнике - орангутаны в роли Адама и Евы. Преамбула к вопросу
                   1772: намекала, что в ваших ответных карточках "яблоко" окажется недалеко от
                   1773: "яблони". В стихотворении так описывается опадающий цвет - лепестки
                   1774: яблони белые и округлые. Но если вы подумали, что речь про маленькие
                   1775: яблочки, то тоже хорошо. :-)
                   1776: 
                   1777: Источник:
                   1778:    1. http://www.ordenknights.ru/l_ballad/ballad20.php
                   1779:    2. Boria Sax. Imaginary animals, p. 48.
                   1780: 
                   1781: Автор:
                   1782: Алексей Трефилов (Калуга), Баур Бектемиров (Алматы)
                   1783: 
                   1784: Вопрос 12:
                   1785: Раньше в продукции комбината, выпускающего киноматериалы, попадалось
                   1786: немало брака, поэтому операторы, заряжая пленку, по словам Эдуарда
                   1787: Тополя, ДЕЛАЛИ ЭТО. Ответьте четырьмя словами: что делали?
                   1788: 
                   1789: Ответ:
                   1790: Играли в русскую рулетку.
                   1791: 
                   1792: Комментарий:
                   1793: Операторы рисковали, ведь значительная часть работы могла оказаться
                   1794: напрасной. "Заряжая пленку" - подсказка. Кроме того, вращающиеся бобины
                   1795: с пленкой могут напомнить барабан револьвера.
                   1796: 
                   1797: Источник:
                   1798: "Story", 2014, N 10. - С. 72.
                   1799: 
                   1800: Автор:
                   1801: Мария Трефилова, Алексей Трефилов (Калуга)
                   1802: 
                   1803: Вопрос 13:
                   1804: По словам Анны Север, поэтесса Серебряного века ИраИда ГУставовна
                   1805: ГЕйнике стала Ириной Владимировной ОдОевцевой, сменив имя практически
                   1806: одновременно с НИМ. Назовите ЕГО.
                   1807: 
                   1808: Ответ:
                   1809: Санкт-Петербург.
                   1810: 
                   1811: Зачет:
                   1812: Петербург, Петроград, Питер. Незачет: Ленинград.
                   1813: 
                   1814: Комментарий:
                   1815: В связи с началом Первой мировой войны и неприятием всего немецкого
                   1816: имена меняли люди и города: Санкт-Петербург стал Петроградом.
                   1817: 
                   1818: Источник:
                   1819: "Story", 2014, N 8. - С. 58.
                   1820: 
                   1821: Автор:
                   1822: Мария Трефилова (Калуга)
                   1823: 
                   1824: Вопрос 14:
                   1825: Расположенная на побережье Балтийского моря КУршская коса долгое время
                   1826: страдала от наступающих песчаных дюн. Из-за нехватки плодородных земель
                   1827: местные жители были вынуждены питаться в основном рыбой. И, чтобы
                   1828: разнообразить рацион, они околачивали груши, расставляя сети на берегу.
                   1829: Восстановите два слова, которые мы заменили в тексте вопроса.
                   1830: 
                   1831: Ответ:
                   1832: Ловили ворон.
                   1833: 
                   1834: Зачет:
                   1835: Ловили галок. Незачет: Считали ворон.
                   1836: 
                   1837: Комментарий:
                   1838: Леса, сдерживающие песчаные дюны, были сильно вырублены. Было сложно
                   1839: найти другую дичь. Да и выращивать сельхозкультуры рядом с домом также
                   1840: было проблематично. Фразеологизм "ловить ворон" употребляется наряду с
                   1841: фразеологизмом "считать ворон".
                   1842: 
                   1843: Источник:
                   1844: Д/ф "Куршская коса". Эфир RTG.
                   1845: 
                   1846: Автор:
                   1847: Алексей Трефилов (Калуга)
                   1848: 
                   1849: Вопрос 15:
                   1850: (pic: 20140518.jpg)
                   1851:    АкутагАва РюнОскэ писал про своего остроумного персонажа, что тот
                   1852: применял два ИКСА: иронию и юмор. Назовите ИКС.
                   1853: 
                   1854: Ответ:
                   1855: [Самурайский] меч.
                   1856: 
                   1857: Комментарий:
                   1858: Раздаточный материал намекает на пару самурайских мечей, лежащих на
                   1859: традиционной подставке. Слово "остроумный" и то, что Акутагава - японец,
                   1860: тоже может помочь в раскрутке. :-)
                   1861: 
                   1862: Источник:
                   1863: http://www.litmir.net/br/?b=221009&p=70
                   1864: 
                   1865: Автор:
                   1866: Баур Бектемиров (Алматы)
                   1867: 
                   1868: Вопрос 16:
                   1869: В историческом романе описывается, как этого человека, помещенного в
                   1870: темницу за вольнодумство, в наказание выводили на прогулку только ночью.
                   1871: Александр Казанцев замечает, что этот человек взял себе имя словно в
                   1872: честь английского коллеги-предшественника. Назовите этого человека.
                   1873: 
                   1874: Ответ:
                   1875: [Томмазо] Кампанелла.
                   1876: 
                   1877: Комментарий:
                   1878: Автора утопии "Город Солнца" лишали солнечного света. Томмазо - это имя,
                   1879: под которым Кампанелла был пострижен в монахи. Томас Мор - другой
                   1880: известный утопист.
                   1881: 
                   1882: Источник:
                   1883: http://lib.ru/RUFANT/KAZANCEW/sunkolokol.txt
                   1884: 
                   1885: Автор:
                   1886: Алексей Трефилов (Калуга)
                   1887: 
                   1888: Вопрос 17:
                   1889: Джотто был новатором и, изображая ИКС святого, делал нимб овальным.
                   1890: Какое слово мы заменили на ИКС?
                   1891: 
                   1892: Ответ:
                   1893: Профиль.
                   1894: 
                   1895: Комментарий:
                   1896: Джотто таким образом старался передать перспективу. Он решил, что при
                   1897: повороте головы нимб тоже поворачивается и визуально выглядит иначе.
                   1898: 
                   1899: Источник:
                   1900: Цикл "Энциклопедия". Джотто.
                   1901: 
                   1902: Автор:
                   1903: Алексей Трефилов (Калуга)
                   1904: 
                   1905: Вопрос 18:
                   1906: В российском документальном фильме говорится, что дома местных рыбаков
                   1907: имели некоторые архитектурные особенности. Например, входные двери
                   1908: состояли из двух створок: нижней и верхней, чтобы после очень ветреной
                   1909: ночи можно было открыть хотя бы верхнюю. Этот фильм рассказывает о НЕЙ.
                   1910: Назовите ЕЕ двумя словами, начинающимися на одну и ту же букву.
                   1911: 
                   1912: Ответ:
                   1913: Куршская коса.
                   1914: 
                   1915: Комментарий:
                   1916: Как вы уже знаете, местные жители на Куршской косе страдали от
                   1917: наступающих дюн. Пески засыпАли деревни. И жители шли на хитрость,
                   1918: позволяющую легко выбраться из засыпанного дома.
                   1919: 
                   1920: Источник:
                   1921: Д/ф "Куршская коса". Эфир RTG.
                   1922: 
                   1923: Автор:
                   1924: Алексей Трефилов (Калуга)
                   1925: 
                   1926: Вопрос 19:
                   1927: Записавшись на курсы в Манчестере, жительница Белфаста Диана ЛИндблатт
                   1928: доехала поездом до Дублина, оттуда паромом до ХОлихеда, а затем еще
                   1929: четыре часа добиралась до Манчестера. У всех слушателей этих курсов была
                   1930: ОНА. Назовите ЕЕ одним словом.
                   1931: 
                   1932: Ответ:
                   1933: Аэрофобия.
                   1934: 
                   1935: Комментарий:
                   1936: Диана записалась на специальные курсы, чтобы избавиться от аэрофобии, и,
                   1937: так как она боялась полетов, ей пришлось проделать столь долгий путь.
                   1938: 
                   1939: Источник:
                   1940: BBC. "Страх".
                   1941: 
                   1942: Автор:
                   1943: Игорь Тюнькин (Москва)
                   1944: 
                   1945: Вопрос 20:
                   1946: Бывший разведчик Михаил Любимов иронично замечает, что распознать
                   1947: предателя непросто. Ведь когда видишь этого человека, то нет никакого
                   1948: навевающего тревогу и подозрения ЕГО. Назовите ЕГО словом английского
                   1949: происхождения.
                   1950: 
                   1951: Ответ:
                   1952: Саундтрек.
                   1953: 
                   1954: Зачет:
                   1955: Soundtrack.
                   1956: 
                   1957: Комментарий:
                   1958: Любимов иронизирует, что при просмотре шпионских фильмов разбираться в
                   1959: людях гораздо проще.
                   1960: 
                   1961: Источник:
                   1962: "Story", 2014, N 5. - С. 33.
                   1963: 
                   1964: Автор:
                   1965: Мария Трефилова (Калуга)
                   1966: 
                   1967: Вопрос 21:
                   1968: [Ведущему: очень важно читать стихотворение очень медленно, под запись,
                   1969: делая значительные паузы в конце строк!]
                   1970:    Послушайте танка поэтессы ЁсАно АкИко:
                   1971:    Белеют плечи мои -
                   1972:    В деревянной бадье купаюсь;
                   1973:    Некто подумает, увидав:
                   1974:    Белых кувшинок бутоны
                   1975:    Распустились.
                   1976:    Комментируя это стихотворение, Елена Дьяконова отмечает, что ЁсАно
                   1977: свойственен ОН. Назовите ЕГО.
                   1978: 
                   1979: Ответ:
                   1980: Нарциссизм.
                   1981: 
                   1982: Комментарий:
                   1983: Ёсано в стихотворении открыто любуется собой, что очень нехарактерно для
                   1984: традиционной японской поэзии. При этом она сравнивает себя с цветком.
                   1985: Вода, цветы, самолюбование - все атрибуты налицо. :-)
                   1986: 
                   1987: Источник:
                   1988: Ёсано Акико. Спутанные волосы. - М.: Эксмо, 2007. - С. 26-27.
                   1989: 
                   1990: Автор:
                   1991: Алексей Трефилов (Калуга)
                   1992: 
                   1993: Вопрос 22:
                   1994: Персонаж игры "Ведьмак 2" считает, что люди и эльфы конфликтуют из-за
                   1995: пустяка. Поэтому некие особенности персонаж описывает с помощью двух
                   1996: прилагательных. Напишите эти прилагательные.
                   1997: 
                   1998: Ответ:
                   1999: Тупоконечный, остроконечный.
                   2000: 
                   2001: Зачет:
                   2002: В любом порядке.
                   2003: 
                   2004: Комментарий:
                   2005: "Война из-за того, остроконечные или тупоконечные у вас уши, так же
                   2006: нелепа, как и конфликт из-за стороны разбивания яйца". Такая вот отсылка
                   2007: к персонажам Свифта.
                   2008: 
                   2009: Источник:
                   2010: Компьютерная игра "Ведьмак 2. Убийца королей".
                   2011: 
                   2012: Автор:
                   2013: Игорь Тюнькин (Москва)
                   2014: 
                   2015: Вопрос 23:
                   2016: Эрих фон Деникен считает, что древние люди на протяжении некоторого
                   2017: времени общались с пришельцами, пока те не улетели. Фон Деникен
                   2018: полагает, что мифы о богах и объекты, такие как в пустыне НАска, - это
                   2019: отголоски своеобразного ЕГО. Термин "ОН" впервые был упомянут в печати в
                   2020: 1945 году. Напишите этот термин.
                   2021: 
                   2022: Ответ:
                   2023: Культ карго.
                   2024: 
                   2025: Зачет:
                   2026: Карго-культ.
                   2027: 
                   2028: Комментарий:
                   2029: Широкую известность термин "культ карго" получил после Второй мировой
                   2030: войны. Американские войска покинули ряд тихоокеанских островов, а
                   2031: местные жители, привыкшие к тому, что американцы снабжали их едой, стали
                   2032: строить копии взлетно-посадочных полос и самолетов из кокосовых пальм и
                   2033: соломы. Они верили, что эти постройки привлекут заполненные грузами
                   2034: самолеты, считавшиеся посланниками духов.
                   2035: 
                   2036: Источник:
                   2037:    1. Д/ф "Новые "Воспоминания о будущем"".
                   2038:    2. http://en.wikipedia.org/wiki/Cargo_cult
                   2039: 
                   2040: Автор:
                   2041: Алексей Трефилов (Калуга)
                   2042: 
                   2043: Вопрос 24:
                   2044: В сказке Петра МордкОвича демиурги разбирают хрустальный купол и
                   2045: нечаянно раскалывают одну из панелей. После чего один спрашивает
                   2046: другого, успел ли он СДЕЛАТЬ ЭТО. Что сделать?
                   2047: 
                   2048: Ответ:
                   2049: Загадать желание.
                   2050: 
                   2051: Комментарий:
                   2052: Осколки хрустального купола неба - это падающие звезды. Демиурги в этой
                   2053: сказке убирали хрустальный купол от геоцентрической системы и собирались
                   2054: устанавливать систему гелиоцентрическую. :-)
                   2055: 
                   2056: Источник:
                   2057: http://bormor.livejournal.com/171643.html
                   2058: 
                   2059: Автор:
                   2060: Алексей Трефилов (Калуга)
                   2061: 
                   2062: Вопрос 25:
                   2063: Хельмут НьЮтон рассказывает, что в 60-х годах в Сен-Тропе жандармерия
                   2064: распыляла с вертолетов несмываемую краску над НИМИ. Над кем?
                   2065: 
                   2066: Ответ:
                   2067: Над нудистами.
                   2068: 
                   2069: Зачет:
                   2070: Над натуристами.
                   2071: 
                   2072: Комментарий:
                   2073: Чтобы потом найти и оштрафовать. Кстати, в старой французской комедии
                   2074: про жандармов из Сен-Тропе тоже охотились на нудистов. :-)
                   2075: 
                   2076: Источник:
                   2077: "Story", 2014, N 5. - С. 120.
                   2078: 
                   2079: Автор:
                   2080: Мария Трефилова (Калуга)
                   2081: 
                   2082: Вопрос 26:
                   2083: В рассказе Евгения Замятина персонаж, отвечая на вопрос, выгнул грудь
                   2084: колесом, выставил ногу вперед и сказал: "Председатель". Какое слово мы
                   2085: пропустили в тексте вопроса?
                   2086: 
                   2087: Ответ:
                   2088: Я.
                   2089: 
                   2090: Комментарий:
                   2091: Отвечая на вопрос, кто в деревне председатель, персонаж принял позу,
                   2092: напоминающую букву "Я" и сказал: "Председатель - я". Как известно,
                   2093: Замятин - автор романа "Мы".
                   2094: 
                   2095: Источник:
                   2096: http://az.lib.ru/z/zamjatin_e_i/text_0150.shtml
                   2097: 
                   2098: Автор:
                   2099: Мария Трефилова (Калуга)
                   2100: 
                   2101: Вопрос 27:
                   2102: Екатерина ЧунькОва рассказывает, что кабинет Пушкина в усадьбе
                   2103: Михайловское воссоздан достаточно точно. Только теперь здесь появился
                   2104: ИКС, какого не было при жизни Пушкина. ИКСОМ мы заменили слово из семи
                   2105: букв. Напишите их в правильной последовательности.
                   2106: 
                   2107: Ответ:
                   2108: Порядок.
                   2109: 
                   2110: Зачет:
                   2111: П, о, р, я, д, о, к.
                   2112: 
                   2113: Комментарий:
                   2114: В кабинете Пушкина всегда был творческий беспорядок. Мы попросили
                   2115: написать буквы в правильном порядке.
                   2116: 
                   2117: Источник:
                   2118: Д/ф "Пушкинские горы". Эфир RTG.
                   2119: 
                   2120: Автор:
                   2121: Алексей Трефилов (Калуга)
                   2122: 
                   2123: Вопрос 28:
                   2124: В старые времена, чтобы добыть ЕГО, использовали багры и сачки. Среди
                   2125: его разновидностей выделяют "капли", "натёки" и "сосульки". Назовите
                   2126: ЕГО.
                   2127: 
                   2128: Ответ:
                   2129: Янтарь.
                   2130: 
                   2131: Комментарий:
                   2132: В Прибалтике багром рыхлили морское дно неподалеку от берега или
                   2133: ворошили водоросли, легкий янтарь всплывал на поверхность, и его
                   2134: собирали сачком. Янтарь нередко различают по тем формам, которые
                   2135: принимала смола при застывании.
                   2136: 
                   2137: Источник:
                   2138:    1. Д/ф "Тайны янтаря". Эфир телеканала "Культура".
                   2139:    2. http://www.nkj.ru/archive/articles/9199/
                   2140: 
                   2141: Автор:
                   2142: Алексей Трефилов (Калуга)
                   2143: 
                   2144: Вопрос 29:
                   2145: (pic: 20140519.jpg)
                   2146:    В этой работе фотохудожник Патрик Гонсалез хотел показать
                   2147: мимолетность молодости, поэтому девушка на портрете держит ЕГО. Назовите
                   2148: ЕГО.
                   2149: 
                   2150: Ответ:
                   2151: Одуванчик.
                   2152: 
                   2153: Комментарий:
                   2154: Молодость так же мимолетна и скоротечна, как цветение одуванчика. На
                   2155: портрете образ девушки как будто сдувается ветром, как пух одуванчика в
                   2156: ее руках. На это также намекают пушинки около ее руки и около ее головы.
                   2157: Слово "мимолетность" - дополнительная подсказка.
                   2158: 
                   2159: Источник:
                   2160: http://patrickgonzales.35photo.ru/photo_396827/
                   2161: 
                   2162: Автор:
                   2163: Мария Трефилова (Калуга)
                   2164: 
                   2165: Вопрос 30:
                   2166: Супруга Франца Иосифа Елизавета Австрийская была вынуждена отказаться от
                   2167: одного своего увлечения. У нее появились проблемы с позвоночником из-за
                   2168: той разновидности, которую она использовала. Назовите эту разновидность
                   2169: двумя словами.
                   2170: 
                   2171: Ответ:
                   2172: Дамское седло.
                   2173: 
                   2174: Зачет:
                   2175: Женское седло.
                   2176: 
                   2177: Комментарий:
                   2178: Елизавета увлекалась верховой ездой. Дамское седло, когда сидят на
                   2179: лошади боком, предполагает скручивание позвоночника.
                   2180: 
                   2181: Источник:
                   2182: "Story", 2013, N 3. - С. 133.
                   2183: 
                   2184: Автор:
                   2185: Мария Трефилова (Калуга)
                   2186: 
                   2187: Вопрос 31:
                   2188: Валентин Берестов в своем стихотворении говорит о том, что усилия, даже
                   2189: кажущиеся напрасными, могут дать результат. Цитата:
                   2190:    Дружно ударились [пропуск 1],
                   2191:    И начался на реке [пропуск 2].
                   2192:    Заполните первый пропуск тремя словами.
                   2193: 
                   2194: Ответ:
                   2195: "... рыбы об лед...".
                   2196: 
                   2197: Комментарий:
                   2198: Фразеологизм "биться как рыба об лед" означает "бесплодно чего-то
                   2199: добиваться", но поэт его переосмыслил. Полностью стихотворение звучит
                   2200: так:
                   2201:    Дружно ударились рыбы об лед,
                   2202:    И начался на реке ледоход.
                   2203: 
                   2204: Источник:
                   2205: http://www.poesis.ru/poeti-poezia/berestov/frm_vers.htm
                   2206: 
                   2207: Автор:
                   2208: Алексей Трефилов (Калуга)
                   2209: 
                   2210: Вопрос 32:
                   2211: По легенде, рыцарь ПаццИно де ПАцци за доблесть при взятии Иерусалима
                   2212: получил камни Храма Гроба Господня и к некоему дню доставил их во
                   2213: Флоренцию. В тот день ПаццИно с помощью этих камней получил... Ответьте
                   2214: максимально точно, что именно.
                   2215: 
                   2216: Ответ:
                   2217: Благодатный огонь.
                   2218: 
                   2219: Зачет:
                   2220: Святой свет; святой огонь; пасхальный огонь; священный огонь; огонь
                   2221: [Храма] Гроба Господня.
                   2222: 
                   2223: Комментарий:
                   2224: Эти камни Паццино доставил к Пасхе и высек с их помощью Благодатный
                   2225: огонь. Обычно этот огонь выносят из Гроба Господня на богослужении
                   2226: накануне Пасхи.
                   2227: 
                   2228: Источник:
                   2229:    1. http://ru.toskana-netz.de/174/2010-0221/scoppio-del-carro-2010.html
                   2230:    2. http://en.wikipedia.org/wiki/Scoppio_del_carro
                   2231:    3. http://www.duomofirenze.it/feste/pasqua_eng.htm
                   2232:    4. http://ru.wikipedia.org/wiki/Благодатный_огонь
                   2233: 
                   2234: Автор:
                   2235: Алексей Трефилов (Калуга)
                   2236: 
                   2237: Вопрос 33:
                   2238: Преследуя человека на темной лестнице, герой Достоевского почувствовал,
                   2239: как пару раз по ПЕРВОМУ ударил край ВТОРОЙ. Назовите ПЕРВЫЙ и ВТОРУЮ.
                   2240: 
                   2241: Ответ:
                   2242: Нос, шинель.
                   2243: 
                   2244: Комментарий:
                   2245: А до того герой преследовал человека на Невском проспекте. Достоевский -
                   2246: славный продолжатель дела Гоголя в написании "петербургских повестей".
                   2247: :-) Очевидно, что в эпизоде отсылка к названиям повестей Гоголя сделана
                   2248: специально.
                   2249: 
                   2250: Источник:
                   2251: http://az.lib.ru/d/dostoewskij_f_m/text_0140.shtml
                   2252: 
                   2253: Автор:
                   2254: Алексей Трефилов (Калуга)
                   2255: 
                   2256: Вопрос 34:
                   2257: Знаете, что меня всегда удивляло в английском языке? Все эти слова с
                   2258: кучей разных значений. Вот попадется вам слово "boobs" [бубз] - не
                   2259: спешите радоваться. Возможно, речь не про женскую грудь. А про чьи-то
                   2260: "досадные ошибки". Или, скажем, репортаж в новостях. Когда журналист
                   2261: берет микрофон, чтобы сказать в кадре что-то несмешное. И как такой
                   2262: прием называется?
                   2263: 
                   2264: Ответ:
                   2265: Стендап.
                   2266: 
                   2267: Зачет:
                   2268: Стэндап; стенд-ап; стэнд-ап; stand up; standup; stand-up.
                   2269: 
                   2270: Комментарий:
                   2271: Репортерский прием, когда журналист работает непосредственно в кадре на
                   2272: месте событий, тоже называется "стендап". Текст вопроса стилизован под
                   2273: монолог стендап-комика. Надеемся, вас это немного позабавило. :-)
                   2274: 
                   2275: Источник:
                   2276:    1. http://ru.wikipedia.org/wiki/Стендап
                   2277:    2. http://www.ehow.com/how_4457601_do-stand-up-tv-news.html
                   2278: 
                   2279: Автор:
                   2280: Алексей Трефилов, Мария Трефилова (Калуга)
                   2281: 
                   2282: Вопрос 35:
                   2283: Во время московского кинофестиваля Джина Лоллобриджида и Элизабет Тейлор
                   2284: улыбались и всё время держались вместе. Однако через некоторое время ОН
                   2285: получил два судебных иска. Напишите его фамилию, использовав дефис.
                   2286: 
                   2287: Ответ:
                   2288: Сен-Лоран.
                   2289: 
                   2290: Комментарий:
                   2291: Так получилось, что у актрис оказались два одинаковых платья от одного
                   2292: модельного агентства. Когда это выяснилось, Элизабет и Джина решили
                   2293: выйти из положения, держась всё время вместе. Словно так и было
                   2294: задумано.
                   2295: 
                   2296: Источник:
                   2297: Легенды мирового кино. Джина Лоллобриджида.
                   2298: 
                   2299: Автор:
                   2300: Игорь Тюнькин (Москва)
                   2301: 
                   2302: Вопрос 36:
                   2303: В абхазской сказке рассказчик, упоминая ИХ, предлагает убедиться: вот,
                   2304: смотрите, до сих пор еще мокрые. Назовите ИХ.
                   2305: 
                   2306: Ответ:
                   2307: Усы.
                   2308: 
                   2309: Комментарий:
                   2310: Это присказка в конце сказки. В абхазском варианте ее начало звучит так:
                   2311: "И я на том пиру был, усы в вине мочил...".
                   2312: 
                   2313: Источник:
                   2314: Сказки народов Закавказья. - Цхинвали: Ирыстон, 1987. - С. 14.
                   2315: 
                   2316: Автор:
                   2317: Алексей Трефилов (Калуга)
                   2318: 
                   2319: Тур:
                   2320: Этап 4
                   2321: 
                   2322: Дата:
                   2323: 14-Dec-2014
                   2324: 
                   2325: Редактор:
                   2326: Дмитрий Великов (Москва), Александр Рождествин (Самара), Сергей Спешков
                   2327: (Москва)
                   2328: 
                   2329: Инфо:
                   2330: Редакторы благодарят за помощь в подготовке пакета команды "Биркиркара",
                   2331: "Гросс-бух" и "Эрудиты" (все - Москва), а также Антона Тахтарова и Елену
                   2332: Конькову (оба - Самара), Евгения Миротина (Минск), Николая Рябых
                   2333: (Королёв), Александра Маркова, Ивана Семушина, Екатерину и Станислава
                   2334: Мереминских, Романа Немучинского, Владимира Цвингли, Сергея Даровских
                   2335: (все - Москва), Ивана Ефремова (Ростов-на-Дону), Константина Сахарова
                   2336: (Ивантеевка), Дмитрия Когана (Фридрихсхафен), Александра Митрякова
                   2337: (Ижевск), Александра Коробейникова (Саратов).
                   2338: 
                   2339: Вопрос 1:
                   2340: Провожая Мари в волшебную страну, Щелкунчик распахивает перед нею дверь
                   2341: ИКСА. Назовите ИКС двумя словами.
                   2342: 
                   2343: Ответ:
                   2344: Платяной шкаф.
                   2345: 
                   2346: Зачет:
                   2347: Волшебный шкаф.
                   2348: 
                   2349: Комментарий:
                   2350: Позже подобным путем дети попадали в Нарнию.
                   2351: 
                   2352: Источник:
                   2353: Э.Т.А. Гофман, Щелкунчик и мышиный король.
                   2354: http://www.deti-online.com/skazki/skazki-gofmana/shelkunchik-i-myshinyi-korol/
                   2355: 
                   2356: Автор:
                   2357: Дмитрий Великов (Москва)
                   2358: 
                   2359: Вопрос 2:
                   2360: Персонаж фильма Дэвида Финчера в русском переводе говорит, что чувствует
                   2361: себя боксерской грушей, из которой пытаются выбить побольше денег. Что
                   2362: персонаж упоминает в этом месте в оригинале?
                   2363: 
                   2364: Ответ:
                   2365: Пиньяту.
                   2366: 
                   2367: Комментарий:
                   2368: Видимо, переводчики решили избавиться от малоизвестной реалии.
                   2369: 
                   2370: Источник:
                   2371: Фильм "Игра" (1997).
                   2372: 
                   2373: Автор:
                   2374: Сергей Спешков (Москва)
                   2375: 
                   2376: Вопрос 3:
                   2377: Читатели недавно опубликованного романа обратили внимание на подробное
                   2378: описание некоторых деталей женского гардероба, после чего продажи книги
                   2379: сильно выросли. Назовите автора этого романа.
                   2380: 
                   2381: Ответ:
                   2382: [Роберт] Гэлбрейт.
                   2383: 
                   2384: Зачет:
                   2385: [Джоан] Роулинг.
                   2386: 
                   2387: Комментарий:
                   2388: Детектив "Зов кукушки" вышел под именем Роберта Гэлбрейта, но
                   2389: внимательные читатели быстро вычислили, что под этим псевдонимом
                   2390: скрывается Джоан Роулинг.
                   2391: 
                   2392: Источник:
                   2393: http://lenta.ru/news/2013/07/14/rowling/
                   2394: 
                   2395: Автор:
                   2396: Александр Рождествин (Самара)
                   2397: 
                   2398: Вопрос 4:
                   2399: Внимание, в вопросе есть замена.
                   2400:    Во время Второй мировой войны ИКСЫ делали из серебра, меди и
                   2401: марганца, потому что весь ИКС отправлялся на военные нужды. Назовите
                   2402: ИКС.
                   2403: 
                   2404: Ответ:
                   2405: Никель.
                   2406: 
                   2407: Комментарий:
                   2408: Никель - обиходное название американской монеты в пять центов. Так
                   2409: называемые "военные никели" никакого никеля на самом деле не содержали.
                   2410: Кстати, и сейчас они делаются из сплава, в котором три четверти - медь.
                   2411: 
                   2412: Источник:
                   2413: http://en.wikipedia.org/wiki/Nickel_(United_States_coin)
                   2414: 
                   2415: Автор:
                   2416: Александр Рождествин (Самара)
                   2417: 
                   2418: Вопрос 5:
                   2419: Со временем ОНА стала утрачивать первоначальное назначение, и в
                   2420: правление губернатора ЛАклана МаккуОри для аналогичных целей стали всё
                   2421: чаще использовать объект, находящийся еще южнее. Назовите этот объект.
                   2422: 
                   2423: Ответ:
                   2424: [Остров] Тасмания.
                   2425: 
                   2426: Зачет:
                   2427: [Остров] Земля Ван-ДИмена.
                   2428: 
                   2429: Комментарий:
                   2430: Австралия поначалу была местом ссылки преступников. Когда заканчивался
                   2431: срок каторги, они чаще всего оставались свободными поселенцами. А для
                   2432: рецидивистов и редких новых партий каторжников стали использовать
                   2433: Тасманию. Фактически Тасмания для Австралии стала тем, чем раньше была
                   2434: Австралия для Британии.
                   2435: 
                   2436: Источник:
                   2437:    1. http://en.wikipedia.org/wiki/Tasmania
                   2438:    2. http://en.wikipedia.org/wiki/Lachlan_Macquarie
                   2439:    3. http://en.wikipedia.org/wiki/New_South_Wales
                   2440: 
                   2441: Автор:
                   2442: Сергей Спешков (Москва)
                   2443: 
                   2444: Вопрос 6:
                   2445: В рекламном ролике одного аукционного дома люди отчаянно выкрикивают
                   2446: цену каждый со своего места, а в это время под ними перемещаются
                   2447: "устроители" аукциона с НИМ в руках. Назовите ЕГО.
                   2448: 
                   2449: Ответ:
                   2450: [Пожарный] тент.
                   2451: 
                   2452: Зачет:
                   2453: [Пожарный] брезент, [пожарный] батут.
                   2454: 
                   2455: Комментарий:
                   2456: Дело происходит на пожаре. Каждый невольный "участник" аукциона из
                   2457: своего окна предлагает цену, а пожарные внизу мечутся от одного к
                   2458: другому.
                   2459: 
                   2460: Источник:
                   2461: http://www.youtube.com/watch?v=fvjGKZNT1vQ
                   2462: 
                   2463: Автор:
                   2464: Сергей Спешков (Москва)
                   2465: 
                   2466: Вопрос 7:
                   2467: Художник Альберт БИрштадт серьезно подходил к выбору вида для своих
                   2468: пейзажей. В статье русскоязычной википедии "БИрштадт" присутствует число
                   2469: 4287. Какое слово мы пропустили в предыдущем предложении?
                   2470: 
                   2471: Ответ:
                   2472: Гора.
                   2473: 
                   2474: Зачет:
                   2475: Вершина, пик.
                   2476: 
                   2477: Комментарий:
                   2478: Речь о статье Википедии "БИрштадт (гора)", которую покорил художник в
                   2479: поисках вдохновения. Гора впоследствии была названа в его честь.
                   2480: Впрочем, в итоге картины он написал только у подножия горы.
                   2481: 
                   2482: Источник:
                   2483:    1. http://www.14ers.com/php14ers/historyview.php?parmpeak=Mt.%20Bierstadt&parmcat=Name%20History
                   2484:    2. http://en.wikipedia.org/wiki/Albert_Bierstadt
                   2485:    3. http://ru.wikipedia.org/wiki/Бирштадт_(гора)
                   2486: 
                   2487: Автор:
                   2488: Александр Рождествин (Самара)
                   2489: 
                   2490: Вопрос 8:
                   2491: Среди причин поражения сипаев в восстании в середине девятнадцатого века
                   2492: было техническое превосходство британцев не только в вооружении. Один из
                   2493: приговоренных к смертной казни мятежников назвал ЕГО "проклятой
                   2494: веревкой, которая душит". Назовите ЕГО, использовав слово греческого
                   2495: происхождения.
                   2496: 
                   2497: Ответ:
                   2498: Телеграфный провод.
                   2499: 
                   2500: Зачет:
                   2501: Телеграф, телеграфный кабель.
                   2502: 
                   2503: Комментарий:
                   2504: Благодаря новому для того времени средству связи британцы могли гораздо
                   2505: лучше и быстрее координировать свои действия.
                   2506: 
                   2507: Источник:
                   2508: Ниал Фергюсон. Империя. Чем современный мир обязан Британии.
                   2509: http://www.flibusta.net/b/378579/read
                   2510: 
                   2511: Автор:
                   2512: Сергей Спешков (Москва)
                   2513: 
                   2514: Вопрос 9:
                   2515: Постоянно курящий герой Джозефа Хеллера носит фамилию ИКС. В африканском
                   2516: фольклоре ИКСОВ уважают за то, что те не боятся солдат. Какое слово в
                   2517: этом вопросе мы заменили на ИКС?
                   2518: 
                   2519: Ответ:
                   2520: Трубкозуб.
                   2521: 
                   2522: Зачет:
                   2523: Аардваарк, Aardvaark.
                   2524: 
                   2525: Комментарий:
                   2526: У героя Хеллера в зубах постоянно находится курительная трубка.
                   2527: Трубкозубы питаются термитами и муравьями и не боятся муравьев-солдат.
                   2528: Впрочем, толстая кожа хорошо защищает от их укусов.
                   2529: 
                   2530: Источник:
                   2531:    1. http://lib.ru/INPROZ/HELLER/catch22.txt
                   2532:    2. http://www.themagicalbuffet.com/Issues/Vol02_Iss07/Article_049.html
                   2533:    3. http://ru.wikipedia.org/wiki/Трубкозуб
                   2534: 
                   2535: Автор:
                   2536: Сергей Спешков (Москва)
                   2537: 
                   2538: Вопрос 10:
                   2539: В 1815 году герцог Ришелье вернулся во Францию, для того чтобы занять
                   2540: пост премьер-министра. Говоря об этом событии, Леонид Парфёнов вспомнил
                   2541: известную фразу, в которой последнее слово заменил топонимом.
                   2542: Воспроизведите получившуюся фразу.
                   2543: 
                   2544: Ответ:
                   2545: Париж стоит Одессы.
                   2546: 
                   2547: Комментарий:
                   2548: Дюк Ришелье вернулся в Париж из России, оставив должность
                   2549: генерал-губернатора Новороссийского края и одесского градоначальника.
                   2550: 
                   2551: Источник:
                   2552:    1. http://en.wikipedia.org/wiki/Armand-Emmanuel_de_Vignerot_du_Plessis,_Duc_de_Richelieu
                   2553:    2. http://www.youtube.com/watch?v=OXnJPzGSWhk&t=38m48s
                   2554: 
                   2555: Автор:
                   2556: Александр Рождествин (Самара)
                   2557: 
                   2558: Вопрос 11:
                   2559: В одной компьютерной игре можно найти могилу некоего Марка. Эпитафия
                   2560: гласит: "Он был полон великих идей, но не работал ни дня". Напишите
                   2561: четырехбуквенную фамилию этого Марка.
                   2562: 
                   2563: Ответ:
                   2564: Сизм.
                   2565: 
                   2566: Зачет:
                   2567: Сист.
                   2568: 
                   2569: Комментарий:
                   2570: (pic: 20140520.jpg)
                   2571:    Это своего рода шутка от разработчиков, которые не очень-то верят в
                   2572: марксизм.
                   2573: 
                   2574: Источник:
                   2575: Игра "Divinity: Original sin".
                   2576: 
                   2577: Автор:
                   2578: Александр Рождествин (Самара)
                   2579: 
                   2580: Вопрос 12:
                   2581: Описывая ощущения от разговора с Антеем, Данте упоминает болонскую
                   2582: АЛЬФУ. Возможно, это связано с тем, что другая АЛЬФА к тому времени была
                   2583: еще не закончена. Какое слово мы заменили словом "АЛЬФА"?
                   2584: 
                   2585: Ответ:
                   2586: Башня.
                   2587: 
                   2588: Комментарий:
                   2589: Склонившегося к нему гиганта Данте сравнивает с Гаризендой - падающей
                   2590: башней, находящейся в Болонье. Самая известная падающая башня -
                   2591: пизанская.
                   2592: 
                   2593: Источник:
                   2594:    1. Данте Алигьери. Божественная комедия.
                   2595: http://www.infoliolib.info/flit/dante/ad31.html
                   2596:    2. http://ru.wikipedia.org/wiki/Пизанская_башня
                   2597: 
                   2598: Автор:
                   2599: Александр Рождествин (Самара)
                   2600: 
                   2601: Вопрос 13:
                   2602: В стихотворении Николая Заболоцкого "Стирка белья" ОНА появляется из
                   2603: корыта. Назовите ЕЕ.
                   2604: 
                   2605: Ответ:
                   2606: Афродита.
                   2607: 
                   2608: Зачет:
                   2609: Венера.
                   2610: 
                   2611: Комментарий:
                   2612: Морскую пену и раковину у поэта, по-видимому, символизируют мыльная пена
                   2613: и корыто.
                   2614: 
                   2615: Источник:
                   2616: http://rupoem.ru/zabolockij/v-storone-ot.aspx
                   2617: 
                   2618: Автор:
                   2619: Сергей Спешков (Москва)
                   2620: 
                   2621: Вопрос 14:
                   2622: Кинокомпании не хотели снимать этот фильм, поскольку ранее в прокате
                   2623: провалилась картина "Братство". Автор сценария этого фильма на встрече с
                   2624: продюсером сказал: "Я задолжал одиннадцать тысяч, если не верну, мне
                   2625: сломают руку". Назовите автора сценария.
                   2626: 
                   2627: Ответ:
                   2628: [Марио] Пьюзо.
                   2629: 
                   2630: Комментарий:
                   2631: Речь идет о сценарии фильма "Крестный отец". Поскольку другой фильм об
                   2632: организованной преступности незадолго до того провалился, снимать новый
                   2633: компании не решались.
                   2634: 
                   2635: Источник:
                   2636: П. Бискинд. Беспечные ездоки, бешеные быки.
                   2637: http://www.flibusta.net/b/240883/read
                   2638: 
                   2639: Автор:
                   2640: Сергей Спешков (Москва)
                   2641: 
                   2642: Вопрос 15:
                   2643: Византийский историк Анна КомнинА, вспоминая времена могущества Римской
                   2644: империи, упоминает еще и ДионИсовы на востоке. Какое слово мы пропустили
                   2645: в этом вопросе?
                   2646: 
                   2647: Ответ:
                   2648: Столпы.
                   2649: 
                   2650: Зачет:
                   2651: Столбы.
                   2652: 
                   2653: Комментарий:
                   2654: Анна пишет, что империя простиралась от столпов Геракла на западе до
                   2655: столпов Диониса на востоке. Впрочем, современные историки не в состоянии
                   2656: объяснить, что именно Анна понимала под столпами Диониса.
                   2657: 
                   2658: Источник:
                   2659: Анна Комнина. Алексиада. http://www.flibusta.net/b/28096/read
                   2660: 
                   2661: Автор:
                   2662: Александр Рождествин (Самара)
                   2663: 
                   2664: Вопрос 16:
                   2665: Первая часть термина "сахамарАна" переводится как "вместе". Первая часть
                   2666: термина "анумарАна" - как "после". Разновидностями чего являлись
                   2667: сахамарАна и анумарАна?
                   2668: 
                   2669: Ответ:
                   2670: САти.
                   2671: 
                   2672: Зачет:
                   2673: [Само]сожжения вдовы.
                   2674: 
                   2675: Комментарий:
                   2676: В Индии жены умерших мужей могли сжигаться либо вместе с телом, либо
                   2677: после. Это определялось в зависимости от определенных условий.
                   2678: 
                   2679: Источник:
                   2680:    1. http://en.wikipedia.org/wiki/Sati_(practice)#Anumarana
                   2681:    2. http://en.wikipedia.org/wiki/Anumarana
                   2682: 
                   2683: Автор:
                   2684: Сергей Спешков (Москва)
                   2685: 
                   2686: Вопрос 17:
                   2687: Джеймс Джойс описывает, как змеи увиваются вокруг посоха. Что потом
                   2688: делают из чешуи этих змей?
                   2689: 
                   2690: Ответ:
                   2691: Пиво.
                   2692: 
                   2693: Комментарий:
                   2694: Так Джойс описывает хмель, который является одним из основных
                   2695: ингредиентов при производстве пива. Хмель - вьющееся растение, шишки
                   2696: которого напоминают чешую.
                   2697: 
                   2698: Источник:
                   2699: Джеймс Джойс. Улисс.
                   2700: 
                   2701: Автор:
                   2702: Сергей Спешков (Москва)
                   2703: 
                   2704: Вопрос 18:
                   2705: ДЕян СУджич иронично назвал его богом домашнего очага. Какое
                   2706: изобретение, по словам СУджича, избавило людей от необходимости
                   2707: регулярного ритуального поклонения этому "богу"?
                   2708: 
                   2709: Ответ:
                   2710: Пульт дистанционного управления.
                   2711: 
                   2712: Зачет:
                   2713: ПДУ, пульт от телевизора, прочие эквивалентные ответы со словом "пульт"
                   2714: без неверных уточнений.
                   2715: 
                   2716: Комментарий:
                   2717: Суджич назвал богом домашнего очага телевизор. До изобретения ПДУ для
                   2718: включения телевизора и переключения каналов нужно было подходить к
                   2719: телевизору и буквально склоняться перед ним.
                   2720: 
                   2721: Источник:
                   2722: Деян Суджич. Язык вещей. http://www.flibusta.net/b/339479/read
                   2723: 
                   2724: Автор:
                   2725: Сергей Спешков (Москва)
                   2726: 
                   2727: Вопрос 19:
                   2728: Слово "АЛЬФА" в вопросе - замена.
                   2729:    У аристократа Герберта Китченера было плохое зрение. Поэтому Китченер
                   2730: с юмором использовал в качестве АЛЬФ Выстрел, Промах и Проклятье. Какая
                   2731: АЛЬФА стала названием произведения девятнадцатого века?
                   2732: 
                   2733: Ответ:
                   2734: Муму.
                   2735: 
                   2736: Комментарий:
                   2737: АЛЬФА - кличка собаки. Китченер назвал своих охотничьих собак Bang
                   2738: [бэнг], Miss [мисс], Damn [дэмн]. "Муму" - рассказ Тургенева.
                   2739: 
                   2740: Источник:
                   2741:    1. http://books.google.ru/books?id=7LSeAwAAQBAJ&pg=PT114#v=onepage&q&f=false
                   2742:    2. И. Тургенев. Муму.
                   2743: 
                   2744: Автор:
                   2745: Сергей Спешков (Москва)
                   2746: 
                   2747: Вопрос 20:
                   2748: Героиня романа, который написал Леонид Леонов, после того как всемирная
                   2749: революция обеспечивала счастье трудящихся, думала, что же дальше, и, как
                   2750: пишет автор, начинала блуждать в лабиринте ИХ. Назовите ИХ двумя
                   2751: словами, начинающимися на одну и ту же букву.
                   2752: 
                   2753: Ответ:
                   2754: Придаточные предложения.
                   2755: 
                   2756: Зачет:
                   2757: Подчиненные предложения.
                   2758: 
                   2759: Комментарий:
                   2760: Так Леонов описывает мучения героини при составлении речи, в которой,
                   2761: очевидно, большинство фраз были до крайности тяжеловесными. Почти как
                   2762: этот вопрос, равно как и комментарий к нему, равно как и мучения авторов
                   2763: при составлении оных.
                   2764: 
                   2765: Источник:
                   2766: Л. Леонов. Собрание сочинений в пяти томах. - М.: Государственное
                   2767: издательство художественной литературы. - Т. 6 (дополнительный). - С.
                   2768: 212.
                   2769: 
                   2770: Автор:
                   2771: Дмитрий Великов (Москва)
                   2772: 
                   2773: Вопрос 21:
                   2774: В этом вопросе слово "ИКС" заменяет другое слово.
                   2775:    ИКСА известного автора вплоть до 1930-х годов можно было увидеть в
                   2776: немецких учебниках по биологии, несмотря на наличие более точных работ
                   2777: Одри и Стаббса, а также фотографий. Назовите ИКСА.
                   2778: 
                   2779: Ответ:
                   2780: Носорог.
                   2781: 
                   2782: Комментарий:
                   2783: Речь идет о знаменитой гравюре "Носорог" Альбрехта Дюрера. Для своего
                   2784: времени она была вполне точна, но всё же много в ней было и выдуманного.
                   2785: В родной Германии к Дюреру испытывали большое уважение.
                   2786: 
                   2787: Источник:
                   2788: http://en.wikipedia.org/wiki/D%C3%BCrer's_Rhinoceros
                   2789: 
                   2790: Автор:
                   2791: Александр Рождествин (Самара)
                   2792: 
                   2793: Вопрос 22:
                   2794: На плакате, рекламирующем услуги одного специалиста, облака постепенно
                   2795: принимают привычную правильную форму. О каком специалисте идет речь?
                   2796: 
                   2797: Ответ:
                   2798: О логопеде.
                   2799: 
                   2800: Комментарий:
                   2801: Облака в данном случае - то, куда помещаются реплики говорящих.
                   2802: 
                   2803: Источник:
                   2804: Рекламный плакат по адресу: Москва, ул. Шереметьевская, д. 91, корп. 1.
                   2805: 
                   2806: Автор:
                   2807: Сергей Спешков (Москва)
                   2808: 
                   2809: Вопрос 23:
                   2810: Чаплин вспоминал, что его отец был чрезвычайно эгоистичным человеком.
                   2811: Какое-то время он даже спорил со своей женой, говоря что еще одно ОНО в
                   2812: семье будет ему мешать. Назовите ЕГО точно.
                   2813: 
                   2814: Ответ:
                   2815: [Имя] Чарли.
                   2816: 
                   2817: Комментарий:
                   2818: Чарли Чаплин-младший так вспоминал о своем отце, знаменитом киноактере.
1.4     ! rubashki 2819: Тот сперва не хотел, чтобы сын был назван в честь него.
1.1       rubashki 2820: 
                   2821: Источник:
                   2822: http://vozduh.afisha.ru/books/charli-chaplin-pitera-akroyda/
                   2823: 
                   2824: Автор:
                   2825: Александр Рождествин (Самара)
                   2826: 
                   2827: Вопрос 24:
                   2828: [Ведущему: слово "чьего" в последней фразе вопроса следует прочитать
                   2829: максимально отчетливо, чтобы команды не услышали "чего".]
                   2830:    В 1910 году будущий лауреат Нобелевской премии Вернер фон ХЕйдестам
                   2831: написал книгу "Шведы и их вожди". Людмила БрАуде считает эту книгу
                   2832: своеобразным продолжением проекта. Чьего?
                   2833: 
                   2834: Ответ:
                   2835: [Сельмы] ЛАгерлёф.
                   2836: 
                   2837: Комментарий:
                   2838: Начало этому "проекту" было положено Сельмой Лагерлёф, чья книга о
                   2839: Нильсе стала своеобразным учебником географии Швеции. Другой нобелиат
                   2840: написал учебник истории Швеции.
                   2841: 
                   2842: Источник:
                   2843:    1. http://bookz.ru/authors/sel_ma-lagerlef/udivitel_858/1-udivitel_858.html
                   2844:    2. http://dic.academic.ru/dic.nsf/enc_colier/5165/
                   2845: 
                   2846: Автор:
                   2847: Александр Рождествин (Самара)
                   2848: 
                   2849: Вопрос 25:
                   2850: На рисунке Алексея Булатова человеку, сидящему перед монитором, вручают
                   2851: ЕЕ со знаком копирайта. Назовите ЕЕ.
                   2852: 
                   2853: Ответ:
                   2854: Черная метка.
                   2855: 
                   2856: Комментарий:
                   2857: Карикатура посвящена соблюдению авторских прав. Человек изображен в виде
                   2858: пирата.
                   2859: 
                   2860: Источник:
                   2861: http://caricatura.ru/art/bulatov/url/parad/bulatov/3704/
                   2862: 
                   2863: Автор:
                   2864: Сергей Спешков (Москва)
                   2865: 
                   2866: Вопрос 26:
                   2867: Дуплет.
                   2868:    1. В произведении Иосифа Бродского ЕГО настойчивое соло заканчивается
                   2869: овациями. Назовите ЕГО.
                   2870:    2. Лирический герой Иосифа Бродского, говоря о НЕЙ, упоминает Музу и
                   2871: Шиву. Назовите ЕЕ.
                   2872: 
                   2873: Ответ:
                   2874:    1. Комар.
                   2875:    2. Муха.
                   2876: 
                   2877: Комментарий:
                   2878:    1. Зуденье комара, очевидно, заканчивается хлопками.
                   2879:    2. Крылатая, как муза поэта, и с шестью конечностями.
                   2880: 
                   2881: Источник:
                   2882: http://lib.ru/BRODSKIJ/brodsky_poetry.txt
                   2883: 
                   2884: Автор:
                   2885: Сергей Спешков (Москва)
                   2886: 
                   2887: Вопрос 27:
                   2888: Мёрфи Купер, героиня фильма "ИнтерстЕллар", с детства интересуется
                   2889: физикой, а вырастая, становится ученым и поступает на работу в НАСА.
                   2890: Режиссер фильма КрИстофер НОлан вспоминает, что сделал изменение в
                   2891: изначальном сценарии после некоего события. Ответьте абсолютно точно,
                   2892: что это было за событие.
                   2893: 
                   2894: Ответ:
                   2895: Рождение дочери.
                   2896: 
                   2897: Комментарий:
                   2898: Сперва Мёрфи Купер был мальчиком, однако после рождения дочери Нолан
                   2899: исправил сценарий, сделав Мёрфи Купер девочкой. Девочка, которая
                   2900: увлекается физикой и космосом, - довольно необычный персонаж в
                   2901: голливудских фильмах.
                   2902: 
                   2903: Источник:
                   2904: http://vozduh.afisha.ru/cinema/kristofer-nolan-stivenu-hokingu-film-ponravilsya/
                   2905: 
                   2906: Автор:
                   2907: Александр Рождествин (Самара)
                   2908: 
                   2909: Вопрос 28:
                   2910: [Ведущему: максимально четко произнести название вина айсвАйн.]
                   2911:    После завершения карьеры известный человек открыл собственную
                   2912: винодельню. Сколько долларов стоит вино айсвАйн из его коллекции?
                   2913: 
                   2914: Ответ:
                   2915: 99.
                   2916: 
                   2917: Зачет:
                   2918: 99 долларов [США].
                   2919: 
                   2920: Комментарий:
                   2921: Бывший хоккеист Уэйн Грецки - владелец собственного винного брэнда. С
                   2922: одной стороны, 99 - игровой номер игрока, с другой стороны, такая цена
                   2923: соответствует психологическим принципам ценообразования. Айсвайн -
                   2924: буквально "ледяное вино", для кого-то, возможно, это стало подсказкой.
                   2925: 
                   2926: Источник:
                   2927: http://www.gretzkyestateswines.com/wines/99-series-1/cabernetfranc-icewine.html
                   2928: 
                   2929: Автор:
                   2930: Сергей Спешков (Москва)
                   2931: 
                   2932: Вопрос 29:
                   2933: Психолог ГУстав ГИлберт вспоминает, что к концу ЕГО один из главных
                   2934: участников всё чаще появлялся без галстука или с расстегнутой пуговицей.
                   2935: Назовите ЕГО.
                   2936: 
                   2937: Ответ:
                   2938: Нюрнбергский процесс.
                   2939: 
                   2940: Комментарий:
                   2941: Речь идет о Риббентропе. Гилберт считал, что на подсознательном уровне
                   2942: галстук вызывал у Риббентропа ассоциации с петлей виселицы, всё туже
                   2943: затягивающейся вокруг его шеи.
                   2944: 
                   2945: Источник:
                   2946: Густав Гилберт. Нюрнбергский дневник.
                   2947: 
                   2948: Автор:
                   2949: Александр Рождествин (Самара)
                   2950: 
                   2951: Вопрос 30:
                   2952: В одном литературоведческом комментарии автор вопроса в середине
                   2953: предложения обнаружил слово "ниже", написанное с большой буквы. Напишите
                   2954: слово, которое стояло сразу перед ним.
                   2955: 
                   2956: Ответ:
                   2957: Поручик.
                   2958: 
                   2959: Зачет:
                   2960: Подпоручик.
                   2961: 
                   2962: Комментарий:
                   2963: То ли в результате опечатки, то ли по незнанию людей, оцифровывавших
                   2964: текст, с поручиком Киже произошла еще одна итерация искажения, и он стал
                   2965: "поручиком Ниже".
                   2966: 
                   2967: Источник:
                   2968: http://www.james-joyce.ru/ulysses/ulysses-text.htm
                   2969: 
                   2970: Автор:
                   2971: Сергей Спешков (Москва)
                   2972: 
                   2973: Вопрос 31:
                   2974: Русские казакИ после НЕЕ продавали лошадей по 15-20 рублей, не зная, что
                   2975: реальная их стоимость была раз в десять выше. Назовите ЕЕ тремя словами.
                   2976: 
                   2977: Ответ:
                   2978: Атака легкой кавалерии.
                   2979: 
                   2980: Зачет:
                   2981: Атака легкой бригады; битва при Балаклаве.
                   2982: 
                   2983: Комментарий:
                   2984: В этой атаке участвовал цвет британской аристократии. Переловив выживших
                   2985: лошадей этой, как ее называли в русских войсках, "сумашедшей кавалерии",
                   2986: казаки, не задумываясь, сбывали их с рук перекупщикам, которые уже
                   2987: знали, какова истинная цена дорогих кровных рысаков.
                   2988: 
                   2989: Источник:
                   2990: Е. Тарле. Крымская война.
                   2991: 
                   2992: Автор:
                   2993: Александр Рождествин (Самара)
                   2994: 
                   2995: Вопрос 32:
                   2996: Когда этой территории давали название, бОльшая часть американского
                   2997: континента была еще не освоена европейцами. Ниал Фергюсон считает это
                   2998: дополнительным основанием для выбора названия. О какой территории идет
                   2999: речь?
                   3000: 
                   3001: Ответ:
                   3002: Виргиния.
                   3003: 
                   3004: Зачет:
                   3005: Вирджиния.
                   3006: 
                   3007: Комментарий:
                   3008: Не только в честь королевы-девственницы Елизаветы I.
                   3009: 
                   3010: Источник:
                   3011: Ниал Фергюсон. Империя. Чем современный мир обязан Британии.
                   3012: http://www.flibusta.net/b/378579/read
                   3013: 
                   3014: Автор:
                   3015: Сергей Спешков (Москва)
                   3016: 
                   3017: Вопрос 33:
                   3018: История ИХ появления связана с американскими военными летчиками, которые
                   3019: не были готовы к мирной жизни, зато были прекрасными пилотами и
                   3020: разбирались в двигателях. Назовите ИХ двумя словами, начинающимися на
                   3021: одну ту же букву.
                   3022: 
                   3023: Ответ:
                   3024: "Ангелы ада".
                   3025: 
                   3026: Комментарий:
                   3027: По той же легенде, многие из них были из эскадрильи, названной в честь
                   3028: фильма Говарда Хьюза. Так они объединились и основали байк-клуб, где их
                   3029: навыки оказались востребованы.
                   3030: 
                   3031: Источник:
                   3032: http://affa.hells-angels.com/hamc-history/
                   3033: 
                   3034: Автор:
                   3035: Александр Рождествин (Самара)
                   3036: 
                   3037: Вопрос 34:
                   3038: Персонаж Уэса Андерсона, говоря о том, как вести себя во время допроса,
                   3039: упоминает ЕЕ. Чтобы показать ЕЕ на жестовом языке, пальцы одной руки
                   3040: вначале держат в виде латинской буквы V [вэ], а потом сводят вместе.
                   3041: Назовите ЕЕ.
                   3042: 
                   3043: Ответ:
                   3044: Застежка-молния.
                   3045: 
                   3046: Зачет:
                   3047: Молния, zip, zipper.
                   3048: 
                   3049: Комментарий:
                   3050: Персонаж говорит, что надо закрыть рот на молнию. В языке жестов пальцы,
                   3051: которые держат в виде буквы V, означают открытую молнию, которую
                   3052: движением пальцев второй руки как бы "застегивают".
                   3053: 
                   3054: Источник:
                   3055:    1. Фильм "Отель "Гранд Будапешт"".
                   3056:    2. http://www.lifeprint.com/asl101/pages-signs/z/zip.htm
                   3057: 
                   3058: Автор:
                   3059: Сергей Спешков (Москва)
                   3060: 
                   3061: Вопрос 35:
                   3062: Выступая с лекцией перед американскими студентами, австриец Виктор
                   3063: Франкл иронично похвастался, что использует ПЕРВЫЙ без малейших
                   3064: признаков ВТОРОГО. Назовите ПЕРВЫЙ и ВТОРОЙ в правильном порядке.
                   3065: 
                   3066: Ответ:
                   3067: Акцент, английский [язык].
                   3068: 
                   3069: Зачет:
                   3070: Обязательно в правильном порядке!
                   3071: 
                   3072: Комментарий:
                   3073: Франкл пошутил, что говорит на чистейшем акценте без малейших признаков
                   3074: английского.
                   3075: 
                   3076: Источник:
                   3077: http://www.youtube.com/watch?v=fD1512_XJEw&t=33s
                   3078: 
                   3079: Автор:
                   3080: Александр Рождествин (Самара)
                   3081: 
                   3082: Вопрос 36:
                   3083: На картине Нормана Роквелла "Русский класс" школьники сидят за партами
                   3084: перед бюстом Ленина. На этой картине ТАМ на первом месте несколько раз
                   3085: стоит предпоследняя. Ответьте максимально точно, где - ТАМ?
                   3086: 
                   3087: Ответ:
                   3088: [В надписи] "Учиться[, учиться и учиться]".
                   3089: 
                   3090: Зачет:
                   3091: По слову "учиться" или "Yчиться".
                   3092: 
                   3093: Комментарий:
                   3094: Американский художник не смог до конца справиться с кириллицей и букву
                   3095: "У" в цитате из Ленина написал как "Y" [уай].
                   3096: 
                   3097: Источник:
                   3098:    1. http://en.wikipedia.org/wiki/Russian_Schoolroom
                   3099:    2. http://ru.wikipedia.org/wiki/Файл:Soviet_classroom-rockwell.jpg
                   3100: 
                   3101: Автор:
                   3102: Сергей Спешков (Москва)
                   3103: 
                   3104: Тур:
                   3105: Этап 5
                   3106: 
                   3107: Дата:
                   3108: 08-Feb-2015
                   3109: 
                   3110: Редактор:
                   3111: Михаил Локшин и Дмитрий Петров (Санкт-Петербург)
                   3112: 
                   3113: Инфо:
                   3114: Редакторы благодарят за тестирование пакета и ценные замечения команды
                   3115: "Ноев ковчег" (Санкт-Петербург), "40 звездочек" (Санкт-Петербург),
                   3116: "Суббота 13" (Нью-Йорк), а также Льва Орлова, Ольгу Кузьму, Антона
                   3117: Чухнова, Кирилла и Маргариту Савицких, Александра Элеазера, Лидию
                   3118: Иванову, Константина Кнопа, Евгения Поникарова (все - Санкт-Петербург),
                   3119: Кирилла Чистякова, Максима Дрозда, Михаила Максимова, Вячеслава
                   3120: Белькова, Александра Иванова (все - Торонто), Марию Гендину (Чикаго),
                   3121: Мурода Хамроева (Монреаль), Якова Зайдельмана (Переславль-Залесский),
                   3122: Дмитрия Когана (Фридрихсхафен), Антона Тахтарова (Самара), Евгения
                   3123: Миротина (Минск), Ивана Ефремова (Ростов-на-Дону), Сергея Лобачёва
                   3124: (Нижний Новгород), Бориса Белозёрова (Москва), продолжение следует.
                   3125: 
                   3126: Вопрос 1:
                   3127: [Ведущему: Перед чтением этого вопроса огласите список благодарностей
                   3128: так, как он приведен в преамбуле к пакету, если вы еще не сделали этого.
                   3129: После сдачи ответов до или после оглашения правильного ответа прочитайте
                   3130: полностью комментарий к этому вопросу.]
                   3131:    В ЮАР полицейские насчитали в спальне одной женщины более десяти
1.3       rubashki 3132: краденых ИХ. Назовите ИХ одним словом.
1.1       rubashki 3133: 
                   3134: Ответ:
                   3135: Овцы.
                   3136: 
                   3137: Зачет:
                   3138: Овечки.
                   3139: 
                   3140: Комментарий:
                   3141: Кто-то при бессоннице читает список кораблей, а кто-то - просто считает
                   3142: овец. Мы список тестеров прочли примерно до середины. Надеемся, он вас
                   3143: не слишком усыпил.
                   3144:    Редакторы также благодарят за тестирование пакета и ценные замечания:
                   3145: клуб знатоков города Винница, Виктора Байрака (Винница - Воронеж), Илону
                   3146: Косенко (Одесса), Екатерину Дубровскую (Омск), Андрея Гатаулина
                   3147: (Караганда), Дмитрия Овчарука (Киев), Александра Толесникова
                   3148: (Тель-Авив), Андрея Погорелова (Тимашевск), Дмитрия Борока (Самара),
                   3149: Александра Коробейникова, Бориса Гуревича (оба - Саратов), Дмитрия
                   3150: Соловьева (Нижний Новгород), Фарита Алиби (Астана), Олега Борцова
                   3151: (Чебоксары), Артема Корсуна (Львов), Анну Овчинникову (Хайфа), Юлию
                   3152: Устюжанину (Тель-Авив), Вадима Ефимова (Хайфа), Игоря Колмакова
                   3153: (Тель-Авив).
                   3154: 
                   3155: Источник:
                   3156: http://www.dailymail.co.uk/news/article-2281768/Cant-pull-wool-eyes-Policeman-finds-men-TEN-sheep-car-animals-allegedly-stolen-farm.html
                   3157: 
                   3158: Автор:
                   3159: Михаил Локшин, Дмитрий Петров (Санкт-Петербург)
                   3160: 
                   3161: Вопрос 2:
                   3162: Самый дорогой в истории ОН выпрыгнул с ликом Богоматери и впоследствии
                   3163: был продан за 28 тысяч долларов. Так пусть и вас ждут приятные сюрпризы!
                   3164: Назовите ЕГО односложным словом.
                   3165: 
                   3166: Ответ:
                   3167: Тост.
                   3168: 
                   3169: Комментарий:
                   3170: На тосте в результате прожарки появился лик Девы Марии.
                   3171: 
                   3172: Источник:
                   3173: Роберт Хьюз. Рим. История города: его культура, облик, люди. - М.: АСТ:
                   3174: CORPUS, 2014. - С. 204.
                   3175: 
                   3176: Автор:
                   3177: Дмитрий Петров (Санкт-Петербург)
                   3178: 
                   3179: Вопрос 3:
                   3180: Андрей ЩербенОк, говоря о культуре времен застоя, отмечает, что даже
                   3181: солнечный летний день тогда выглядел тусклым и пыльным. При этом он
                   3182: упоминает предприятие, находившееся в небольшом украинском городе.
                   3183: Назовите это предприятие.
                   3184: 
                   3185: Ответ:
                   3186: Свема.
                   3187: 
                   3188: Зачет:
                   3189: По слову "Свема" или по словам "Светочувствительные материалы".
                   3190: 
                   3191: Комментарий:
                   3192: Речь идет о цветном кино, снимавшемся на пленку шосткинского
                   3193: производственного объединения "Свема" (о чем можно знать из финальных
                   3194: кадров многих советских фильмов). Но если вы подумали о фотопленке,
                   3195: ничего страшного.
                   3196: 
                   3197: Источник:
                   3198:    1. http://magazines.russ.ru/nlo/2013/123/8sh.html
                   3199:    2. http://ru.wikipedia.org/wiki/Свема
                   3200: 
                   3201: Автор:
                   3202: Дмитрий Петров (Санкт-Петербург)
                   3203: 
                   3204: Вопрос 4:
                   3205: Описывая здание заводоуправления, Алексей Иванов замечает, что у входа в
                   3206: гуще акаций заблудился ОН. В другом произведении ОН осветил путь.
                   3207: Назовите ЕГО.
                   3208: 
                   3209: Ответ:
                   3210: Владимир Ильич Ленин.
                   3211: 
                   3212: Зачет:
                   3213: По фамилии Ленин или Ульянов. Незачет: Памятник Ленину.
                   3214: 
                   3215: Комментарий:
                   3216: В гимне СССР были слова "И Ленин великий нам путь озарил". Иванов имел в
                   3217: виду памятник вождю.
                   3218: 
                   3219: Источник:
                   3220:    1. А. Иванов. Географ глобус пропил.
                   3221: http://lib.ru/RUFANT/IWANOW_A/geograf_globus_propil.txt
                   3222:    2. http://ru.wikipedia.org/wiki/Гимн_СССР
                   3223: 
                   3224: Автор:
                   3225: Михаил Локшин (Санкт-Петербург)
                   3226: 
                   3227: Вопрос 5:
                   3228: Эмиру Кустурице пришлось сделать 45 дублей эпизода с ЕГО участием.
                   3229: Режиссер заметил: чтобы снять фильм, важнее быть не талантливым, а
                   3230: таким, как ОН. В работах Леонида Гайдая и Эндрю Адамсона ОН тоже есть.
                   3231: Назовите ЕГО одним словом.
                   3232: 
                   3233: Ответ:
                   3234: Осел.
                   3235: 
                   3236: Зачет:
                   3237: Ишак, ослик (с отвращением).
                   3238: 
                   3239: Комментарий:
                   3240: Кустурица в шутку отметил, что важнее быть упрямым, чем талантливым.
                   3241: Ослик снимался в "Кавказской пленнице". А в "Шреке" Осел - вообще один
                   3242: из главных персонажей.
                   3243: 
                   3244: Источник:
                   3245:    1. http://ria.ru/radio_brief/20140806/1019017787.html
                   3246:    2. http://ru.wikipedia.org/wiki/Кавказская_пленница,_или_Новые_приключения_Шурика
                   3247:    3. http://ru.wikipedia.org/wiki/Шрек_(мультфильм)
                   3248: 
                   3249: Автор:
                   3250: Михаил Локшин (Санкт-Петербург)
                   3251: 
                   3252: Вопрос 6:
                   3253: Подраздел теории графов, изучающий случайные графы, используется в
                   3254: социальных науках. У встречающихся на практике случайных графов
                   3255: расстояние между узлами обычно невелико. Назовите ученого,
                   3256: разработавшего совместно с Альфредом РЕньи теорию случайных графов.
                   3257: 
                   3258: Ответ:
                   3259: [Пол] Эрдёш.
                   3260: 
                   3261: Комментарий:
                   3262: "Расстояние" от ученого до Пола Эрдёша через цепочку совместных
                   3263: публикаций - частный случай расстояния на графе. Свойства этого
                   3264: расстояния применительно к социальным наукам описываются "теорией
                   3265: тесного мира", она же "теория шести рукопожатий".
                   3266: 
                   3267: Источник:
                   3268: http://lenta.ru/articles/2014/02/06/memez/
                   3269: 
                   3270: Автор:
                   3271: Дмитрий Петров (Санкт-Петербург)
                   3272: 
                   3273: Вопрос 7:
                   3274: Живущий в приморском городе герой произведения Маркеса каждый вечер
                   3275: неоднократно на полминуты превращается в полосатого тигра. Причина
                   3276: метаморфоз - ОН. Если известный список построить по хронологическому
                   3277: принципу, другой ОН окажется в конце. Назовите ЕГО одним словом.
                   3278: 
                   3279: Ответ:
                   3280: Маяк.
                   3281: 
                   3282: Комментарий:
                   3283: Свет маяка проникал сквозь жалюзи. Маркес не уточняет, включался ли
                   3284: светильник на 30 секунд с интервалами или вращался с заданной
                   3285: периодичностью. Александрийский маяк - одно из самых поздних по времени
                   3286: создания из семи чудес света (между 280 и 247 гг. до н.э.).
                   3287: 
                   3288: Источник:
                   3289:    1. Г. Гарсиа Маркес. Осень патриарха.
                   3290: http://lib.rin.ru/doc/i/134694p9.html
                   3291:    2. http://en.wikipedia.org/wiki/Seven_Wonders_of_the_Ancient_World
                   3292:    3. http://en.wikipedia.org/wiki/Lighthouse_of_Alexandria
                   3293: 
                   3294: Автор:
                   3295: Михаил Локшин (Санкт-Петербург)
                   3296: 
                   3297: Вопрос 8:
                   3298: Футуристы известны своими нападками на классиков девятнадцатого века.
                   3299: Согласно пародийной "Истории советской фантастики", Осип Брик предлагал
                   3300: фантастам и своим литературным спутникам объединить усилия в борьбе с
                   3301: искусством прежних лет. Заголовок его статьи об этом на слух мало
                   3302: отличается от названия, под которым в русском переводе не раз выходило
                   3303: произведение 1865 года. Воспроизведите любое из этих названий.
                   3304: 
                   3305: Ответ:
                   3306: "Из Пушкина - Луну!".
                   3307: 
                   3308: Зачет:
                   3309: "Из пушки на Луну".
                   3310: 
                   3311: Комментарий:
                   3312: Осип Брик призывал, в том числе, низвергнуть "Солнце русской поэзии".
                   3313: 
                   3314: Источник:
                   3315:    1. http://www.rulit.net/books/istoriya-sovetskoj-fantastiki-read?151516?5.html
                   3316:    2. http://en.wikipedia.org/wiki/From_the_Earth_to_the_Moon
                   3317:    3. http://www.litmir.net/bd/?b=171307
                   3318: 
                   3319: Автор:
                   3320: Дмитрий Петров (Санкт-Петербург)
                   3321: 
                   3322: Вопрос 9:
                   3323: Польский исследователь Людвиг Стомма не верит, что сейчас могут быть
                   3324: какие-то аргументы в ЕГО пользу, кроме одного: ОН дает меньше
                   3325: возможностей для шантажа в случае преследований. Назовите ЕГО словом
                   3326: латинского происхождения.
                   3327: 
                   3328: Ответ:
                   3329: Целибат.
                   3330: 
                   3331: Комментарий:
                   3332: В странах, где религия тесно связана с политикой, тема преследований за
                   3333: веру, к сожалению, остается актуальной. Давшего обет безбрачия
                   3334: священника невозможно шантажировать женой и детьми.
                   3335: 
                   3336: Источник:
                   3337:    1. Людвиг Стомма. Недооцененные события истории. Книга исторических
                   3338: заблуждений. - М.: АСТ, 2014. - С. 9.
                   3339:    2. http://ru.wikipedia.org/wiki/Целибат
                   3340: 
                   3341: Автор:
                   3342: Дмитрий Петров (Санкт-Петербург)
                   3343: 
                   3344: Вопрос 10:
                   3345: Мэтт Грейнинг говорит, что в его коллекции пиратской продукции очень
                   3346: много АЛЬФ с Гомером Симпсоном, возможно, указывая на страну-источник
                   3347: контрафакта. Скорее всего, в этой коллекции можно встретить и других
                   3348: персонажей сериала "Симпсоны". Какое слово мы заменили на АЛЬФУ?
                   3349: 
                   3350: Ответ:
                   3351: Матрешка.
                   3352: 
                   3353: Комментарий:
                   3354: Семейство Гомера Симпсона - Гомер, Мардж, Лиза, Барт и Мэгги - хорошо
                   3355: подходит для изображения на наборе матрешек.
                   3356: 
                   3357: Источник:
                   3358:    1. http://esquire.ru/wil/matt-groening
                   3359:    2. http://www.artguide.com/posts/319-kartinki-niedieli-12-03-19-03-349
                   3360:    3. http://azbukivedi.livejournal.com/292426.html
                   3361:    4. http://joyreactor.cc/post/1695
                   3362:    5. http://ru.wikipedia.org/wiki/Грейнинг,_Мэтт
                   3363: 
                   3364: Автор:
                   3365: Дмитрий Петров (Санкт-Петербург)
                   3366: 
                   3367: Вопрос 11:
                   3368: Томас Джефферсон и Джон Адамс были политическими противниками.
                   3369: Англоязычная Википедия упоминает интересную особенность картины Джона
                   3370: ТрАмбулла "Декларация независимости": многим кажется, что в нижней части
                   3371: полотна Джефферсон [ПРОПУСК] Адамсу. Заполните пропуск тремя словами,
                   3372: начинающимися на одну и ту же букву.
                   3373: 
                   3374: Ответ:
                   3375: "... наступает на ногу...".
                   3376: 
                   3377: Зачет:
                   3378: "... наступил на ногу...".
                   3379: 
                   3380: Источник:
                   3381: http://en.wikipedia.org/wiki/Declaration_of_Independence_(Trumbull)
                   3382: 
                   3383: Автор:
                   3384: Дмитрий Петров (Санкт-Петербург)
                   3385: 
                   3386: Вопрос 12:
                   3387: Один медицинский сайт утверждает, что сам ОН безопасен, но с ЕГО помощью
                   3388: распространяется пыльца других растений. ЕМУ посвящено произведение 1998
                   3389: года. Назовите ЕГО двумя словами.
                   3390: 
                   3391: Ответ:
                   3392: Тополиный пух.
                   3393: 
                   3394: Зачет:
                   3395: Пух тополя/тополей.
                   3396: 
                   3397: Комментарий:
                   3398: Опровергая популярное мнение, сайт "Доктор Питер" пишет, что пух - это
                   3399: семена, а не пыльца. Разнося пыльцу других растений, он вызывает
                   3400: обострения аллергии. "Тополиный пух" - песня группы "Иванушки
                   3401: International", вышедшая в 1998 году.
                   3402: 
                   3403: Источник:
                   3404:    1. http://www.doctorpiter.ru/articles/9069/
                   3405:    2. http://www.megalyrics.ru/lyric/ivanushki-int/topolinyi-pukh.htm
                   3406:    3. http://ru.wikipedia.org/wiki/Иванушки_International
                   3407: 
                   3408: Автор:
                   3409: Михаил Локшин (Санкт-Петербург)
                   3410: 
                   3411: Вопрос 13:
                   3412: Подопечные Гуса Хиддинка неоднократно вырывали победу за секунды до
                   3413: финального свистка. При жеребьевке им редко доставались сильные
                   3414: соперники. Напишите прозвище Хиддинка, двумя буквами отличающееся от
                   3415: названия известного музыкального произведения.
                   3416: 
                   3417: Ответ:
                   3418: Везучий голландец.
                   3419: 
                   3420: Комментарий:
                   3421: У Хиддинка репутация не просто хорошего, но и удачливого тренера.
                   3422: 
                   3423: Источник:
                   3424:    1. http://ru.wikipedia.org/wiki/Хиддинк,_Гус
                   3425:    2. http://www.eurosport.ru/football/story_sto945410.shtml
                   3426:    3. http://ng.sb.by/soyuznoe-veche/article/s-khiddinkom-rossii-stalo-vezti.html
                   3427: 
                   3428: Автор:
                   3429: Михаил Локшин (Санкт-Петербург)
                   3430: 
                   3431: Вопрос 14:
                   3432: Один российский тренер сказал о больной спортсменке: "Самочувствие ее
                   3433: стало хуже. Знаю, что решается вопрос о лечении серьезными
                   3434: медикаментами, но это уже [ПРОПУСК]". Заполните пропуск словосочетанием
                   3435: из двух слов, печально известным с середины прошлого века.
                   3436: 
                   3437: Ответ:
                   3438: "... дело врачей".
                   3439: 
                   3440: Комментарий:
                   3441: Дело врачей - уголовное дело против группы видных советских
                   3442: врачей-евреев, которых обвиняли в заговоре и убийстве советских лидеров.
                   3443: Истоки кампании относятся к 1948 году, окончание - к 1953 году. Медиков
                   3444: называли "врачами-вредителями". Тренер имел в виду не это, но получилась
                   3445: двусмысленность.
                   3446: 
                   3447: Источник:
                   3448:    1. http://www.itar-tass.com/sport/978633
                   3449:    2. http://ru.wikipedia.org/wiki/Дело_врачей
                   3450: 
                   3451: Автор:
                   3452: Михаил Локшин (Санкт-Петербург)
                   3453: 
                   3454: Вопрос 15:
                   3455: [Ведущему: не сообщать о том, что слово "Лицо" написано с заглавной
                   3456: буквы.]
                   3457:    Пушкин, заметив сидевшего между Булгариным и Гречем лицеиста
                   3458: Семенова, вспомнил аналогичную ситуацию и упомянул известное Лицо и
                   3459: топоним. Какой именно топоним?
                   3460: 
                   3461: Ответ:
                   3462: Голгофа.
                   3463: 
                   3464: Комментарий:
                   3465: Неприязненные отношения Пушкина и Булгарина общеизвестны. Обращаясь к
                   3466: Семенову, поэт язвительно поддел его соседей: "Ты, брат Семенов, словно
                   3467: Христос на горе Голгофе!". Так он намекал на то, что Спаситель был
                   3468: распят на кресте между двумя разбойниками. Иисус Христос - Второе Лицо
                   3469: Святой Троицы.
                   3470: 
                   3471: Источник:
                   3472:    1. http://www.spbvedomosti.ru/print.htm?id=10298989@SV_Articles
                   3473:    2. http://ru.wikipedia.org/wiki/Троица
                   3474: 
                   3475: Автор:
                   3476: Михаил Локшин (Санкт-Петербург)
                   3477: 
                   3478: Вопрос 16:
                   3479: Внимание, в вопросе есть замены.
                   3480:    В Ветхом завете говорится о голодных годах, в которые - цитата - "ни
                   3481: ПИТЬ, ни ЗАКУСЫВАТЬ не будут". При игре в брэйн-ринг необходимо
                   3482: ЗАКУСЫВАТЬ, а в некоторых случаях эмоциональные знатоки могут ПИТЬ.
                   3483: Какие глаголы мы заменили на "ПИТЬ" и "ЗАКУСЫВАТЬ"?
                   3484: 
                   3485: Ответ:
                   3486: Орать, жать.
                   3487: 
                   3488: Зачет:
                   3489: В любом порядке.
                   3490: 
                   3491: Комментарий:
                   3492: В первом случае "орать" - пахать, "жать" - пожинать. Во втором "орать" -
                   3493: кричать, "жать" - нажимать на кнопку.
                   3494: 
                   3495: Источник:
                   3496:    1. http://days.pravoslavie.ru/Bible/B_byt45.htm
                   3497:    2. http://dic.academic.ru/dic.nsf/enc2p/291570/
                   3498:    3. ЛОАВ.
                   3499: 
                   3500: Автор:
                   3501: Михаил Локшин (Санкт-Петербург)
                   3502: 
                   3503: Вопрос 17:
                   3504: (pic: 20140521.jpg)
                   3505:    Умер осенью. Жил в семнадцатом веке. Как его зовут?
                   3506: 
                   3507: Ответ:
                   3508: Мацуо Басё.
                   3509: 
                   3510: Зачет:
                   3511: По слову Басё; по имени Мацуо, Мунэфуса или Дзинситиро.
                   3512: 
                   3513: Комментарий:
                   3514:    Весь этот вопрос
                   3515:    Напоминает хокку.
                   3516:    Судите сами:
                   3517:    Умер осенью.
                   3518:    Жил в семнадцатом веке.
                   3519:    Как его зовут?
                   3520:    В "правильном" хокку
                   3521:    Нужно указание
                   3522:    На время года.
                   3523: 
                   3524: Источник:
                   3525:    1. http://ru.wikipedia.org/wiki/Мацуо_Басё
                   3526:    2. http://www.world-art.ru/people.php?id=10799
                   3527:    3. http://ru.wikipedia.org/wiki/Хайку
                   3528: 
                   3529: Автор:
                   3530: Михаил Локшин (Санкт-Петербург)
                   3531: 
                   3532: Вопрос 18:
                   3533: [Ведущему: кавычки не озвучивать.]
                   3534:    Для взятия вопроса не обязательно быть поклонником ЭТОГО.
                   3535:    Действие ЭТОГО, вышедшего в 1998 году, происходит в будущем. Люди
                   3536: колонизировали Солнечную систему, в том числе - небесное тело, в котором
                   3537: можно найти "ЭТО". Назовите небесное тело.
                   3538: 
                   3539: Ответ:
                   3540: Ганимед.
                   3541: 
                   3542: Комментарий:
                   3543: В слове "Ганимед" можно найти слово "аниме". В фантастическом
                   3544: аниме-сериале "Cowboy Bebop" Ганимед и некоторые другие спутники Юпитера
                   3545: колонизированы.
                   3546: 
                   3547: Источник:
                   3548:    1. http://www.imdb.com/title/tt0213338/
                   3549:    2. http://en.wikipedia.org/wiki/Cowboy_Bebop
                   3550: 
                   3551: Автор:
                   3552: Дмитрий Петров (Санкт-Петербург)
                   3553: 
                   3554: Вопрос 19:
                   3555: Сергей ДорогАвцев заметил, что торги на бирже в некий момент проходили
                   3556: под знаком ЕГО: на фоне новостей из США дешевела нефть, снижались акции.
                   3557: Назовите ЕГО несклоняемым словом.
                   3558: 
                   3559: Ответ:
                   3560: Гризли.
                   3561: 
                   3562: Комментарий:
                   3563: Трейдеров, которые ожидают понижения цен, называют медведями. Медведи
                   3564: придавливают соперника лапами вниз. Гризли - типичный для Америки
                   3565: медведь.
                   3566: 
                   3567: Источник:
                   3568:    1. http://www.finam.ru/analysis/marketnews3DDE5/default.asp
                   3569:    2. http://ru.wikipedia.org/wiki/Гризли
                   3570:    3. http://ru.wikipedia.org/wiki/Трейдер
                   3571: 
                   3572: Автор:
                   3573: Михаил Локшин (Санкт-Петербург)
                   3574: 
                   3575: Вопрос 20:
                   3576: [Ведущему: не пропустить преамбулу к дуплету!]
                   3577:    Дуплет. Два вопроса по 30 секунд обсуждения каждый, ответы сдаются на
                   3578: одном бланке. Чтобы получить очко, вам нужно правильно ответить на оба
                   3579: вопроса.
                   3580:    1. Скульптура ЕГО на крыше в одном украинском городе кому-то
                   3581: показалась неуместной. Назовите ЕГО.
                   3582:    2. По сообщению журнала "Популярная механика", одной австралийской
                   3583: лошади ЕГО отчасти заменил 3D-принтер. Назовите ЕГО.
                   3584: 
                   3585: Ответ:
                   3586:    1. Скрипач.
                   3587:    2. Кузнец.
                   3588: 
                   3589: Комментарий:
                   3590: Как известно из фильмов "Кин-дза-дза!" и "Формула любви", "скрипач не
                   3591: нужен" и "нам кузнец не нужен". Австралийская скаковая лошадь получила
                   3592: титановые подковы, индивидуально изготовленные на 3D-принтере на основе
                   3593: отсканированных моделей копыт.
                   3594: 
                   3595: Источник:
                   3596:    1. http://ru.wikipedia.org/wiki/Скрипач_на_крыше_(скульптура)
                   3597:    2. http://www.popmech.ru/article/14055-a-zachem-nam-kuznets/
                   3598:    3. http://ru.wikiquote.org/wiki/Кин-дза-дза!
                   3599:    4. http://ru.wikiquote.org/wiki/Формула_любви
                   3600: 
                   3601: Автор:
                   3602: Дмитрий Петров (Санкт-Петербург)
                   3603: 
                   3604: Вопрос 21:
                   3605: [Ведущему: отточия в цитате игнорировать.]
                   3606:    Внимание, в вопросе слова "ОДИН" и "ДРУГОЙ" - замены.
                   3607:    Цитата из интервью: "Мне всегда был симпатичен ОДИН. ДРУГОЙ...
                   3608: замкнутый, слишком сосредоточенный на своей миссии. У него короткая
                   3609: борода... ОДИН - ... жизнелюб, который запросто может выпить с друзьями,
                   3610: любит веселье, хорошие шутки" (конец цитаты). Так британский актер
                   3611: высказался о своей роли. Ответьте в правильном порядке: кто такой ОДИН и
                   3612: кто - ДРУГОЙ?
                   3613: 
                   3614: Ответ:
                   3615: Гэндальф Серый, Гэндальф Белый.
                   3616: 
                   3617: Зачет:
                   3618: В правильном порядке.
                   3619: 
                   3620: Комментарий:
                   3621: ОДИН и ДРУГОЙ - это Гэндальф Серый и Гэндальф Белый соответственно
                   3622: ("Один серый, другой - белый"; кстати, Толкин представлял Гэндальфа
                   3623: похожим на Одина). Интервью дал Иэн Маккеллен, сыгравший Гэндальфа в
                   3624: трилогиях "Властелин Колец" и "Хоббит".
                   3625: 
                   3626: Источник:
                   3627:    1. "Панорама ТВ", 2013, N 5. - С. 84.
                   3628:    2. http://ru.wikipedia.org/wiki/Гэндальф
                   3629: 
                   3630: Автор:
                   3631: Михаил Локшин (Санкт-Петербург)
                   3632: 
                   3633: Вопрос 22:
                   3634: На гобелене из Байё можно увидеть пронзенного стрелой английского короля
                   3635: ГАрольда. Эндрю УИттакер сравнил это изображение с кадрами кинопленки,
                   3636: снятой оператором-любителем... В каком городе?
                   3637: 
                   3638: Ответ:
                   3639: В Далласе.
                   3640: 
                   3641: Комментарий:
                   3642: Абрахам ЗапрУдер запечатлел убийство Джона Кеннеди. Гобелен из Байё -
                   3643: длинный, узкий и по композиции напоминает кинопленку.
                   3644: 
                   3645: Источник:
                   3646: Э. Уиттакер. Великобритания. - М.: РИПОЛ классик, 2013. - С. 29.
                   3647: 
                   3648: Автор:
                   3649: Дмитрий Петров (Санкт-Петербург)
                   3650: 
                   3651: Вопрос 23:
                   3652: Фрэнк МаклИнн назвал вождя чЕроки, настаивавшего на переговорах с
                   3653: губернатором Южной Каролины, единственным ПЕРВЫМ в племени ВТОРЫХ.
                   3654: Назовите ПЕРВЫХ и ВТОРЫХ.
                   3655: 
                   3656: Ответ:
                   3657: Голуби, ястребы.
                   3658: 
                   3659: Комментарий:
                   3660: Оппоненты вождя хотели войны. Применение к миролюбивым и воинственным
                   3661: индейцам терминов "голуби" и "ястребы" выглядит довольно забавно.
                   3662: 
                   3663: Источник:
                   3664: Ф. Маклинн. 1759. Год завоевания Британией мирового господства. - М.:
                   3665: АСТ: АСТ МОСКВА, 2009. - С. 33.
                   3666: 
                   3667: Автор:
                   3668: Дмитрий Петров (Санкт-Петербург)
                   3669: 
                   3670: Вопрос 24:
                   3671: Линейные крейсера класса "Конго" сравнивали с НЕЙ за сужающиеся к верху
                   3672: высокие надстройки. ОНА, установленная около двух месяцев назад в
                   3673: приграничной зоне на территории Южной Кореи, расценивается Севером как
                   3674: орудие психологической войны. Назовите ЕЕ двумя словами.
                   3675: 
                   3676: Ответ:
                   3677: Рождественская елка.
                   3678: 
                   3679: Зачет:
                   3680: Новогодняя елка; рождественская ель; новогодняя ель.
                   3681: 
                   3682: Комментарий:
                   3683: Высокие надстройки со множеством прожекторов ночью светились, как
                   3684: рождественская ель. Светящаяся рождественская елка в южнокорейской
                   3685: приграничной зоне воспринимается в КНДР как средство психологической
                   3686: войны: у нас тут праздник и веселье, а у вас - нет.
                   3687: 
                   3688: Источник:
                   3689:    1. http://en.wikipedia.org/wiki/Pagoda_mast
                   3690:    2. http://english.yonhapnews.co.kr/national/2014/12/02/94/0301000000AEN20141202003800315F.html
                   3691: 
                   3692: Автор:
                   3693: Дмитрий Петров (Санкт-Петербург)
                   3694: 
                   3695: Вопрос 25:
                   3696: Короткое знаменитое высказывание ОН связал с поэзией. Ему вспомнилась
                   3697: строка Маяковского о незнаемом. Действительно, что-то подобное
                   3698: предстояло тогда и ему самому. Разъяснения какого известного человека мы
                   3699: вам пересказали?
                   3700: 
                   3701: Ответ:
                   3702: [Юрия Алексеевича] Гагарина.
                   3703: 
                   3704: Комментарий:
                   3705: "Я думаю, это относилось к термину "езда в незнаемое". Ведь так говорил
                   3706: Владимир Маяковский о поэзии. А что может быть поэтичнее космоса?". Так
                   3707: Гагарин объяснил свое знаменитое "Поехали!". Первые буквы предложений
                   3708: вопроса составляют слово "кедр" - позывной Гагарина.
                   3709: 
                   3710: Источник:
                   3711:    1. http://www.piter.fm/share/7873/20140311/fromtime:08:48:22
                   3712:    2. http://ru.wikipedia.org/wiki/Гагарин,_Юрий_Алексеевич
                   3713:    3. http://az.lib.ru/m/majakowskij_w_w/text_0890.shtml
                   3714: 
                   3715: Автор:
                   3716: Михаил Локшин (Санкт-Петербург)
                   3717: 
                   3718: Вопрос 26:
                   3719: Один изобретатель придумал чудо-машину, помогающую СДЕЛАТЬ ЭТО за три
                   3720: дня. Чтобы СДЕЛАТЬ ЭТО в три этапа, пригодится название денежных единиц
                   3721: двух азиатских государств. Широко известен случай, когда ЭТО БЫЛО
                   3722: СДЕЛАНО... Ответьте точно: где именно?
                   3723: 
                   3724: Ответ:
                   3725: В Кане [Галилейской].
                   3726: 
                   3727: Зачет:
                   3728: По упоминанию Каны ("на брачном пире в Кане" и т.п.).
                   3729: 
                   3730: Комментарий:
                   3731: СДЕЛАТЬ ЭТО - превратить воду в вино. По словам изобретателя Филиппа
                   3732: Джеймса, машина использует водопроводную воду и сокращает процесс
                   3733: виноделия до трех дней. Слово "вода" превращается в слово "вино" в три
                   3734: этапа: вода - вона - вина - вино. И в Республике Корея, и в КНДР
                   3735: используются воны. В Кане Галилейской Иисус Христос совершил чудо -
                   3736: превратил воду в вино.
                   3737: 
                   3738: Источник:
                   3739:    1. http://www.gazeta.ru/tech/news/2014/03/07/n_5997049.shtml
                   3740:    2. http://www.iqfun.ru/articles/muha-slon.shtml
                   3741:    3. http://ru.wikipedia.org/wiki/Северокорейская_вона
                   3742:    4. http://ru.wikipedia.org/wiki/Южнокорейская_вона
                   3743:    5. http://ru.wikipedia.org/wiki/Кана_Галилейская
                   3744: 
                   3745: Автор:
                   3746: Михаил Локшин (Санкт-Петербург)
                   3747: 
                   3748: Вопрос 27:
                   3749: Литературовед Лоурэнс Дэнсон пишет, что к середине девятнадцатого века
                   3750: ОН стал более женственным. ОН перестал быть деятельным мстителем, стал
                   3751: более пассивным, задумчивым, сомневающимся. Назовите его имя.
                   3752: 
                   3753: Ответ:
                   3754: Гамлет.
                   3755: 
                   3756: Комментарий:
                   3757: По мнению Дэнсона, в интерпретации современников образ становился всё
                   3758: более "женственным". Его даже стали играть женщины, в том числе - Сара
                   3759: Бернар.
                   3760: 
                   3761: Источник:
                   3762: http://muse.jhu.edu/journals/shq/summary/v059/59.1danson.html
                   3763: 
                   3764: Автор:
                   3765: Дмитрий Петров (Санкт-Петербург)
                   3766: 
                   3767: Вопрос 28:
                   3768: Некоторые считают, что колесно-гусеничный танк А-20 был разработан
                   3769: специально для ИКСОВ. В онлайн-игре "Мир танков" ИКС можно получить,
                   3770: например, за стрельбу по союзникам. Какое слово мы заменили на ИКС?
                   3771: 
                   3772: Ответ:
                   3773: Автобан.
                   3774: 
                   3775: Комментарий:
                   3776: Некоторые вслед за Виктором Суворовым считают, что танк разработан
                   3777: специально для немецких скоростных трасс и буква "A" значит
                   3778: "автострадный". Автобан - это еще и автоматический бан пользователя.
                   3779: 
                   3780: Источник:
                   3781:    1. http://lurkmore.to/Автострадный_танк
                   3782:    2. http://ru.wikipedia.org/wiki/А-20_(танк)
                   3783:    3. http://forum.worldoftanks.ru/index.php?/topic/97748-автобан
                   3784: 
                   3785: Автор:
                   3786: Дмитрий Петров (Санкт-Петербург)
                   3787: 
                   3788: Вопрос 29:
                   3789: Дуплет. Два вопроса по 30 секунд обсуждения каждый, ответы сдаются на
                   3790: одном бланке.
                   3791:    1. Угандиец Джон АкИи-БуА удостоился особых почестей, но позже был
                   3792: вынужден бежать из страны. Впоследствии АкИи-БуА был принят на работу в
                   3793: известную компанию. Какую?
                   3794:    2. В некоторых речевых информаторах серии РИ [эр-и] сообщения
                   3795: передавались женским голосом. За это система получила от летчиков
                   3796: уменьшительное прозвище. Какое?
                   3797: 
                   3798: Ответ:
                   3799:    1. "Puma".
                   3800:    2. "Рита".
                   3801: 
                   3802: Комментарий:
                   3803: Джон Акии-Буа, олимпийский чемпион в беге с препятствиями, был особо
                   3804: отмечен Иди Амином, но после смены власти ему пришлось покинуть страну.
                   3805: Слово "пума", записанное латинскими буквами, можно при желании прочитать
                   3806: как "Рита". "Рита" - это уменьшительная форма имени "Маргарита".
                   3807: 
                   3808: Источник:
                   3809:    1. http://en.wikipedia.org/wiki/John_Akii-Bua
                   3810:    2. http://ru.wikipedia.org/wiki/Речевой_информатор
                   3811: 
                   3812: Автор:
                   3813: Дмитрий Петров (Санкт-Петербург)
                   3814: 
                   3815: Вопрос 30:
                   3816: [Ведущему: кавычки не озвучивать.]
                   3817:    Отца "отца" отца звали Эдвард. "Отец" отца в 1922 году принял
                   3818: католичество. Назовите фамилию Эдварда.
                   3819: 
                   3820: Ответ:
                   3821: Честертон.
                   3822: 
                   3823: Комментарий:
                   3824: Литературный отец католического священника отца Брауна Гилберт Кит
                   3825: Честертон тоже стал католиком.
                   3826: 
                   3827: Источник:
                   3828: http://www.catholicauthors.com/chesterton.html
                   3829: 
                   3830: Автор:
                   3831: Михаил Локшин (Санкт-Петербург)
                   3832: 
                   3833: Вопрос 31:
                   3834: В стихотворении Владимира САлимона церквушка на холме, уцелевшая при
                   3835: пожаре, кажется, может потерять равновесие и упасть от малейшего
                   3836: дуновения ветра. Поэт сравнивает ее с героиней произведения 1905 года.
                   3837: Какого?
                   3838: 
                   3839: Ответ:
                   3840: "Девочка на шаре".
                   3841: 
                   3842: Комментарий:
                   3843:    Совсем как девочка на шаре,
                   3844:    стоит церквушка на холме,
                   3845:    что уцелела при пожаре
                   3846:    Москвы...
                   3847: 
                   3848: Источник:
                   3849:    1. http://magazines.russ.ru/arion/2014/2/1s.html
                   3850:    2. http://ru.wikipedia.org/wiki/Девочка_на_шаре
                   3851: 
                   3852: Автор:
                   3853: Дмитрий Петров (Санкт-Петербург)
                   3854: 
                   3855: Вопрос 32:
                   3856: После 2011 года оставшиеся ОНИ попали в музеи. Один американец в связи с
                   3857: этим с грустью вспоминает запоздалый дребезг оконных стекол. Назовите ИХ
                   3858: словом с удвоенной согласной.
                   3859: 
                   3860: Ответ:
                   3861: Шаттлы.
                   3862: 
                   3863: Комментарий:
                   3864: В 2011 году программа полетов шаттлов была свернута. Корабли "Атлантис",
                   3865: "Дискавери", "Индэвор" и "Колумбия" были переданы различным музеям США.
                   3866: Американец с ностальгией вспоминает, как когда-то видел пролетающий на
                   3867: сверхзвуковой скорости шаттл. Больше над его городом шаттлы не летают.
                   3868: 
                   3869: Источник:
                   3870:    1. http://ru.wikipedia.org/wiki/Спейс_шаттл
                   3871:    2. http://io9.com/that-was-the-shuttle-for-me-i-remember-one-day-when-t-1626890085
                   3872: 
                   3873: Автор:
                   3874: Дмитрий Петров (Санкт-Петербург)
                   3875: 
                   3876: Вопрос 33:
                   3877: Половина ответа присутствует в вопросе.
                   3878:    По одной из версий, ЕГО название происходит от нижненемецкого "de
                   3879: engil" [дэ Энгиль]: считается, что о его целебных свойствах людям
                   3880: поведал ангел. О НЕМ напоминает фамилия человека, умершего на чужбине.
                   3881: Ответьте одним словом: что принято оставлять на могиле этого человека?
                   3882: 
                   3883: Ответ:
                   3884: Пуанты.
                   3885: 
                   3886: Комментарий:
                   3887: ОН - дягиль. По одной из версий, слово происходит от нижненемецкого "de
                   3888: engil", "ангел". Человек - Сергей Дягилев, на могиле которого в Венеции
                   3889: хореографы и танцовщики по традиции оставляют балетные туфли. В этом
                   3890: вопросе тоже есть пуант (как заключительная часть шутки, загадки или
                   3891: вопроса "Что? Где? Когда?").
                   3892: 
                   3893: Источник:
                   3894:    1. http://en.wikipedia.org/wiki/Angelica_archangelica
                   3895:    2. http://ru.wikipedia.org/wiki/Дягиль_лекарственный
                   3896:    3. http://ru.wikipedia.org/wiki/Дягилев,_Сергей_Павлович
                   3897:    4. http://ru.wikipedia.org/wiki/Пуанты
                   3898:    5. http://dic.academic.ru/dic.nsf/dic_synonims/144775/
                   3899:    6. http://chgk.wikia.com/wiki/Сленг
                   3900: 
                   3901: Автор:
                   3902: Дмитрий Петров (Санкт-Петербург)
                   3903: 
                   3904: Вопрос 34:
                   3905: Авторы называли ЕЕ символом движения человека к Будущему - медленного,
                   3906: изнурительного, но неуклонного. Назовите объект Всемирного наследия
                   3907: ЮНЕСКО, который с НЕЙ связан.
                   3908: 
                   3909: Ответ:
                   3910: Фудзияма.
                   3911: 
                   3912: Зачет:
                   3913: Фудзи, Фудзисан.
                   3914: 
                   3915: Комментарий:
                   3916: Так Стругацкие разъясняют смысл взятого в качестве эпиграфа хокку
                   3917: Кобаяси Иссы "Тихо, тихо ползи, улитка, по склону Фудзи, вверх, до самых
                   3918: высот".
                   3919: 
                   3920: Источник:
                   3921:    1. Б.Н. Стругацкий. Комментарии к пройденному.
                   3922: http://www.litmir.me/br/?b=84548&p=17
                   3923:    2. http://ru.wikipedia.org/wiki/Фудзияма
                   3924: 
                   3925: Автор:
                   3926: Михаил Локшин (Санкт-Петербург)
                   3927: 
                   3928: Вопрос 35:
                   3929: Один из ЕЕ вариантов, появившийся в 1931 году, англоязычная Википедия
                   3930: сравнивает с изображением электрической цепи. На постере Дэвида Бута
                   3931: линии на НЕЙ получаются из тюбиков с красками. Назовите ЕЕ тремя
                   3932: словами.
                   3933: 
                   3934: Ответ:
                   3935: Карта лондонского метрополитена.
                   3936: 
                   3937: Зачет:
                   3938: Карта/схема лондонского метро[политена], карта/схема метро[политена]
                   3939: Лондона.
                   3940: 
                   3941: Комментарий:
                   3942: Известное название лондонского метро - "tube" (т.е. "труба") - созвучно
                   3943: слову "тюбик" (и омонимично английскому слову "tube" с соответствующим
                   3944: значением). До 1931 года карты метро были привязаны к географическим
                   3945: картам. Сотрудник лондонской подземки Гарри Бек сделал первую карту
                   3946: метро в виде топологической диаграммы, что сделало ее более читаемой.
                   3947: 
                   3948: Источник:
                   3949: http://en.wikipedia.org/wiki/Tube_map
                   3950: 
                   3951: Автор:
                   3952: Дмитрий Петров (Санкт-Петербург)
                   3953: 
                   3954: Вопрос 36:
                   3955: Мостранссоюз решил принять ряд мер, чтобы распрощаться с нелегальным
                   3956: частным извозом. Рассказывая об этом, один сайт процитировал припев
                   3957: зарубежной песни, чуть изменив второе слово из трех. Напишите то, что
                   3958: получилось в результате.
                   3959: 
                   3960: Ответ:
                   3961: "Чао, "бомбила", сорри".
                   3962: 
                   3963: Зачет:
                   3964: С произвольной пунктуацией.
                   3965: 
                   3966: Комментарий:
                   3967: Частных таксистов, особенно нелегальных, называют бомбилами. В оригинале
                   3968: - "Чао, бамбино, сорри". Слово "распрощаться" - подсказка.
                   3969: 
                   3970: Источник:
                   3971:    1. http://www.solidarnost.org/thems/obshestvo/obshestvo_8260.html
                   3972:    2. http://megalyrics.ru/lyric/mireille-mathieu/ciao-bambino-sorry.htm
                   3973: 
                   3974: Автор:
                   3975: Михаил Локшин (Санкт-Петербург)
                   3976: 
                   3977: Тур:
                   3978: Этап 6
                   3979: 
                   3980: Дата:
                   3981: 01-Mar-2015
                   3982: 
                   3983: Редактор:
                   3984: Сергей Лобачёв и Ярослав Косарев (Нижний Новгород)
                   3985: 
                   3986: Инфо:
                   3987: Редакторы благодарят за тестирование вопросов и помощь в подготовке
                   3988: пакета: Якова Зайдельмана (Переславль-Залесский), Антона Тахтарова
                   3989: (Самара - Рамат-Ган), Вадима Молдавского (Лондон), Бориса Белозёрова,
                   3990: Александра Карчевского (оба - Москва), Ивана Ефремова (Ростов-на-Дону),
                   3991: Николая Слюняева, Алексея Сергеева, Илью Редькина, Михаила Царёва,
                   3992: Алексея Новолокина, Антона Седова, Егора Господчикова, Марию Лазареву
                   3993: (все - Нижний Новгород).
                   3994: 
                   3995: Вопрос 1:
                   3996: Во время эвакуации жителей многоквартирных домов работникам газовых
                   3997: компаний запрещается пользоваться ИКСАМИ. Один дизайнерский ИКС
                   3998: стилизован под клавишу "Enter". Назовите ИКС.
                   3999: 
                   4000: Ответ:
                   4001: Дверной звонок.
                   4002: 
                   4003: Зачет:
                   4004: Звонок; Входной звонок; Электрический звонок.
                   4005: 
                   4006: Комментарий:
                   4007: Электрический звонок может спровоцировать взрыв газа.
                   4008: 
                   4009: Источник:
                   4010:    1. https://www.nipsco.com/stay-safe/first-responders/preventing-natural-gas-ignition
                   4011:    2. http://www.ubergizmo.com/2008/09/enterbell-concept/
                   4012: 
                   4013: Автор:
                   4014: Сергей Лобачёв (Нижний Новгород)
                   4015: 
                   4016: Вопрос 2:
                   4017: По словам ландшафтного архитектора Пита Ди, настоящий ОН доберется и до
                   4018: вершины Эвереста, если там будет установлен флажок. Назовите ЕГО.
                   4019: 
                   4020: Ответ:
                   4021: Гольфист.
                   4022: 
                   4023: Зачет:
                   4024: Игрок в гольф.
                   4025: 
                   4026: Комментарий:
                   4027: Как известно, лунки для гольфа оборудуются флажками. Ландшафтный
                   4028: дизайнер Питер Ди, упомянутый в вопросе, занимался также и
                   4029: проектированием полей для гольфа.
                   4030: 
                   4031: Источник:
                   4032: http://en.wikipedia.org/wiki/Portal:Golf/Quotes_archive
                   4033: 
                   4034: Автор:
                   4035: Сергей Лобачёв (Нижний Новгород)
                   4036: 
                   4037: Вопрос 3:
                   4038: Внимание, цитата! "Я странник. Я пришел с миром. Отведите меня к своему
                   4039: вождю, и вы будете награждены навеки". Конец цитаты. Специально для
                   4040: Джона перевод этой фразы был подготовлен сразу на несколько языков.
                   4041: Назовите фамилию Джона.
                   4042: 
                   4043: Ответ:
                   4044: Гленн.
                   4045: 
                   4046: Комментарий:
                   4047: Место посадки американского астронавта Джона Гленна не могло быть точно
                   4048: рассчитано.
                   4049: 
                   4050: Источник:
                   4051:    1. http://www.futilitycloset.com/2014/05/06/to-whom-it-may-concern-3/
                   4052:    2. http://ru.wikipedia.org/wiki/Гленн,_Джон
                   4053: 
                   4054: Автор:
                   4055: Сергей Лобачёв (Нижний Новгород)
                   4056: 
                   4057: Вопрос 4:
                   4058: В сериале "Куприн" - экранизации нескольких произведений писателя -
                   4059: неизменным героем всех серий является сам Куприн. Говоря о его роли в
                   4060: сериале, автор одной рецензии сравнивает телезрителей с ПЕРВЫМ, а
                   4061: Куприна со ВТОРЫМ. Кого мы заменили на ПЕРВОГО и ВТОРОГО?
                   4062: 
                   4063: Ответ:
                   4064: Данте, Вергилий.
                   4065: 
                   4066: Комментарий:
                   4067: Куприн словно проводит зрителя по своему творчеству, как Вергилий
                   4068: проводил Данте по чистилищу и аду.
                   4069: 
                   4070: Источник:
                   4071: http://www.kinopoisk.ru/user/497065/comment/2118167/
                   4072: 
                   4073: Автор:
                   4074: Ярослав Косарев (Нижний Новгород)
                   4075: 
                   4076: Вопрос 5:
                   4077: Чарльз Бэббидж СДЕЛАТЬ ЭТОГО не сумел, но отрицательный результат
                   4078: вдохновил его на создание вычислительной машины. Назовите уроженца
                   4079: городка Дрё, который был одним из немногих, сумевших СДЕЛАТЬ ЭТО.
                   4080: 
                   4081: Ответ:
                   4082: [Франсуа-Андре Даникан] Филидор.
                   4083: 
                   4084: Комментарий:
                   4085: Чарльз Бэббидж не сумел победить шахматный автомат "Турок". А вот
                   4086: известному шахматисту того времени Франсуа-Андре Филидору это удалось.
                   4087: 
                   4088: Источник:
                   4089:    1. http://science.slashdot.org/story/02/04/21/1719215/1770-mechanical-chess-player-inspired-babbage
                   4090:    2. http://en.wikipedia.org/wiki/The_Turk
                   4091: 
                   4092: Автор:
                   4093: Сергей Лобачёв (Нижний Новгород)
                   4094: 
                   4095: Вопрос 6:
                   4096: Автогонщик Даниил Квят сказал, что с приходом в "Формулу-1" он в
                   4097: некоторых случаях стал использовать "более быструю" АЛЬФУ. Назовите
                   4098: АЛЬФУ одним словом.
                   4099: 
                   4100: Ответ:
                   4101: Подпись.
                   4102: 
                   4103: Зачет:
                   4104: Роспись.
                   4105: 
                   4106: Комментарий:
                   4107: После перехода в "Формулу-1" Квяту пришлось чаще давать автографы, при
                   4108: этом на оригинальную роспись уходит больше времени.
                   4109: 
                   4110: Источник:
                   4111: http://www.championat.com/auto/article-196537-kvjat---o-toro-rosso-avtografakh-i-futbole.html
                   4112: 
                   4113: Автор:
                   4114: Ярослав Косарев (Нижний Новгород)
                   4115: 
                   4116: Вопрос 7:
                   4117: В своем романе "Война миров" Герберт Уэллс призывает не судить
                   4118: инопланетян строго и напоминает читателю о судьбе ИКСОВ. Назовите
                   4119: соотечественника Уэллса, носившего прозвище ИКС.
                   4120: 
                   4121: Ответ:
                   4122: [Чарльз Лютвидж] Доджсон.
                   4123: 
                   4124: Зачет:
                   4125: [Льюис] Кэрролл.
                   4126: 
                   4127: Комментарий:
                   4128: Писатель призывает вспомнить о судьбе дронтов, полностью истребленных
                   4129: человеком. Широко распространено мнение, что Доджсон заикался и часто
1.4     ! rubashki 4130: произносил свою фамилию как "До-До-Доджсон", за что и получил прозвище
1.1       rubashki 4131: "Додо".
                   4132: 
                   4133: Источник:
                   4134:    1. http://lib.ru/INOFANT/UELS/warworld.txt
                   4135:    2. http://en.wikipedia.org/wiki/Dodo_(nickname)
                   4136: 
                   4137: Автор:
                   4138: Сергей Лобачёв (Нижний Новгород)
                   4139: 
                   4140: Вопрос 8:
                   4141: В своем последнем письме Циолковскому Беляев извиняется за то, что
                   4142: ДЕЛАЕТ ЭТО. Вопреки распространенной версии, ДЕЛАТЬ ЭТО в космосе было
                   4143: опасно. Какие два слова мы заменили словами "ДЕЛАТЬ ЭТО"?
                   4144: 
                   4145: Ответ:
                   4146: Писать карандашом.
                   4147: 
                   4148: Комментарий:
                   4149: Беляев был уже тяжело болен и буквально прикован к постели. Поэтому он
                   4150: не мог пользоваться чернилами и ему приходилось писать карандашом.
                   4151: Вопреки известной городской легенде, советские космонавты не
                   4152: использовали карандаши на борту орбитальных станций, так как плавающие
                   4153: по космическому кораблю частички графита представляют опасность для
                   4154: экипажа и техники.
                   4155: 
                   4156: Источник:
                   4157:    1. http://magazines.russ.ru/neva/2005/4/kudr25.html
                   4158:    2. http://inosmi.ru/world/20121118/202319832.html
                   4159: 
                   4160: Автор:
                   4161: Сергей Лобачёв (Нижний Новгород)
                   4162: 
                   4163: Вопрос 9:
                   4164: Герой одного фантастического рассказа совершает удивительное открытие во
                   4165: время экспедиции по затерянным районам Центральной Америки. Сетуя на
                   4166: отсутствие доказательств, которые он мог бы предъявить научному миру, он
                   4167: упоминает ИКСА. Назовите ИКСА словом с двумя греческими корнями.
                   4168: 
                   4169: Ответ:
                   4170: Птеродактиль.
                   4171: 
                   4172: Комментарий:
                   4173: Рассказ содержит отсылку к роману Конан Дойля "Затерянный мир", героям
                   4174: которого удалось привезти с собой в Лондон птеродактиля.
                   4175: 
                   4176: Источник:
                   4177: Хюберт Лампо. Рождение бога. http://www.flibusta.net/b/216698/read
                   4178: 
                   4179: Автор:
                   4180: Сергей Лобачёв (Нижний Новгород)
                   4181: 
                   4182: Вопрос 10:
                   4183: Рассказывая о Стивене Хокинге, один журнал пишет, что в университетские
                   4184: годы он занимался спортом, однако был... Ответьте одним словом, кем
                   4185: именно.
                   4186: 
                   4187: Ответ:
                   4188: Рулевым.
                   4189: 
                   4190: Комментарий:
                   4191: Стивен обучался в Британии, где академическая гребля и по сей день
                   4192: является весьма популярным видом спорта.
                   4193: 
                   4194: Источник:
                   4195: "Машины и механизмы", 2014, N 10.
                   4196: 
                   4197: Автор:
                   4198: Сергей Лобачёв (Нижний Новгород)
                   4199: 
                   4200: Вопрос 11:
                   4201: Во время певческих выступлений императора Нерона никому не позволялось
                   4202: выходить из театра. Поэтому иногда во время таких выступлений некоторые
                   4203: люди ДЕЛАЛИ ЭТО. Тауэрские вороны иногда ДЕЛАЮТ ЭТО, злостно подшучивая
                   4204: над своим смотрителем. Что мы заменили словами "ДЕЛАТЬ ЭТО"?
                   4205: 
                   4206: Ответ:
                   4207: Притворяться мертвым.
                   4208: 
                   4209: Зачет:
                   4210: Притворяться мертвыми; Симулировать смерть.
                   4211: 
                   4212: Комментарий:
                   4213: Люди притворялись мертвыми, чтобы их выносили из театра. История
                   4214: умалчивает относительно дальнейшей судьбы таких хитрецов. Тауэрские
                   4215: вороны же подобным образом играют на нервах своего смотрителя.
                   4216: 
                   4217: Источник:
                   4218:    1. http://www.bards.ru/press/press_show.php?id=238
                   4219:    2. "Англия в общем и в частности", 2-я серия, 15-я минута.
                   4220: 
                   4221: Автор:
                   4222: Ярослав Косарев (Нижний Новгород)
                   4223: 
                   4224: Вопрос 12:
                   4225: По словам Юрия Лопухина, репутация российских специалистов настолько
                   4226: высока, что сотрудники лаборатории при этом учреждении время от времени
                   4227: посещают Ханой. Назовите это учреждение двумя словами.
                   4228: 
                   4229: Ответ:
                   4230: Мавзолей Ленина.
                   4231: 
                   4232: Комментарий:
                   4233: Упомянутые сотрудники периодически приводят в порядок забальзамированное
                   4234: тело Хо Ши Мина.
                   4235: 
                   4236: Источник:
                   4237: http://www.mk.ru/science/article/2010/05/18/491259-ho-shi-min-umiral-pod-obektivom-kinokameryi-foto.html
                   4238: 
                   4239: Автор:
                   4240: Сергей Лобачёв (Нижний Новгород)
                   4241: 
                   4242: Вопрос 13:
                   4243: В фантастическом романе "Свидание с Рамой" говорится, что к
                   4244: определенному моменту римский и греческий ИКСЫ закончились. Какое слово
                   4245: мы заменили на ИКС?
                   4246: 
                   4247: Ответ:
                   4248: Пантеон.
                   4249: 
                   4250: Комментарий:
                   4251: Персонажи греческой и римской мифологии закончились, и называть
                   4252: космические объекты пришлось в честь богов других культур. Таким образом
                   4253: очередной космический объект получил имя "Рама".
                   4254: 
                   4255: Источник:
                   4256: Артур Кларк. Свидание с Рамой. http://lib.ru/KLARK/rama1.txt
                   4257: 
                   4258: Автор:
                   4259: Сергей Лобачёв (Нижний Новгород)
                   4260: 
                   4261: Вопрос 14:
                   4262: Адвокат "Ювентуса", оспаривая дисквалификацию Златана Ибрагимовича,
                   4263: утверждал, что того неоднократно провоцировали соперники. При этом
                   4264: адвокат воспользовался услугами человека, который умеет ДЕЛАТЬ ЭТО.
                   4265: Согласно одному сайту, учиться ДЕЛАТЬ ЭТО "можно всегда и везде: в кафе,
                   4266: в метро, на улице, на работе, на отдыхе". Что мы заменили словами
                   4267: "ДЕЛАТЬ ЭТО"?
                   4268: 
                   4269: Ответ:
                   4270: Читать по губам.
                   4271: 
                   4272: Комментарий:
                   4273: Человек, умеющий читать по губам, при просмотре записи матча смог
                   4274: засвидетельствовать, какие именно фразы говорили соперники Златану.
                   4275: Согласно упомянутому сайту, чтобы научиться читать по губам, нужно
                   4276: смотреть на людей вокруг и пытаться понять то, что они говорят.
                   4277: 
                   4278: Источник:
                   4279:    1. Златан Ибрагимович. Я - Златан.
                   4280: http://www.sports.ru/tribuna/blogs/legaserie/556064.html
                   4281:    2. http://www.kak-bog.ru/kak-nauchitsya-chitat-po-gubam
                   4282: 
                   4283: Автор:
                   4284: Ярослав Косарев (Нижний Новгород)
                   4285: 
                   4286: Вопрос 15:
                   4287: По одной из версий, ПЕРВАЯ символизирует две скрещенные руки, а ВТОРАЯ
                   4288: была выбрана за сходство с ладонью. Изобразите ПЕРВУЮ и ВТОРУЮ в
                   4289: правильном порядке.
                   4290: 
                   4291: Ответ:
                   4292: XV.
                   4293: 
                   4294: Зачет:
                   4295: X, V.
                   4296: 
                   4297: Комментарий:
                   4298: Речь идет о римских цифрах.
                   4299: 
                   4300: Источник:
                   4301: Айзек Азимов. О времени, пространстве и других вещах.
                   4302: http://www.flibusta.net/b/358185/read
                   4303: 
                   4304: Автор:
                   4305: Сергей Лобачёв (Нижний Новгород)
                   4306: 
                   4307: Вопрос 16:
                   4308: Фрэнк Стэннард считал, что объекты, находящиеся в двух параллельных
                   4309: вселенных, могут существовать не сталкиваясь между собой. При этом он
                   4310: упоминает две пары соперников, играющих... Во что?
                   4311: 
                   4312: Ответ:
                   4313: В шашки.
                   4314: 
                   4315: Комментарий:
                   4316: На черных и белых полях одной доски могут проводиться две партии
                   4317: одновременно.
                   4318: 
                   4319: Источник:
                   4320: Джеймс Линдон. Как окрестить червя.
                   4321: http://www.flibusta.net/b/364464/read
                   4322: 
                   4323: Автор:
                   4324: Сергей Лобачёв (Нижний Новгород)
                   4325: 
                   4326: Вопрос 17:
                   4327: В фильме "Бесы" есть сцена нелегальной сходки. Одна опасливая героиня
                   4328: перед важным обсуждением просит ИКСА приступить к делу. 23 марта 2003
                   4329: года "ИКСА" обошло "Чикаго". Какое слово мы заменили на ИКС?
                   4330: 
                   4331: Ответ:
                   4332: Пианист.
                   4333: 
                   4334: Комментарий:
                   4335: Героиня просила начать играть, чтобы обсуждение революционеров никто не
                   4336: подслушал. В 2003 году "Оскар" за лучший фильм получил мюзикл "Чикаго",
                   4337: а "Пианист" Романа Полански был среди номинантов.
                   4338: 
                   4339: Источник:
                   4340:    1. Сериал "Бесы", реж. В. Хотиненко.
                   4341:    2. http://ru.wikipedia.org/wiki/Оскар_(кинопремия,_2003)
                   4342: 
                   4343: Автор:
                   4344: Ярослав Косарев (Нижний Новгород)
                   4345: 
                   4346: Вопрос 18:
                   4347: В средние века в Англии считали, что того, кто весел и крепок духом,
                   4348: минует ужасная участь. Поэтому ОН, по мнению Игоря Шайтанова, - это не
                   4349: метафора, а довольно частое явление в те времена. Назовите ЕГО.
                   4350: 
                   4351: Ответ:
                   4352: Пир во время чумы.
                   4353: 
                   4354: Комментарий:
                   4355: Согласно распространенным в те времена поверьям, спасти от чумы могут не
                   4356: только снадобья, но и хорошее морально-психологическое состояние,
                   4357: которому способствовали различные увеселения.
                   4358: 
                   4359: Источник:
                   4360: И. Шайтанов. Шекспир. http://www.flibusta.net/b/366509/read
                   4361: 
                   4362: Автор:
                   4363: Ярослав Косарев (Нижний Новгород)
                   4364: 
                   4365: Вопрос 19:
                   4366: Внимание, в вопросе есть замена.
                   4367:    "Географические наименования в честь ученых, государственных и
                   4368: общественных деятелей, военачальников и воинов, корифеев искусства и
                   4369: мастеров литературы - это, образно говоря, вторая серия". Какие три
                   4370: слова мы заменили одним в этой цитате из книги Эдуарда Вартаньяна?
                   4371: 
                   4372: Ответ:
                   4373: Жизнь замечательных людей.
                   4374: 
                   4375: Комментарий:
                   4376: Заменено было слово "серия". "Жизнь замечательных людей" - серия
                   4377: биографических книг о жизни замечательных людей.
                   4378: 
                   4379: Источник:
                   4380: Э. Вартаньян. История с географией, или Жизнь и приключения
                   4381: географических названий. http://www.flibusta.net/b/106518/read
                   4382: 
                   4383: Автор:
                   4384: Ярослав Косарев (Нижний Новгород)
                   4385: 
                   4386: Вопрос 20:
                   4387: Одно из самых своих известных произведений в жанре поэзии ЭТОТ ЧЕЛОВЕК
                   4388: написал под псевдонимом C.3.3 [си три три]. Назовите ЭТОГО ЧЕЛОВЕКА.
                   4389: 
                   4390: Ответ:
                   4391: [Оскар] Уайлд.
                   4392: 
                   4393: Комментарий:
                   4394: Речь идет о "Балладе Редингской тюрьмы", написанной Оскаром Уайлдом.
                   4395: Уайлд отбывал в этой тюрьме двухлетний срок, и C.3.3 - это его номер
                   4396: заключенного, который означал третью камеру третьей площадки галереи C.
                   4397: 
                   4398: Источник:
                   4399: http://ru.wikipedia.org/wiki/Баллада_Редингской_тюрьмы
                   4400: 
                   4401: Автор:
                   4402: Ярослав Косарев (Нижний Новгород)
                   4403: 
                   4404: Вопрос 21:
                   4405: Герой романа Нила Стивенсона упоминает барабанный бой и сравнивает ЕЕ с
                   4406: исполинской сороконожкой. Назовите ЕЕ одним словом.
                   4407: 
                   4408: Ответ:
                   4409: Галера.
                   4410: 
                   4411: Зачет:
                   4412: Триера.
                   4413: 
                   4414: Комментарий:
                   4415: Барабаны задавали ритм гребцам, налегавшим на весла. Весельная галера
                   4416: имеет сходство с сороконожкой.
                   4417: 
                   4418: Источник:
                   4419: Нил Стивенсон. Король бродяг. http://www.flibusta.net/b/365885/read
                   4420: 
                   4421: Автор:
                   4422: Сергей Лобачёв (Нижний Новгород)
                   4423: 
                   4424: Вопрос 22:
                   4425: Компания "Caterpillar" [катерпИллер] в качестве демонстрации точности
                   4426: своей техники провела показ игры в гигантскую АЛЬФУ. Назовите АЛЬФУ
                   4427: словом, пришедшим из языка суахили.
                   4428: 
                   4429: Ответ:
                   4430: Дженга.
                   4431: 
                   4432: Комментарий:
                   4433: "Caterpillar" [катерпИллер] - компания, являющаяся одним из мировых
                   4434: лидеров по производству спецтехники. В рекламном ролике показано, как
                   4435: погрузчик и экскаватор играют в дженгу, двигая огромные деревянные
                   4436: блоки.
                   4437: 
                   4438: Источник:
                   4439:    1. http://metkere.com/2014/04/catjenga.html
                   4440:    2. http://en.wikipedia.org/wiki/Jenga
                   4441: 
                   4442: Автор:
                   4443: Сергей Лобачёв (Нижний Новгород)
                   4444: 
                   4445: Вопрос 23:
                   4446: Олдос Хаксли скончался 22 ноября 1963 года. Рассказывая об этом, Дмитрий
                   4447: Шушарин упоминает другого деятеля культуры. Назовите его фамилию.
                   4448: 
                   4449: Ответ:
                   4450: Прокофьев.
                   4451: 
                   4452: Комментарий:
                   4453: Смерть Олдоса Хаксли затмило более громкое событие - убийство Джона
                   4454: Кеннеди. Схожая ситуация имела место в 1953 году, когда в один день
                   4455: скончались Иосиф Сталин и Сергей Прокофьев.
                   4456: 
                   4457: Источник:
                   4458:    1. http://en.wikipedia.org/wiki/Aldous_Huxley
                   4459:    2. http://en.wikipedia.org/wiki/John_F._Kennedy
                   4460:    3. https://www.facebook.com/dshush/posts/715909355103949/
                   4461: 
                   4462: Автор:
                   4463: Сергей Лобачёв (Нижний Новгород)
                   4464: 
                   4465: Вопрос 24:
                   4466: Согласно одной книге, будучи выходцем из знатной семьи, Иван Тургенев
                   4467: познакомился с ИКСОМ на улице. Звучит, наверное, странно, но в "ИКСЕ"
                   4468: всего 50 слов. Что мы заменили на ИКС?
                   4469: 
                   4470: Ответ:
                   4471: Русский язык.
                   4472: 
                   4473: Комментарий:
                   4474: В доме Тургеневых говорили на французском языке. Во втором предложении
                   4475: имеется в виду стихотворение в прозе "Русский язык" авторства того же
                   4476: Тургенева.
                   4477: 
                   4478: Источник:
                   4479:    1. Литературная матрица. Учебник, написанный писателями. Том 1.
                   4480: http://www.flibusta.net/b/215508/read
                   4481:    2. http://ru.wikisource.org/wiki/Русский_язык_(Тургенев)
                   4482: 
                   4483: Автор:
                   4484: Ярослав Косарев (Нижний Новгород)
                   4485: 
                   4486: Вопрос 25:
                   4487: В одном романе, действие которого происходит в Нидерландах, девушка, с
                   4488: большим трудом пытающаяся устоять на коньках, сравнивается с НЕЮ.
                   4489: Назовите ЕЕ.
                   4490: 
                   4491: Ответ:
                   4492: Мельница.
                   4493: 
                   4494: Зачет:
                   4495: Ветряная мельница.
                   4496: 
                   4497: Комментарий:
                   4498: Героиня активно машет руками, словно крыльями мельницы.
                   4499: 
                   4500: Источник:
                   4501: Нил Стивенсон. Король бродяг. http://www.flibusta.net/b/365885/read
                   4502: 
                   4503: Автор:
                   4504: Сергей Лобачёв (Нижний Новгород)
                   4505: 
                   4506: Вопрос 26:
                   4507: Герой романа "Последний конунг" ДЕЛАЕТ ЭТО перед военным походом.
                   4508: Богатые китайцы, стараясь подчеркнуть свое благородное происхождение,
                   4509: ДЕЛАЛИ ЭТО крайне редко. Какие два слова мы заменили словами "ДЕЛАТЬ
                   4510: ЭТО"?
                   4511: 
                   4512: Ответ:
                   4513: Стричь ногти.
                   4514: 
                   4515: Зачет:
                   4516: Подстригать ногти; Состригать ногти; Обрезать ногти.
                   4517: 
                   4518: Комментарий:
                   4519: Герой предчувствует свою скорую гибель и желает всячески отсрочить
                   4520: постройку Нагльфара. Богатые китайцы ходили с длинными ногтями,
                   4521: демонстрируя свое пренебрежение к тяжелому труду.
                   4522: 
                   4523: Источник:
                   4524:    1. Тим Северин. Последний конунг.
                   4525: http://www.flibusta.net/b/95047/read
                   4526:    2. http://www.stunningbeauty.ru/index.php/menu-nails-aticles/58-aticle-nails-a4
                   4527: 
                   4528: Автор:
                   4529: Сергей Лобачёв (Нижний Новгород)
                   4530: 
                   4531: Вопрос 27:
                   4532: Небольшой английский парк ТвИкенхэм за свое биологическое разнообразие
                   4533: получил прозвище "ИКС природы". Главный герой известного кинофильма,
                   4534: увидев ИКС, перестал обращать внимание на окружающую обстановку.
                   4535: Назовите ИКС.
                   4536: 
                   4537: Ответ:
                   4538: Конспект.
                   4539: 
                   4540: Комментарий:
                   4541: В этом небольшом парке наблюдается такое разнообразие флоры, что его
                   4542: назвали "конспектом природы". Упомянутый известный кинофильм - "Операция
                   4543: "Ы"".
                   4544: 
                   4545: Источник:
                   4546: Наталья Юдина. 100 великих заповедников и парков.
                   4547: http://www.flibusta.net/b/175126/read
                   4548: 
                   4549: Автор:
                   4550: Ярослав Косарев (Нижний Новгород)
                   4551: 
                   4552: Вопрос 28:
                   4553: Согласно одной книге, деятельности ЭТОГО ЧЕЛОВЕКА мешали распускавшиеся
                   4554: слухи, согласно которым у людей могли появиться рога, копыта и хвост.
                   4555: Назовите ЭТОГО ЧЕЛОВЕКА.
                   4556: 
                   4557: Ответ:
                   4558: [Эдвард] Дженнер.
                   4559: 
                   4560: Комментарий:
                   4561: Дженнер прививал людям коровью оспу. Недоброжелатели и скептики
                   4562: рассказывали о том, что вместе с оспой человек может стать обладателем и
                   4563: других атрибутов коровы.
                   4564: 
                   4565: Источник:
                   4566: Елена Грицак. Популярная история медицины.
                   4567: http://www.flibusta.net/b/113382/read
                   4568: 
                   4569: Автор:
                   4570: Ярослав Косарев (Нижний Новгород)
                   4571: 
                   4572: Вопрос 29:
                   4573: Египетского бога Нила по имени Хапи изображали с НЕЙ, что было знаком
                   4574: плодородия. Может прозвучать странно, но у матадора РивьЕро Эль ПримЕро
                   4575: была тайна, заключавшаяся в избавлении от НЕЕ. Назовите ЕЕ двумя
                   4576: словами.
                   4577: 
                   4578: Ответ:
                   4579: Женская грудь.
                   4580: 
                   4581: Комментарий:
                   4582: Ривьеро Эль Примеро - это женщина, которая ради карьеры матадора
                   4583: выдавала себя за мужчину. От груди ей пришлось избавиться, потому что
                   4584: иначе ее тайна была бы быстро раскрыта.
                   4585: 
                   4586: Источник:
                   4587:    1. Мэрилин Ялом. История груди. http://www.flibusta.net/b/359437/read
                   4588:    2. http://www.maximonline.ru/skills/sex/_article/putevoditel-po-zhenskoj-grudi/
                   4589: 
                   4590: Автор:
                   4591: Ярослав Косарев (Нижний Новгород)
                   4592: 
                   4593: Вопрос 30:
                   4594: Основатели Яндекса Аркадий ВОлож и Илья СегалОвич, поступив в московские
                   4595: вузы, написали письмо своему бывшему классному руководителю. Забавно,
                   4596: что, описывая степень своего недовольства столичной жизнью, они
                   4597: упомянули ЭТО. Напишите ЭТО, используя пять символов.
                   4598: 
                   4599: Ответ:
                   4600: 10^100.
                   4601: 
                   4602: Зачет:
                   4603: Гугол; 100^50.
                   4604: 
                   4605: Комментарий:
                   4606: Провинциальная школьная жизнь Воложу и Сегаловичу нравилась намного
                   4607: больше, чем московская студенческая. Десять в сотой степени - это число
                   4608: "googol", от названия которого произошло название компании "Google". Как
                   4609: известно, "Google" и "Яндекс" - конкуренты.
                   4610: 
                   4611: Источник:
                   4612: Дмитрий Соколов-Митрич. Яндекс.Книга.
                   4613: http://www.flibusta.net/b/379792/read
                   4614: 
                   4615: Автор:
                   4616: Ярослав Косарев (Нижний Новгород)
                   4617: 
                   4618: Вопрос 31:
                   4619: При описании голландских лугов Всеволод Овчинников упоминает ЕГО. На
                   4620: одном фешенебельном судне ОН оборудован электроприводом и высокоточными
                   4621: гироскопами, что позволяет использовать ЕГО даже во время качки.
                   4622: Назовите ЕГО.
                   4623: 
                   4624: Ответ:
                   4625: Бильярдный стол.
                   4626: 
                   4627: Комментарий:
                   4628: Большая часть территории Нидерландов лишена гор и возвышенностей,
                   4629: поэтому ее зеленые луга напомнили Всеволоду Овчинникову ровную
                   4630: поверхность бильярдного стола. На круизном судне "Сияние морей"
                   4631: бильярдные столы оборудованы механизмами, практически полностью
                   4632: компенсирующими качку.
                   4633: 
                   4634: Источник:
                   4635:    1. http://www.rg.ru/2011/08/04/yaponcy.html
                   4636:    2. http://www.geek.com/geek-cetera/gyroscope-used-for-self-leveling-cruise-ship-pool-table-1467721/
                   4637: 
                   4638: Автор:
                   4639: Сергей Лобачёв (Нижний Новгород)
                   4640: 
                   4641: Вопрос 32:
                   4642: В сериале "Демоны да Винчи" представитель Ватикана предлагает художнику
                   4643: предать Флоренцию в обмен на доступ в секретный архив Ватикана. Далее
                   4644: этот герой сравнивает себя с ИКСОМ. Назовите ИКСА, используя дефис.
                   4645: 
                   4646: Ответ:
                   4647: Змей-искуситель.
                   4648: 
                   4649: Комментарий:
                   4650: А ватиканский архив при этом сравнивается с древом познания.
                   4651: 
                   4652: Источник:
                   4653: Сериал "Демоны да Винчи", s01e02, 47-я минута (в озвучке FOX).
                   4654: 
                   4655: Автор:
                   4656: Ярослав Косарев (Нижний Новгород)
                   4657: 
                   4658: Вопрос 33:
                   4659: В самом начале легенды о Поле Баньяне - гигантском дровосеке из
                   4660: американского фольклора - говорится, что потребовалось целых пять ИХ.
                   4661: Назовите ИХ одним словом.
                   4662: 
                   4663: Ответ:
                   4664: Аисты.
                   4665: 
                   4666: Комментарий:
                   4667: Чтобы доставить такого крепыша родителям, одного аиста явно не хватило
                   4668: бы.
                   4669: 
                   4670: Источник:
                   4671: http://www.americanfolklore.net/folklore/2010/07/the_birth_of_paul_bunyan.html
                   4672: 
                   4673: Автор:
                   4674: Сергей Лобачёв (Нижний Новгород)
                   4675: 
                   4676: Вопрос 34:
                   4677: Ответьте двумя словами, что король Фридрих II Гогенцоллерн, правивший в
                   4678: восемнадцатом веке, назвал "артишоком, который предназначен для того,
1.2       rubashki 4679: чтобы его съели лист за листом".
1.1       rubashki 4680: 
                   4681: Ответ:
                   4682: Речь Посполитая.
                   4683: 
                   4684: Комментарий:
                   4685: В восемнадцатом веке было три раздела Речи Посполитой, в одном из
                   4686: которых успел поучаствовать Фридрих.
                   4687: 
                   4688: Источник:
                   4689: Ниал Фергюсон. Цивилизация: чем Запад отличается от остального мира.
                   4690: 
                   4691: Автор:
                   4692: Ярослав Косарев (Нижний Новгород)
                   4693: 
                   4694: Вопрос 35:
                   4695: (pic: 20140522.jpg)
                   4696:    Почтовое управление Китая выпустило серию марок, одну из которых мы
                   4697: вам раздали. Другая марка этой серии содержит фрагмент известного
                   4698: произведения. Назовите автора этого произведения.
                   4699: 
                   4700: Ответ:
                   4701: [Альбрехт] Дюрер.
                   4702: 
                   4703: Комментарий:
                   4704: Точки на спине черепахи образуют магический квадрат. Другой магический
                   4705: квадрат изображен на известной гравюре Дюрера "Меланхолия".
                   4706: 
                   4707: Источник:
                   4708:    1. http://www.futilitycloset.com/2014/10/16/stamps-and-math/
                   4709:    2. https://www.macaupost.gov.mo/Philately/XVersion/ProductList.aspx?admcode=MAC&emicode=201408&lang=en-us
                   4710: 
                   4711: Автор:
                   4712: Сергей Лобачёв (Нижний Новгород)
                   4713: 
                   4714: Вопрос 36:
                   4715: Действие романа "КриптономикОн" происходит во время Второй мировой
                   4716: войны. Рассказывая об уроженцах островка Йглм [ийглым], расположенного у
                   4717: берегов Великобритании, герой романа упоминает другую народность.
                   4718: Назовите эту народность.
                   4719: 
                   4720: Ответ:
                   4721: Навахо.
                   4722: 
                   4723: Комментарий:
                   4724: У островитян был свой собственный язык, который был известен только
                   4725: йглымцам. Этот факт позволил использовать жителей острова для передачи
                   4726: секретных сообщений. Сходную роль в ВМФ США выполняли индейцы племени
                   4727: навахо.
                   4728: 
                   4729: Источник:
                   4730: Нил Стивенсон. Криптономикон.
                   4731: 
                   4732: Автор:
                   4733: Сергей Лобачёв (Нижний Новгород)
                   4734: 

FreeBSD-CVSweb <freebsd-cvsweb@FreeBSD.org>